You are on page 1of 69

SRI MARUTHI P.

U COLLEGE, HOSKOTE I PUC PART-2 CET/NEET STUDY


MATERIAL
Waves/Sound, Oscillations/SHM, Mechanical Properties of Matter/Elasticity, Fluid Mechanics (Fluid
Statics, Fluid Dynamics, Viscosity, Surface Tension),
Thermal Properties Matter/Thermometry (Thermal Expansion, Calorimetry, Transmission of Heat),
Thermo Dynamics, Kinetic Theory of Gases (KTG)

1
1.WAVE MOTION/SOUND (c) Path difference 2 2 2 = 4m
1. Velocity of sound waves in air is 330 m/sec. For
a particular sound in air, a path difference of 40 Hence v n = 120 4 480 m/ s
6 Velocity of sound measured in hydrogen and
cm is equivalent to a phase difference of 1.6 .
oxygen gas at a given temperature will be in the
The frequency of this wave is ratio
(a) 165 Hz (b) 150 Hz (a) 1 : 4 (b) 4 : 1
(c) 660 Hz (d) 330 Hz (c) 2 : 1 (d) 1 : 1
2 vH 2 M O2
1 32 vH 2 4
v
Ans (c)Phase difference = path difference M vO2 M H2 2 vO2 1
(b)
2
1.6 40
50 cm= 0.5m 7 The equation of a wave is y 2 sin (0.5x 200t) ,
where x and y are expressed in cm and t in sec.
330 0.5 n n 660Hz
v n The wave velocity is
2. The ratio of the speed of sound in nitrogen gas (a) 100 cm/sec (b) 200 cm/sec
to that in helium gas, at 300 K is (c) 300 cm/sec (d) 400 cm/sec
(d)Comparing given equation with standard
equation of progressive wave. The velocity of
(a) 2 / 7 (b) 1/ 7
wave
(c) 3 / 5 (d) 6 / 5 (Co - efficient
of t) 200
v 400cm/ s
of x) 0.5
k (Co - efficient
RT T
v v 8 A plane wave is represented by
(c)Velocity of s-ound in gas M M

7
R 4 x 1.2 sin(314t 12.56y)
vN2 N2 M 5 3
He Where x and y are distances measured along in x
vHe He M H2 5 5
R 28 and y direction in meters and t is
3
time in seconds. This wave has
3. In a sinusoidal wave, the time required for a
(a) A wavelength of 0.25 m and travels in +
particular point to move from maximum
ve x direction
displacement to zero displacement is 0.170
second. The frequency of the wave is (b) A wavelength of 0.25 m and travels in +
ve y direction
(a) 1.47 Hz (b) 0.36 Hz
(c) A wavelength of 0.5 m and travels in ve y
(c) 0.73 Hz (d) 2.94 Hz
direction
(a)Time required for a point to move from
(d) A wavelength of 0.5 m and travels in ve x
maximum displacement to zero displacement is direction
T 1 (c)The given equation representing a wave
t
4 4n travelling along y direction (because + sign is
given between t term and x term).
1 1
n 1.47Hz x A sin t ky
4t 4 0.170 On comparing it with
2 2 3.14
4 If the density of oxygen is 16 times that of k 12.56 0.5 m
We get 12.56
hydrogen, what will be the ratio of their
corresponding velocities of sound waves 9 The displacement y (in cm) produced by a
(a) 1 : 4 (b) 4 : 1 10 x
y sin 2000t
17
(c) 16 : 1 (d) 1 : 16 simple harmonic wave is . The
periodic time and maximum velocity of the
P vO H 2 1 1
v 2 particles in the medium will respectively be
o2vH 2 16 4
(a) 3
(a) 10 sec and 330 m/sec (b)
5 The phase difference between two points 4
10 sec and 20 m/sec
separated by 1m in a wave of frequency 120 Hz is
3
90o . The wave velocity is (c) 10 sec and 200 m/sec (d)
2
(a) 180 m/s (b) 240 m/s 10 sec and 2000 m/sec
(c) 480 m/s (d) 720 m/s 10
a , 200
(c)Comparing with y a sin(t kx)
SRI MARUTHI P.U COLLEGE, HOSKOTE I PUC PART-2 CET/NEET STUDY
MATERIAL
Waves/Sound, Oscillations/SHM, Mechanical Properties of Matter/Elasticity, Fluid Mechanics (Fluid
Statics, Fluid Dynamics, Viscosity, Surface Tension),
Thermal Properties Matter/Thermometry (Thermal Expansion, Calorimetry, Transmission of Heat),
Thermo Dynamics, Kinetic Theory of Gases (KTG)
10 (b) Its wavelength is 100 cm
vmax a 2000 200m/ sec
(c) Its frequency is 50 cycles/sec
2 2 3
200 (d) Its propagation velocity is 50 10 cm/sec
and T T T 103 sec
t x
10 The equation of a transverse wave is given by y 4 sin2
0.02 100 .
y 10sin (0.01x 2t) (d)
where x and y are in cm and t is in second. Its t x
y a sin2
frequency is Comparing this equation with T
1
1 2 sec Co - efficient
of t 1 / 0.02
(a) 10sec (b) v
Co - efficient
of x 1 / 100
1 sec1 0.01sec1
(c) (d) 2
(c)Comparing with the standard equation, y 0.08sin (200t x)
15 If the wave equation then
2 the velocity of the wave will be
y A sin (vt x)
, we have
(a) 400 2 (b) 200 2
v
n 1 (c) 400 (d) 200
v 200 cm/ sec 200 cm; sec1
, (d) Comparing with standard wave equation
11 The equation of a wave is y 2 sin (0.5x 200t) , 2
y a sin (vt x)
where x and y are expressed in cm and t in sec. v 200m / s.
, we get,
The wave velocity is
16 Two sound waves (expressed in CGS units)
(a) 100 cm/sec (b) 200 cm/sec
2 2
(c) 300 cm/sec (d) 400 cm/sec y1 0.3 sin (vt x) y2 0.4 sin (vt x )
given by and
(d)Comparing given equation with standard
equation of progressive wave. The velocity of interfere. The resultant amplitude at a place where
wave phase difference is / 2 will be
(Co - efficient
of t) 200
v 400cm/ s
of x) 0.5
k (Co - efficient (a) 0.7 cm (b) 0.1 cm
12 The equation of progressive wave is 1
7 cm
y a sin(200t x) (c) 0.5 cm (d) 10
. where x is in meter and t is in
second. The velocity of wave is
a12 a22 2a1a2 cos
(a) 200 m/sec (b) 100 m/sec (c) Resultant amplitude =
(c) 50 m/sec (d) None of these
0.32 0.42 2 0.3 0.4 cos
(a)comparing the given equation with y a sint kx = 2 = 0.5 cm
17 The superposing waves are represented by the
v 200
200, k 1 k m/ s following equations :
so
13 Two waves represented by the following y1 5 sin2 (10t 0.1x) y2 10sin2 (20t 0.2x)
,
equations are travelling in the same medium
I max
y1 5 sin2 (75t 0.25x) y2 10sin2 (150t 0.50x)
, I min
Ratio of intensities will be
I1 / I 2 (a) 1 (b) 9
The intensity ratio of the two waves is
(a) 1 : 2 (b) 1 : 4 (c) 4 (d) 16
2
(c) 1 : 8 (d) 1 : 16 I max (a1 a2)2 5 10 9

a1 5, a2 10 Imin (a1 a2)2 5 10 1
I1 a12 I 25 1 (b)
2 1
I 2 a2 I 2 100 4
(b)
18 If the ratio of amplitude of wave is 2 : 1, then
the ratio of maximum and minimum intensity is
(a) 9 : 1 (b) 1 : 9
(c) 4 : 1 (d) 1 : 4
14 Which of the following is not true for this 2
a1
1
t x I max a 2 1
2
y 4 sin2 2 9 /1

progressive wave 0.02 100 where y and I min

a1
1 2 1
x are in cm & t in sec (a) a2

(a) Its amplitude is 4 cm


SRI MARUTHI P.U COLLEGE, HOSKOTE I PUC PART-2 CET/NEET STUDY
MATERIAL
Waves/Sound, Oscillations/SHM, Mechanical Properties of Matter/Elasticity, Fluid Mechanics (Fluid
Statics, Fluid Dynamics, Viscosity, Surface Tension),
Thermal Properties Matter/Thermometry (Thermal Expansion, Calorimetry, Transmission of Heat),
Thermo Dynamics, Kinetic Theory of Gases (KTG)
19 The two interfering waves have intensities in As is clear from Fig.
the ratio 9 : 4. The ratio of intensities of maxima
and minima in the interference pattern will be

(a) 1 : 25 (b) 25 : 1
(c) 9 : 4 (d) 4 : 9

2
I1
2
1 9
1
I max I2 4 25

Imin I1 9 1
1 2
4
I2
(b)
20 Two tuning forks have frequencies 450 Hz and
454 Hz respectively. On sounding these forks 1
l
together, the time interval between successive
4 1 4l
maximum intensities will be
3 4l
(a) 1/4 sec (b) 1/2 sec l 2 2
(c) 1 sec (d) 2 sec 4 3
(a) The time interval between successive 5 4l
maximum intensities will be l 3
4 5
1 1 1
sec.
n1 ~ n2 454 450 4 1 1
1 : 2 : 3 1 : : .
21 The frequencies of two sound sources are 256 3 5
Hz and 260 Hz. At t = 0, the intensity of sound is 24 Two vibrating tuning forks produce progressive waves
maximum. Then the phase difference at the time t given by y1 = 4 sin 500 t, y2 = 2 sin 506 t and held near
= 1/16 sec will be the ear of a person. Number of beats heard per minute is
(a) 180 (b) 3 (c) 360 (d) 60
(a) Zero (b)
Ans(a)
(c) /2 (d) /4
Here, 1=
2 n1 500 n1 250 hz
(c) Time interval between two consecutive beats
1 1 1 1 T
w 2 2n2 506,n2 253hz
T sec t sec
n1 n2 260 256 4
so, 16 4 No.of beats/sec n2 n1 253 250 3hz
T
No. of beats/min.= 3 60 180
By using time difference = 2 Phase difference
T T 25 The length of a sonometer wire AB is 110 cm. where

4 2 2 should the two bridges be placed from A to divide the wire
22 Beats are produced by two waves given by in three segments whose fundamental frequencies are in
y1 a sin2000t y a sin2008t the ratio of 1: 2 : 3
and 2 . The number of
(a) 30 cm, 90cm (b) 60 cm, 90 cm
beats heard per second is
(c) 40 cm, 70 cm (d) None of these
(a) Zero (b) One
(c) Four (d) Eight Ans (b)

(c)Number of beats per second =


n1 ~ n2 As n1 : n 2 : n 3 1 : 2 : 3
1 2000 2n1
n1 = 1000 1 1 1
l1 : l 2 : l 3 : : 6:3:2
2 2008 2n2 1 2 3
and n2 = 1004 sum of the ratios =6+3+2=11
Number of beats heard per sec 1004 1000 4 110
l1 6 60cm
23 If 1 : 2 : 3 are the wavelengths of the waves giving 11
resonance with the fundamental, first and second
overtones respectively of a closed organ pipe, then the 110
l2 3 30cm
ratio of 1 : 2 : 3 is 11
1:
1 1
:
wedges should be placed from A at 60 cm and
(a) 1: 3 : 5(b) 1: 2 : 3(c) 5 : 3 :1(d) 3 5 30+60=90 cm.
Ans(d) 26. When two tuning forks (fork 1 and fork 2) are sounded
SRI MARUTHI P.U COLLEGE, HOSKOTE I PUC PART-2 CET/NEET STUDY
MATERIAL
Waves/Sound, Oscillations/SHM, Mechanical Properties of Matter/Elasticity, Fluid Mechanics (Fluid
Statics, Fluid Dynamics, Viscosity, Surface Tension),
Thermal Properties Matter/Thermometry (Thermal Expansion, Calorimetry, Transmission of Heat),
Thermo Dynamics, Kinetic Theory of Gases (KTG)
simultaneously, 4 beats per second are heard. Now, some 100
v 104
tape is attached on the prong of the fork 2. When the k 0.01 m/ s

tuning forks are sounded again, 6 beats per second are
30 Two sinusoidal waves with same wavelengths
heard. If the frequency of fork 1 is 200 Hz, then what was
the original frequency of fork 2 ?
and amplitudes travel in opposite directions along
(a) 204 Hz(b) 196 Hz (c) 202 Hz(d) 200 Hz a string with a speed 10 ms1. If the minimum time
interval between two instants when the string is
1n n 4
2 flat is 0.5 s, the wavelength of the waves is
Ans (b)Here,
when mass of 2nd tuning fork increases, n2 (a) 25 m (b) 20 m
decreases. As no. of beats/ sec increases, therefore, (c) 15 m (d) 10 m
n1 > n2
(d)Minimum time interval between two instants
n 2 n1 4 200 4 196 Hz. T
0.5 sec T 1 sec
x when the string is flat = 2
y 4 sin cos(96t)
27 For the stationary wave 15 , the v T 10 1 10m
Hence .
distance between a node and the next antinode is 31 A string vibrates according to the equation
2x
(a) 7.5 (b) 15 y 5 sin cos20t
3 , where x and y are in cm and t
(c) 22.5 (d) 30
(a)Comparing given equation with standard in sec. The distance between two adjacent nodes
equation is
(a) 3 cm (b) 4.5 cm
2x 2vt 2
y 2a sin cos 30 (c) 6 cm (d) 1.5 cm
gives us 15
2x
Distance between nearest node and antinodes = y 5 sin cos20 t,
(d) 3 comparing with equation
30
7.5
4 4 2x 2vt
y 2asin cos
28 The equation of a stationary wave is 3, distance between two
x
y 0.8 cos sin200t adjacent nodes / 2 1.5cm.
20 , where x is in cm and t is in 32 Two similar sonometer wires given fundamental
sec. The separation between consecutive nodes frequencies of 500Hz. These have same tensions.
will be By what amount the tension be increased in one
(a) 20 cm (b) 10 cm wire so that the two wires produce 5 beats/sec
(c) 40 cm (d) 30 cm (a) 1% (b) 2%
(a)On comparing the given equation with standard
(c) 3% (d) 4%
2x 2vt
y 2a sin cos (b)To produce 5 beats/sec. Frequency of one wire
equation ]
should be increase up to 505 Hz . i.e. increment of
2
40 1% in basic frequency.
We get 20 T n
Separation between two consecutive nodes = 2
n T or T n2 T n
40
20 cm 2(1%) 2%
2 2 percentage change in Tension =
29 A standing wave is represented by 33 A string is producing transverse vibration
Y A sin(100t) cos(0.01x) y 0.021sin(x 30t)
whose equation is , Where x and y
where Y and A are in millimetre, t is in seconds are in meters and t is in seconds. If the linear
and x is in metre. 4
The velocity of wave is density of the string is 1.3 10 kg/m, then the

104 m/ s
tension in the string in N will be
(a) (a) 10 (b) 0.5
1m/ s (c) 1 (d) 0.117
(b)
30
104 m/ s v 30m / s
(c)
(d) y 0.021sin(x 30t) k 1 .
(d) Not derivable from above data
T T
(a) By comparing given equation with v 30 T 0.117
Using, m 1.3 10 4 N
y a sin(t) coskx
SRI MARUTHI P.U COLLEGE, HOSKOTE I PUC PART-2 CET/NEET STUDY
MATERIAL
Waves/Sound, Oscillations/SHM, Mechanical Properties of Matter/Elasticity, Fluid Mechanics (Fluid
Statics, Fluid Dynamics, Viscosity, Surface Tension),
Thermal Properties Matter/Thermometry (Thermal Expansion, Calorimetry, Transmission of Heat),
Thermo Dynamics, Kinetic Theory of Gases (KTG)
34 Four wires of identical length, diameters and of (c) 500 Hz (d) 1000 Hz
the same material are stretched on a sonometre n T
wire. If the ratio of their tensions is 1 : 4 : 9 : 16
(c) n T n 2T
then the ratio of their fundamental frequencies are
If tension increases by 2%, then frequency must
increases by 1%.
(a) 16 : 9 : 4 : 1 (b) 4 : 3 : 2 : 1
(c) 1 : 4 : 2 : 16 (d) 1 : 2 : 3 : 4 n1 n
If initial frequency then final frequency n2 n1
=5
101
(d) n T n n 5
100 n 500Hz.
n : n : n : n 1 : 4 : 9 : 16 1 : 2 : 3 : 4 Short trick : If you can remember then apply
1 2 3 4
35 In order to double the frequency of the following formula to solve such type of problems.
fundamental note emitted by a stretched string, Initial frequency of each wire (n)
3 (Numberof beatsheardpersec) 200

4
the length is reduced to th of the original length (percentage
change in tension
of thewire)
and the tension is changed. The factor by which 5 200
the tension is to be changed, is n 500Hz
Here 2
3 2
39 If the length of a stretched string is shortened
(a) 8 (b) 3 by 40% and the tension is increased by 44%, then
8 9 the ratio of the final and initial fundamental
(c) 9 (d) 4 frequencies is
1 T T (a) 2 : 1 (b) 3 : 2
n n
(d) 2l m l (c) 3 : 4 (d) 1 : 3
2 2 2
T2 n2 l2 3 9
(2)2
T1 n1 l1 4 4

36 Transverse waves of same frequency are (a)Fundamental frequency in case of string is
generated in two steel wires A and B. The 1 T T n' T' l
n n
diameter of A is twice of B and the tension in A is 2l m l n T l'
half that in B. The ratio of velocities of wave in A 3
144
and B is T ' T 0.44T T l' l 0.4l l
putting 100 and 5
1: 3 2 1: 2 2 n' 2
(a) (b)
We get n 1 .
1: 2 2 :1
(c) (d) 40 Three similar wires of frequency n1, n2 and n3
T T are joined to make one wire. Its frequency will be
v
m r 2 1 1 1 1
(b)
n n1 n2 n3 n n1 n2 n3
T v TA rB 1 1 1
(a) (b)
v A . .
r vB TB rA 2 2 2 2 1 1 1 1 1 1 1 1

1
n n1 n2 n3 n12 n22 n32
37 The tension of a stretched string is increased (c) (d) n
by 69%. In order to keep its frequency of vibration
constant, its length must be increased by 1 T
n
(b) 2l m n1l1 n2l2 n3l3 k
(a) 20% (b) 30%
k k k k

(c) 69% (d) 69% l1 l2 l3 l n1 n2 n3 n

T 1 1 1 1
n ........
(b) l l T (As n = constant) n n1 n2 n3

l2 T 169 41 Calculate the frequency of the second harmonic
2 l1 l2 1.3l1 l1 30%
l1 T1 100 l formed on a string of length 0.5 m and mass 2
of 1
38 Two wires are in unison. If the tension in one of 104 kg when stretched with a tension of 20 N
the wires is increased by 2%, 5 beats are produced (a) 274.4 Hz (b) 744.2 Hz
per second. The initial frequency of each wire is (c) 44.72 Hz (d) 447.2 Hz
(a) 200 Hz (b) 400 Hz
SRI MARUTHI P.U COLLEGE, HOSKOTE I PUC PART-2 CET/NEET STUDY
MATERIAL
Waves/Sound, Oscillations/SHM, Mechanical Properties of Matter/Elasticity, Fluid Mechanics (Fluid
Statics, Fluid Dynamics, Viscosity, Surface Tension),
Thermal Properties Matter/Thermometry (Thermal Expansion, Calorimetry, Transmission of Heat),
Thermo Dynamics, Kinetic Theory of Gases (KTG)
2 104 (a) 2.00 m (b) 1.50 m
m kg/ m
(d)Mass per unit length 0.5 4 104 kg/ m (c) 1.00 m (d) 0.50 m
n2 2n1 v v 332
Frequency of 2nd harmonic n1 l 0.5m
(d) For closed pipe 4l 4n 4 166
1 T 1 20
2 447.2Hz 46 If the velocity of sound in air is 350 m/s. Then
2l m 0.5 4 10 4
the fundamental frequency of an open organ pipe
42 Two vibrating strings of the same material but of length 50 cm, will be
lengths L and 2L have radii 2r and r respectively.
(a) 350 Hz (b) 175 Hz
They are stretched under the same tension. Both
(c) 900 Hz (d) 750 Hz
the strings vibrate in their fundamental modes, the
one of length L with frequency n1 and the other (a)Fundamental frequency of open pipe
with frequency n2. The ratio n1/n2 is given by v 350
n1 350
2l 2 0.5 Hz .
(a) 2 (b) 4
(c) 8 (d) 1 47 If the length of a closed organ pipe is 1m and
velocity of sound is 330 m/s, then the frequency
for the second note is
1 T
n 330 330
2l r 2 4 Hz 3 Hz
(d)Fundamental frequency 4 4
(a) (b)
where m = Mass per unit length of wire
330 4
1 n r l r 2L 1 2 Hz 2 Hz
n 1 2 2 (c) 4 (d) 330
lr n2 r1 l1 2r L 1

v 330
43 A string is rigidly tied at two ends and its n1
(b) For closed pipe 4l 4 Hz
y cos2 t sinsinx.
equation of vibration is given by 3 300
Then minimum length of string is 3n1
Second note = 4 Hz .
1 48 The fundamental note produced by a closed
m
(a) 1 m (b) 2
organ pipe is of frequency f. The fundamental
(c) 5 m (d) 2m note produced by an open organ pipe of same
(b)Given equation of stationary wave is length will be of frequency
y sin2x cos2t , comparing it with standard f

y 2A sin
2x
cos
2x (a) 2 (b) f
equation
(c) 2f (d) 4 f
2x
2x v v
nclosed , nopen
We have 1m 4l 2l nopen 2nclosed 2f
(c)
Minimum distance of string (first mode)
P1
1 49 An organ pipe closed at one end vibrating in
Lmin m
2 2 P
its first overtone and another pipe 2 open at both
44 A closed pipe and an open pipe have their first ends vibrating in its third overtone are in
overtones identical in frequency. Their lengths are resonance with a given tuning fork. The ratio of
in the ratio
P P2
lengths of 1 and is
(a) 1 : 2 (b) 2 : 3 (a) 1 : 2 (b) 1 : 3
(c) 3 : 4 (d) 4 : 5 (c) 3 : 8 (d) 3 : 4
3v
n1
(c) If is given that 4l1
(c) First overtone of closed organ pipe
First over tone of closed pipe = First over tone of
4v
v v n2
3 2
2l
2l 2
4l1 2
Third overtone of open organ pipe
open pipe ; where l1 and l2 are the
3v 4v l 3
lengths of closed and open organ pipes hence 1
n1 n2 4 l 2l l 8
l1 3 (Given) 1 2 2

l2 4 50 If the length of a closed organ pipe is 1.5 m and
velocity of sound is 330 m/s, then the frequency
45 An air column in a pipe, which is closed at one
for the second note is
end, will be in resonance with a vibrating body of
(a) 220 Hz (b) 165 Hz
frequency 166 Hz, if the length of the air column is
(c) 110 Hz (d) 55 Hz
SRI MARUTHI P.U COLLEGE, HOSKOTE I PUC PART-2 CET/NEET STUDY
MATERIAL
Waves/Sound, Oscillations/SHM, Mechanical Properties of Matter/Elasticity, Fluid Mechanics (Fluid
Statics, Fluid Dynamics, Viscosity, Surface Tension),
Thermal Properties Matter/Thermometry (Thermal Expansion, Calorimetry, Transmission of Heat),
Thermo Dynamics, Kinetic Theory of Gases (KTG)
overtone of a closed pipe of length l. The value of
3v 3 330 lc is (m)
165
(b) For closed pipe second note = 4l 4 1.5 (a) 1.5 (b) 0.75
Hz . (c) 2 (d) 1
51 A pipe 30 cm long is open at both ends. Which (b)First tone of open pipe = first overtone of
harmonic mode of the pipe is resonantly excited v 3v 3 2 0.5
lc 0.75m
by a 1.1 kHz source ? (Take speed of sound in air = 2l0 4lc 4
330 ms1) closed pipe
(a) First (b) Second 56 In a resonance tube the first resonance with a
tuning fork occurs at 16 cm and second at 49 cm.
(c) Third (d) Fourth
If the velocity of sound is 330 m/s, the frequency
(a) Fundamental frequency of open pipe
of tuning fork is
v 330 (a) 500 (b) 300
n1 550
2l 2 0.3 Hz (c) 330 (d) 165
2 n1 1100Hz 1.1 kHz v 3v
First harmonic = .= l1 l2
52 Two closed organ pipes, when sounded (a)For closed pipe 4n ; 4n v 2n(l2 l1)
simultaneously gave 4 beats per sec. If longer pipe v 330
has a length of 1m. Then length of shorter pipe n 500Hz
2(l2 l1) 2 (0.49 0.16)
will be, (v = 300 m/s)
(a) 185.5 cm (b) 94.9 cm 57 If in an experiment for determination of velocity
(c) 90 cm (d) 80 cm of sound by resonance tube method using a tuning
v v fork of 512 Hz, first resonance was observed at
n1 n2
4l1 4l2 30.7 cm and second was obtained at 63.2 cm,
(b) For first pipe and for second pipe
then maximum possible error in velocity of sound
n2 n1 4
So, number of beats = is (consider actual speed of sound in air is 332
v 1 1 1 1 m/s)
4 16 300
4 l2 l1 cm l2 1 l2 94.9 (a) 204 cm/sec (b) 110 cm/sec
53 A closed organ pipe and an open organ pipe are (c) 58 cm/sec (d) 80 cm/sec
tuned to the same fundamental frequency. What is
the ratio of lengths
(a) 1 : 2 (b) 2 : 1
(c) 2 : 3 (d) 4 : 3 2(l2 l1) v 2n(l2 l1)
(d) Using

l l 2 512(63.2 30.7) 33280cm/ s


(a)Let 1 and 2 be the lengths of closed and open
v 332m / s 33200cm/ s
pipes respectively. (Neglecting end correction) Actual speed of sound 0
1 Hence error 33280 33200 80cm/ s
l1 1 4l1 l2 2 2 2l2
4 and 2
58 An organ pipe open at one end is vibrating in
v v v v l 1 first overtone and is in resonance with another
1
n n 1 2 4l1 2l2 l2 2 pipe open at both ends and vibrating in third
Given 1 2 so
54 An open pipe is suddenly closed at one end harmonic. The ratio of length of two pipes is
with the result that the frequency of third (a) 1 : 2 (b) 4 : 1
harmonic of the closed pipe is found to be higher (c) 8 : 3 (d) 3:8
by 100 Hz, then the fundamental frequency of
open pipe is: (a)In first overtone of organ pipe open at one end,
(a) 480 Hz (b) 300 Hz 3v
nc
4lc
(c) 240 Hz (d) 200 Hz end, .....(i)
v Third harmonic or second overtone of organ pipe

2l 3v
(d)Fundamental frequency of open organ pipe n0
2l0
3v open at both end,
.....(ii)
Frequency of third harmonic of closed pipe 4l
3v 3v0 l 1
3v v 3v 2v v v c .
100 100 200Hz. nc no 4l c 2l 0 lo 2
4l 2l 4l 4l 4l 2l given
55 Fundamental frequency of an open pipe of 59A source of sound of frequency 450 cycles/sec is
length 0.5 m is equal to the frequency of the first moving towards a stationary observer with 34
SRI MARUTHI P.U COLLEGE, HOSKOTE I PUC PART-2 CET/NEET STUDY
MATERIAL
Waves/Sound, Oscillations/SHM, Mechanical Properties of Matter/Elasticity, Fluid Mechanics (Fluid
Statics, Fluid Dynamics, Viscosity, Surface Tension),
Thermal Properties Matter/Thermometry (Thermal Expansion, Calorimetry, Transmission of Heat),
Thermo Dynamics, Kinetic Theory of Gases (KTG)
m/sec speed. If the speed of sound is 340 m/sec, (c) 1 : 1 (d) 9 : 10
then the apparent frequency will be (a)When source is approaching the observer, the
(a) 410 cycles/sec (b) 500 cycles/sec frequency heard
(c) 550 cycles/sec (d) 450 cycles/sec v
n
340
na 1000 1063Hz
v vS 340 20
v
450
340
n' n 500
v vO 340 34 cycles/ sec When source is receding, the frequency
(b)
v 340
60 The wavelength is 120 cm when the source is nr n 1000 944
v vS 340 20
stationary. If the source is moving with relative heard =
velocity of 60 m/sec towards the observer, then na : nr 9 : 8
the wavelength of the sound wave reaching to the na v vS 340 20 9
.
observer will be (velocity of sound = 330 m/s) nr v vS 340 20 8
Short tricks :
(a) 98 cm (b) 140 cm
64 A source and listener are both moving towards
(c) 120 cm (d) 144 cm each other with speed v/10, where v is the speed
v v vS of sound. If the frequency of the note emitted by
n n

v vs v the source
v is f, the frequency heard by the listener
(a)
would be nearly
330 60 (a) 1.11 f (b) 1.22 f
120 98cm.
330 vS (c) f
vO (d) 1.27 f

61 An observer moves towards a stationary source (b)When source and listener both are moving
of sound of frequency n. The apparent frequency towards each other then, the frequency heard
heard by him is 2n. If the velocity of sound in air is
332 m/sec, then the velocity of the observer is

(a) 166 m/sec (b) 664 m/sec


(c) 332 m/sec (d) 1328 m/sec

v vO v v0 v v0 v vO
n' n 2n n 2 n n v v / 10
v v v n f 1.22 f.
(c) v vS v v / 10

vO v 332m/sec 65 A table is revolving on its axis at 5 revolutions


per second. A sound source of frequency 1000 Hz
62 Two passenger trains moving with a speed of
O is fixed on the table at 70 cm from the axis. The
108 km/hour cross each other. One of them blows
vO minimum frequency heard by a listener standing
a whistle whose frequency is 750 Hz. If sound
at a distance from the table will be (speed of
speed is 330 m/s, then passengers sitting in the
sound = 352 m/s
other train, after trains cross each other will hear
(a) 1000 Hz (b) 1066 Hz
sound whose frequency will be
(c) 941 Hz (d) 352 Hz
(a)900 Hz (b)625 Hz
v
(c)750 Hz (d)800 Hz Q11-DE (c)For source vS = r 0.70 2 5 22 m/sec
Minimum frequency is heard when the source is
AnsvS(b) receding the
S v
nmin n
v vS
man. It is given by
352
1000 941
5 = 352 22 Hz
v vO 330 180
n' n 750 18 625 66 A source of sound S of frequency 500 Hz
5
v vS 330 108 situated between a stationary observer O and a
18 Hz wall W, moves towards the wall with a speed of 2
63 A source of sound is moving with constant m/s. If the velocity of sound is 332 m/s, then the
velocity of 20 m/s emitting a note of frequency number of beats per second heard by the observer
1000 Hz. The ratio of frequencies observed by a is (approximately)
stationary observer while the source is (a) 8 (b) 6
approaching him and after it crosses him will be
(c) 4 (d) 2
(a) 9 : 8 (b) 8 : 9
SRI MARUTHI P.U COLLEGE, HOSKOTE I PUC PART-2 CET/NEET STUDY
MATERIAL
Waves/Sound, Oscillations/SHM, Mechanical Properties of Matter/Elasticity, Fluid Mechanics (Fluid
Statics, Fluid Dynamics, Viscosity, Surface Tension),
Thermal Properties Matter/Thermometry (Thermal Expansion, Calorimetry, Transmission of Heat),
Thermo Dynamics, Kinetic Theory of Gases (KTG)
(b)For direct sound source is moving away from v vO 330 33
the observes so frequency heard in this case n' n 100 90
(a) v 330 Hz
W
Direct sound
69 Two sirens situated one kilometer apart are
O S vS vS producing sound of frequency 330 Hz. An observer
starts moving from one siren to the other with a
Source of sound Image of source speed of 2 m/s. If the speed of sound be 330 m/s,
Echo sound what will be the beat frequency heard by the
vO
observer
Stationary siren 1
v (a) 8 (b) 4
500
332 332
n1 n 500 Hz
v vs 332 2 334 (c) 6 (d) 1
The other sound is echo, reaching the observer (b) Observer is moving away from siren 1 and
from the wall and towards the siren 2.
can be regarded as coming from the image of
source formed by
reflection at the wall. This image is approaching
the observer in the
direction of sound.
Hence for reflected sound, frequency heard by the
observer is

v
500
332 332
n2 n 500 Hz Hearing frequency of sound emitted by siren 1
v vS 332 2 330

1 1 v v0 330 2
n2 n1 500 332 6. n1 n 330 328Hz
330 334 v 330
Beats frequency
67 A motor car blowing a horn of frequency Hearing frequency of sound emitted by siren 2
124vib/sec moves with a velocity 72 km/hr v vO 330 2
n2 n 330 332Hz
towards a tall wall. The frequency of the reflected v 330
sound heard by the driver will be (velocity of
sound in air is 330 m/s) n n 332 328 4.
Hence, beat frequency 2 1

(a) 109 vib/sec (b) 132 vib/sec 70 A siren placed at a railway platform is emitting
(c) 140 vib/sec (d) 248 vib/sec sound of frequency 5 kHz. A passenger sitting in a
(c)Similar to previous question moving train A records a frequency of 5.5 kHz
while the train approaches the siren. During his
return journey in a different train B he records a
frequency of 6.0 kHz while approaching the same
vO vS = vO
siren. The ratio of the velocity of train B to that of
train A is
Echo sound
(a) 242/252 (b) 2
v v (c) 5/6 (d) 11/6
The frequency of reflected sound heard by (b)In both the cases observer is moving towards,
the driver
v v0
n n
v (vO ) v vO v
n n n
v v
the source. Hence by using
v vS S

330 (72 5 / 18)


124
330 (72 5 / 18) = 140 vibration/sec. vA vB
68 An observer is moving away from source of Train A Train B

sound of frequency 100 Hz. His speed is 33 m/s. If


speed of sound is 330 m/s, then the observed
frequency is When passenger is sitting in train A, then
(a) 90 Hz (b) 100 Hz v vA
5.5 5
(c) 91 Hz (d) 110 Hz v
(i)
SRI MARUTHI P.U COLLEGE, HOSKOTE I PUC PART-2 CET/NEET STUDY
MATERIAL
Waves/Sound, Oscillations/SHM, Mechanical Properties of Matter/Elasticity, Fluid Mechanics (Fluid
Statics, Fluid Dynamics, Viscosity, Surface Tension),
Thermal Properties Matter/Thermometry (Thermal Expansion, Calorimetry, Transmission of Heat),
Thermo Dynamics, Kinetic Theory of Gases (KTG)
when passenger is sitting in train B, then
v vB Hence number of beats heard per second
6 5
v (ii)
v v

vB v v n v v n
2 = S S
vA
On solving equation (i) and (ii) we get 2nvvS 2 256 330 5
71 Two trains are moving towards each other at
2
vS2 335 325
= v = 7.8 Hz
speeds of 20 m/s and 15 m/s relative to the
74 The apparent frequency of a note, when a
ground. The first train sounds a whistle of
listener moves towards a stationary source, with
frequency 600 Hz. the frequency of the whistle
velocity of 40 m/s is 200 Hz. When he moves away
heard by a passenger in the second train before from the same source with the same speed, the
the train meets is (the speed of sound in air is 340 apparent frequency of the same note is 160 Hz.
m/s) The velocity of sound in air is (in m/s)
(a) 600 Hz (b) 585 Hz (a) 360 (b) 330
(c) 645 Hz (d) 666 Hz (c) 320 (d) 340
(d) The frequency of whistle heard by passenger (a)When a listener moves towards a stationary
in the train B, is source apparent frequency
v
v vO
n
v n 200
..(i)
vS vO
A B When listener moves away from the same source
(v vO )
n n 160
v v0 v ..(ii)
340 15
n n 600 666Hz
From (i) and (ii)
v vs 340 20
v vO 200 v vO 5
72 When an engine passes near to a stationary
v vO 160 v vO 4
observer then its apparent frequencies occurs in v 360m/ sec
the ratio 5/3. If the velocity of engine is 75 A spherical source of power 4 W and frequency
800 Hz is emitting sound waves. The intensity of
(a) 540 m/s (b) 270 m/s
waves at a distance 200 m is
(c) 85 m/s (d) 52.5 m/s
8 106 W / m2 2 104 W / m2
(c)When engine approaches towards observer (a) (b)
v 1 104 W / m2 4 W / m2
n' n (c) (d)

v vS Power 4
7.9 10 6
v (a)Intensity = Area 4 (200
)2
W / m2
n n

when engine going away from observer v vS 76 If the amplitude of sound is doubled and the
n' v vS 5 340 vS frequency reduced to one-fourth, the intensity of
vS 85 sound at the same point will be
n'' v vS 3 340 vS m/ s .
(a) Increased by a factor of 2
73 A source of sound of frequency 256 Hz is (b) Decreased by a factor of 2
moving rapidly towards a wall with a velocity of (c) Decreased by a factor of 4
5m/s. The speed of sound is 330 m/s. If the
(d) Unchanged
observer is between the wall and the source, then
2 2
beats per second heard will be I1 a1 n
1
n
(c) I 2 a n v I a n 2 2 2
(a) 7.8 Hz (b) 7.7 Hz 2 2 2 2 2 I a
(c) 3.9 Hz (d) Zero 2 2
1 1 I1
(a)The observer will hear two sound, one directly I2
2 1/ 4 4
from source and other from reflected image of
77 Intensity level of a sound of intensity I is 30 dB.
sound
I
Direct sound I0 I0
The ratio is (Where is the threshold of
vS vS hearing)
(a) 3000 (b) 1000
Source of sound Image of source (c) 300 (d) 30
Echo sound
SRI MARUTHI P.U COLLEGE, HOSKOTE I PUC PART-2 CET/NEET STUDY
MATERIAL
Waves/Sound, Oscillations/SHM, Mechanical Properties of Matter/Elasticity, Fluid Mechanics (Fluid
Statics, Fluid Dynamics, Viscosity, Surface Tension),
Thermal Properties Matter/Thermometry (Thermal Expansion, Calorimetry, Transmission of Heat),
Thermo Dynamics, Kinetic Theory of Gases (KTG)
I I 2.OSCILLATIONS/SHM
L 10log10 30 103

I0 I0 82 A particle is oscillating according to the
78 If T is the reverberation time of an auditorium equation X 7 cos0.5t , where t is in second. The
of volume V then point moves from the position of equilibrium to
1
maximum displacement in time
1
T T (a) 4.0 sec (b) 2.0 sec
(a) V (b) V2
(c) 1.0 sec (d) 0.5 sec
(d) T V
2
(c) T V 2
0.5
T
kV (c)From given equation T T 4 sec
d)Reverberation time S T V. Time taken from mean position to the
79 The power of a sound from the speaker of a
1
T 1
radio is 20 mW. By turning the knob of the volume maximum displacement 4 sec.
control, the power of the sound is increased to 400 20
83 A 1.00 10 kg particle is vibrating with simple
mW. The power increase in decibels as compared
5
to the original power is harmonic motion with a period of 1.00 10 sec and a
(a) 13 dB (b) 10 dB 1.00 103m/s
maximum speed of . The maximum
(c) 20 dB (d) 800 dB displacement of the particle is
(a) 1.59 mm (b) 1.00 m
(a) P I
(c) 10 m (d) None of these
I1 I
L1 10log10
L2 10log10 2 2 v T
I0 I0 vmax a a a max
and (a) T 2
I
L 2 L1 10log10 2 1.00 103 (1 105)
I1 a 1.59
So 2 mm
P2 400 84 Two simple harmonic motions are represented
10log10
10log10
P1 20 10log10 20
= = = y1 0.1 sin 100 t
3 y 0.1 cos t.
= 10log(2 10) = 10(0.301 1) 13dB by the equations and 2
The phase difference of
80 The intensity level due to two waves of the
same frequency in a given medium are 1 bel and 5
bel. Then the ratio of amplitudes is the velocity of particle 1 with respect to the
velocity of particle 2 is
(a) 1 : 4 (b) 1 : 2
4
(c) 1 : 10 (d) 1 : 102
(a) 3 (b) 6
I
L log10
I0
(d)By using
(c) 6 (d) 3
I2 I
L2 L1 log10 log10 1 dy1
I0 I0 v1 0.1 100 cos 100t
dt 3
I2 I2 I2 (c)
5 1 log10 4 log10 104
I1 I1 I1 dy2
v2 0.1 sint 0.1 cos t
dt 2
a22 a2 102 a1 1
2
104 2 Phase difference of velocity of first particle with
a a 1 a 10
1 1 2 respect to the velocity of 2nd particle
81 A point source emits sound equally in all at t = 0 is
directions in a non-absorbing medium. Two points
1 2 .
P and Q are at distances of 2m and 3m respectively 3 2 6
from the source. The ratio of the intensities of the 85 A simple pendulum performs simple harmonic
motion about X = 0 with an amplitude A and time
waves at P and Q is A
X
(a) 9 : 4 (b) 2 : 3 period T. The speed of the pendulum at 2 will
(c) 3 : 2 (d) 4 : 9 be
2
1 I1 d2 3
2
9 A 3 A

2
(Distance) I 2 d1 2 4 (a) T (b) T
(a) Intensity .
SRI MARUTHI P.U COLLEGE, HOSKOTE I PUC PART-2 CET/NEET STUDY
MATERIAL
Waves/Sound, Oscillations/SHM, Mechanical Properties of Matter/Elasticity, Fluid Mechanics (Fluid
Statics, Fluid Dynamics, Viscosity, Surface Tension),
Thermal Properties Matter/Thermometry (Thermal Expansion, Calorimetry, Transmission of Heat),
Thermo Dynamics, Kinetic Theory of Gases (KTG)
A 3 3 2 A 90 A particle is performing simple harmonic
motion with amplitude A and angular velocity .
(c) 2T (d) T
The ratio of maximum velocity to maximum
(a)Velocity of a particle executing S.H.M. is given
acceleration is
by
(a) (b) 1/
2 A 2 2 3A 2 A 3
v a x 2 2
A 2
(c) 2 (d) A
T 4 T 4 T .
vmax a 1
86 A body is executing simple harmonic motion
Amax a 2
(b)
with an angular frequency 2rad/ s . The velocity of 91 The displacement equation of a particle is
the body at 20 mm displacement, when the x 3 sin2t 4 cos2t.
The amplitude and maximum
amplitude of motion is 60 mm, is velocity will be respectively
(a) 40 mm /s (b) 60mm/ s (a) 5, 10 (b) 3, 2
(c) 4, 2 (d) 3, 4
(c) 113mm/ s (d) 120mm/ s
(a) x 3 sin2t 4 cos2t. From given equation
v (a y ) 2 60 20 113mm/ s
2 2 2 2

(c) .
a1 3, a2 4, 2
87 A body of mass 5 gm is executing S.H.M. about and
a point with amplitude 10 cm. Its maximum a a12 a22 32 42 5 vmax a 5 2 10

velocity is 100 cm/sec. Its velocity will be 50
cm/sec at a distance 92 The amplitude of a particle executing S.H.M.
with frequency of 60 Hz is 0.01 m. The maximum
(a) 5 (b) 5 2 value of the acceleration of the particle is
144 2m /sec2 144m /sec2
(c) 5 3 (d) 10 2 (a) (b)
144
vmax 100cm/ sec m /sec2
(c)It is given , a = 10 cm. 2 288 2m /sec2
(c) (d)
100 2 2 2
vmax a 10rad/ sec (a)Maximum acceleration a a 4 n
10
0.01 4 ( )2 (60)2 144 2m/ sec
2 2 2 2
v a y 50 10 (10) y
Hence 93 A small body of mass 0.10 kg is executing
S.H.M. of amplitude 1.0 m and period 0.20 sec.
y 5 3 cm
The maximum force acting on it is
88 The maximum velocity and the maximum (a) 98.596 N (b)
acceleration of a body moving in a simple 985.96 N
2m/s 4 m/s2 . (c) 100.2 N (d) 76.23 N
harmonic oscillator are and Then
angular velocity will be (a)Maximum acceleration
(a) 3 rad/sec (b) 0.5 rad/sec a 4 2 1 4 (3.14)2
a 2
Amax T2 0.2 0.2
(c) 1 rad/sec (d) 2 rad/sec
A 4 0.1 4 (3.14)2
max 2rad/ sec Fmax m Amax 98.596 N
v a A a 2 vmax 2 0.2 0.2
(d) max and max
94 A body executing simple harmonic motion has
89 A particle executing simple harmonic motion
has an amplitude of 6 cm. Its acceleration at a 24metres/sec2
a maximum acceleration equal to and
2
8 cm/s 16 metres/sec
distance of 2 cm from the mean position is . maximum velocity equal to . The
The maximum speed of the particle is amplitude of the simple harmonic motion is
(a) 8 cm/s (b) 12 cm/s 32 3
metres metres
(c) 16 cm/s (d) 24 cm/s (a) 3 (b) 32

8 1024 64
2 A/y 2rad/ sec metres metres
(b) A y 2 (c) 9 (d) 9

vmax a 6 2 12cm/ sec (a) Maximum velocity a = 16


Now
2
Maximum acceleration a 24
SRI MARUTHI P.U COLLEGE, HOSKOTE I PUC PART-2 CET/NEET STUDY
MATERIAL
Waves/Sound, Oscillations/SHM, Mechanical Properties of Matter/Elasticity, Fluid Mechanics (Fluid
Statics, Fluid Dynamics, Viscosity, Surface Tension),
Thermal Properties Matter/Thermometry (Thermal Expansion, Calorimetry, Transmission of Heat),
Thermo Dynamics, Kinetic Theory of Gases (KTG)
(a )2 16 16 32 2 2
a 2
m (c) Velocity v A x and acceleration
a 24 3
2
95A particle of mass 10 grams is executing simple x
harmonic motion with an amplitude of 0.5 m and 2 2 2 2 2 2
Now given, x A x .1 2 1
periodic time of ( / 5) seconds. The maximum 2 2
value of the force acting on the particle is T

3 3
(a) 25 N (b) 5 N
99 A particle is vibrating in a simple harmonic
(c) 2.5 N (d) 0.5 N
motion with an amplitude of 4 cm. At what
4 2 displacement from the equilibrium position, is its
m(a 2 ) ma
T 2 energy half potential and half kinetic
(d)Maximum force
4 2 (a) 1 cm (b) 2 cm
0.5 2 0.01 0.5N
/ 25
(c) 3 cm (d) 2 2 cm
96 A body is vibrating in simple harmonic motion
(d)Let x be the point where K.E. = P.E.
with an amplitude of 0.06 m and frequency of 15
Hz. The velocity and acceleration of body is 1 1
m 2(a2 x2) m 2 x2
Hence 2 2
5.65m/s 5.32 102m/s2
(a) and a 4
x 2 2 cm
2 2
2x a 2 2
6.82m/s 7.62 102m/s2 100 The kinetic energy and potential energy of a
(b) and
particle executing simple harmonic motion will be
8.91m/s 8.21 102m/s2 equal, when displacement (amplitude = a) is
(c) and
9.82m/s 9.03 102m/s2 a
(d) and
(a) 2 (b) a 2
v a a 2 n
(a)Velocity a a 2
0.06 2 15 5.65 m/ s (c) 2 (d) 3
2 2 2 2 2 (c)Suppose at displacement y from mean position
Acceleration A a 4 n a 5.32 10 m/ s potential energy = kinetic energy
97 What is the maximum acceleration of the 1 1
m(a2 y2) 2 m 2y2
t 2 2
y 2sin
2 where 2 is in
particle doing the SHM a
y
cm 2 2
a 2y 2
2
101 The potential energy of a particle executing
cm/ s2 cm/ s2
(a) 2 (b) 2 S.H.M. is 2.5 J, when its displacement is half of
amplitude. The total energy of the particle be
cm/ s2 cm/ s2
4 4 (a) 18 J (b) 10 J
(c) (d)
(c) 12 J (d) 2.5 J
(b)Comparing given equation with
standard equation, 1
m 2y2
Potential
energy(U) 2 y2
2
Total energy
(E) 1
y a sin(t ), we get, a 2cm, 2 m 2a2 a
(b) 2
2
2 2
2 2 cm/ s2 a

Amax A 2 2 2.5 2
. 2
98 A particle executes linear simple harmonic So E a E 10J
motion with an amplitude of 2 cm. When the 102 The angular velocity and the amplitude of a
particle is at 1 cm from the mean position the
magnitude of its velocity is equal to that of its simple pendulum is and a respectively. At a
acceleration. Then its time period in seconds is displacement X from the mean position if its
1 kinetic energy is T and potential energy is V, then
the ratio of T to V is
(a) 2 3 (b) 2 3
2 2 2 2 2 2 2 2
2 3 (a) X /(a X ) (b) X /(a X )
(c) 3 (d) 2
SRI MARUTHI P.U COLLEGE, HOSKOTE I PUC PART-2 CET/NEET STUDY
MATERIAL
Waves/Sound, Oscillations/SHM, Mechanical Properties of Matter/Elasticity, Fluid Mechanics (Fluid
Statics, Fluid Dynamics, Viscosity, Surface Tension),
Thermal Properties Matter/Thermometry (Thermal Expansion, Calorimetry, Transmission of Heat),
Thermo Dynamics, Kinetic Theory of Gases (KTG)
2 2 2 2 2 2 2 2 1 1
(c) (a X ) / X (d) (a X ) / X K max m 2a2 1 (100)2 (6 10 2)2 18 J
2 2
1 107 The total energy of a particle executing S.H.M.
T m 2(a2 x2)
d)Kinetic energy 2 is 80 J. What is the potential energy when the
1 particle is at a distance of 3/4 of amplitude from
V m 2x2 the mean position
and potential energy, 2
2 2
T a x
(a) 60 J (b) 10 J
V x2
103 When the potential energy of a particle (c) 40 J (d) 45 J
executing simple harmonic motion is one-fourth of 1
m 2y2 3
2
U 2 y2
its maximum value during the oscillation, the 2 a
E 1 m 2a2 a U 4 9
displacement of the particle from the equilibrium
(d) 2 80 a2 16 U 45J
position in terms of its amplitude a is
108 A particle starts simple harmonic motion from
(a) a / 4 (b) a / 3 the mean position. Its amplitude is a and total
energy E. At one instant its kinetic energy is 3E / 4.
(c) a / 2 (d) 2a / 3
Its displacement at that instant i
1
m 2y2
U
2 (a) a / 2 (b) a / 2
Umax 1 1 y2 a
m 2a2 2 y a
(c) 2 4 a 2
3/ 2
104 A particle of mass 10 gm is describing S.H.M. (c) (d) a / 3
along a straight line with period of 2 sec and 1
m 2(a2 y2 )
K 2 a2 y2 y2
amplitude of 10 cm. Its kinetic energy when it is at 1 2
E 1 a 2
a
5 cm from its equilibrium position is m 2a2
(b) 2
2 2
(a) 37.5 ergs (b) 3.75 ergs 3E
2 2
4 1 y y 1 3 1 a
2 2 y
(c) 375 ergs (d) 0.375 ergs
2 2
So, E a a 4 4 2.

1 109 The amplitude of a particle executing SHM is


K m 2(a2 y2) made three-fourth keeping its time period
(c)Kinetic energy 2
constant. Its total energy will be
2
1 2 2 2
10 [10 5 ] E 3
2 2 375 2ergs E
(a) 2 (b) 4
105 When the displacement is half the amplitude,
9
the ratio of potential energy to the total energy is E
(c) 16 (d) None of these
1 1
2
3
(a) 2 (b) 4 a
1
E a 2

E 4 3
E m 2a2 2 a a
1 2 a2 4
(c) E a E
(c) 1 (d) 8
9
E E
2
16
1 a
m 2y2
U 2 y2 2 1 110 The equation of motion of a particle is
2
E 1 m 2a2 a a 4
2 d2y
(b) Ky 0
dt2 , where K is positive constant. The time
1kg
106 A body of mass is executing simple period of the motion is given by
harmonic motion. Its displacement at t y(cm) 2

seconds is given by
y 6 sin(100t /4)
. Its maximum (a) K (b) 2K
kinetic energy is 2
(a) 6 J (b) 18 J (c) K (d) 2 K
(c) 24 J (d) 36 J
(c)On comparing with standard equation
a 6cm, 100rad/ sec 2 2 2
(b) So d y 2 K K T
2y 0
dt2 T K .
we get
SRI MARUTHI P.U COLLEGE, HOSKOTE I PUC PART-2 CET/NEET STUDY
MATERIAL
Waves/Sound, Oscillations/SHM, Mechanical Properties of Matter/Elasticity, Fluid Mechanics (Fluid
Statics, Fluid Dynamics, Viscosity, Surface Tension),
Thermal Properties Matter/Thermometry (Thermal Expansion, Calorimetry, Transmission of Heat),
Thermo Dynamics, Kinetic Theory of Gases (KTG)
111 The maximum speed of a particle executing y 5 sin( t 4 )
(d) , comparing it with standard
S.H.M. is 1m/ s and its maximum acceleration is equation
1.57m / sec2 . The time period of the particle will be 2 t
y a sin( t ) a sin
T
1
sec 2 t
(a) 57
1. (b) 1.57 sec a 5m t
and T T = 2 sec.
(c) 2 sec (d) 4 sec
116 The mass and diameter of a planet are twice
(d)Given max velocity a 1 and maximum those of earth. The period of oscillation of
2 pendulum on this planet will be (If it is a second's
acceleration a 1.57 pendulum on earth)
2a 2 1
1.57 1.57 1.57 T 4
a T 2 sec
(a) (b) 2 2 sec
112 The acceleration of a particle performing
1
12cm/sec2 (c) 2 sec (d) 2 sec
S.H.M. is at a distance of 3 cm from the
mean position. Its time period is GM
g
(a) 0.5 sec (b) 1.0 sec (b)As we know R2
2
(c) 2.0 sec (d) 3.14 sec gearth M e R g 2
2 e
g
planet M p Re gp 1

1 Te gp 2 1
T
g Tp ge Tp 2
Also
Displaceme
nt 3
T 2 2 3.14 sec
(d) Accelerati
on 12 Tp 2 2 sec.
113 If a simple harmonic oscillator has got a 117 A simple pendulum is set up in a trolley which
displacement of 0.02 m and acceleration equal to moves to the right with an acceleration a on a
2.0ms2 at any time, the angular frequency of the horizontal plane. Then the thread of the pendulum
oscillator is equal
in the mean position makes an angle with the
10rads1 0.1rads1
(a) (b) vertical
1
100rads 1rads1 a
(c) (d) tan1
(a) g in the forward direction
Accelerati
on 2.0
1
(a) Displaceme
nt 0.02 10rads tan1
a
(b) g in the backward direction
114A particle in SHM is described by the
displacement equation x(t) A cos(t ). If the initial tan1
g
(t = 0) position of the particle is 1 cm and its initial (c) a in the backward direction

velocity is cm/s, what is its amplitude? The g


tan1
1
(d) a in the forward direction
angular frequency of the particle is s

(a) 1 cm (b) 2 cm
(c) 2 cm (d) 2.5 cm
v cm/ sec, x 1cm 1
(b)Given, and s
2 2 2
using v a x a 1

1 a2 1 a 2 cm.
115 A simple harmonic wave having an amplitude
a and time period T is represented by the equation
y 5 sin (t 4)m. Then the value of amplitude (a) in
(m) and time period (T) in second are
a 10, T 2 a 5, T 1
(a) (b)
a 5, T 2
(c) a 10, T 1 (d)
a SRI MARUTHI P.U COLLEGE, HOSKOTE I PUC PART-2 CET/NEET STUDY
MATERIAL
Waves/Sound, Oscillations/SHM, Mechanical Properties of Matter/Elasticity, Fluid Mechanics (Fluid
Statics, Fluid Dynamics, Viscosity, Surface Tension),
Thermal Properties Matter/Thermometry (Thermal Expansion, Calorimetry, Transmission of Heat),
Thermo
mg ma Dynamics, Kinetic Theory of Gases (KTG)

(b)In accelerated frame of reference, a fictitious 120The length of the second pendulum on the
force (pseudo force) ma acts on the bob of surface of earth is 1 m. The length of seconds
pendulum as shown in figure.
pendulum on the surface of moon, where g is
ma a a 1/6th value of g on the surface of earth, is
tan tan1
Hence, mg g in the backward g
(a) 1 / 6 m (b) 6 m
direction.
(c) 1 / 36 m (d) 36 m
118A man measures the period of a simple
pendulum inside a stationary lift and finds it to be
T sec. If the lift accelerates upwards with an l l
T 2
g g
(a) = constant
acceleration g / 4 , then the period of the pendulum
lm 1 g 1
will be lm m
l g; 1 6 g 6
(a) T
121 If the length of second's pendulum is
T Pendulum decreased by 2%, how many seconds it will lose
(b) 4
per day
Lift
2T
(a) 3927 sec (b) 3727 sec
(c) 5
(c) 3427 sec (d) 864 sec
(d) 2T 5 T 1 l 0.02
0.01
T 0.01T
(d) T l T 2 l 2
l Loss of time per day 0.01 24 60 60 864sec
T 2
g 122 The length of a simple pendulum is increased
(c)In stationary lift
by 1%. Its time period will
l
T 2 (a) Increase by 1% (b) Increase by 0.5%
(g a)
a In upward moving lift
l (c) Decrease by 0.5% (d)Increase by 2%
a
( Acceleration of lift)
T 1 l 1
T g g 4 1% 0.5%
T

g a

g

5 2T (b) T l T 2 l 2
g T
4 5 123 The bob of a pendulum of length l is pulled
119 A simple pendulum is suspended from the roof aside from its equilibrium position through an
of a trolley which moves in a horizontal direction
angle and then released. The bob will then pass
g
with an acceleration a, then the time period is
a h=l (1 cos ) through its equilibrium position with a speed v,
l
T 2 where v equals
g
, where g is equal to
g
given by
l 2gl(1 sin ) 2gl(1 cos )
(a) g (b) g a (a) (b)

g2 ha2 2gl(1 cos ) 2gl(1 sin )


(c) g a (d) (c) (d)
(c)If suppose bob rises up to a height h as shown
then after releasing potential energy at extreme
2 2
(d) g g a position becomes kinetic energy of mean position
1 2
mgh mvmax
2 vmax 2gh
lh
cos
Also, from figure l

h l(1 cos )
SRI MARUTHI P.U COLLEGE, HOSKOTE I PUC PART-2 CET/NEET STUDY
MATERIAL
Waves/Sound, Oscillations/SHM, Mechanical Properties of Matter/Elasticity, Fluid Mechanics (Fluid
Statics, Fluid Dynamics, Viscosity, Surface Tension),
Thermal Properties Matter/Thermometry (Thermal Expansion, Calorimetry, Transmission of Heat),
Thermo Dynamics, Kinetic Theory of Gases (KTG)
vmax 2gl(1 cos )
127 The ratio of frequencies of two pendulums are
So, 2 : 3, then their length are in ratio

124 A pendulum bob has a speed of 3 m/s at its


lowest position. The pendulum is 0.5 m long. The (a) 2 / 3 (b) 3 / 2
speed of the bob, when the length makes an angle (c) 4 / 9 (d) 9 / 4
2
of 60 to the vertical, will be (If g 10m/ s )
o
1 n1 l l1 n22 32 9
n 2 2 2
n2 l1
1
m/ s (d)Frequency l l2 n1 2 4
(a) 3m/ s (b) 3 128 Two pendulums begin to swing simultaneously.
1 If the ratio of the frequency of oscillations of the
m/ s
(c) 2 (d) 2m/ s two is 7 : 8, then the ratio of lengths of the two
(d)Let bob velocity be v at point B where it makes pendulums will be
an angle of 60o with the vertical, then using (a) 7 : 8 (b) 8 : 7
conservation of mechanical energy (c) 49 : 64 (d) 64 : 49
(d)Suppose at t 0 , pendulums begins to swing
l cos 60 l=0.5m
simultaneously.
Hence, they will again swing simultaneously
B
l (1-cos) n1T1 n2T2
if
2 2 A
n1 T2 l l n 8 v=3m/sec
64
2 P 1 2
n2 T1 l1 l2 n1 7 49
2m
KE A PE A KE B PE B 129A pendulum of length 2m lift at P. When it
reaches Q, it losses 10% of its total energy due to
1 1 Q
m 32 mv2 mgl(1 cos ) air resistance. The velocity at Q is
2 2

1 (a) 6 m/sec
9 v2 2 10 0.5 v 2 m/ s
2
(b) 1 m/sec
125 The time period of a simple pendulum is 2
(c) 2 m/sec
sec. If its length is increased 4 times, then its
period becomes (d) 8 m/sec
(a) 16 sec (b) 12 sec
(c) 8 sec (d) 4 sec (a)If v is velocity of pendulum at Q
and 10% energy is lost while moving from P to Q
T1 l 2 l
1 Hence, by applying conservation of between P and
T l T2 l2 T2 4l T2 4sec Q
(d)
126 A simple pendulum is executing simple 1 2
mv 0.9 (mgh) 2 v 6 m / sec
harmonic motion with a time period T. If the length 2 v 2 0.9 10 2
of the pendulum is increased by 21%, the 130 The period of a simple pendulum measured
percentage increase in the time period of the inside a stationary lift is found to be T. If the lift
pendulum of increased length is
starts accelerating upwards with acceleration of
(a) 10% (b) 21%
g / 3, then the time period of the pendulum is
(c) 30% (d) 50%
l1 100 l2 121 T T
(a) If initial length then
(a) 3 (b) 3
l T l
T 2 1 1
g T2 l2 3
By using T 3T
(c) 2 (d)
T1 100
T2 1.1T1 l
T2 121 T1 2
Hence, g
(c) For stationary lift
T2 T1 For ascending lift with acceleration a,
100 10%
T1
% increase = l
T2 2
g a
SRI MARUTHI P.U COLLEGE, HOSKOTE I PUC PART-2 CET/NEET STUDY
MATERIAL
Waves/Sound, Oscillations/SHM, Mechanical Properties of Matter/Elasticity, Fluid Mechanics (Fluid
Statics, Fluid Dynamics, Viscosity, Surface Tension),
Thermal Properties Matter/Thermometry (Thermal Expansion, Calorimetry, Transmission of Heat),
l
Thermo Dynamics, Kinetic Theory of Gases (KTG)

h=l (1 cos )

qE
+ ++ + + + + +

g L
g L 3L
T 2
g a gef 2 2
T1 T 3 4 3
T2 T
T2 g T2 g 3 2 2g / 3 2g
131 Length of a simple pendulum is l and its 133 A simple pendulum has time period T. Themgbob

maximum angular displacement is , then its is given negative charge and surface below it is
maximum K.E. is given positive charge. The new time period will be
(a)Less than T (b) Greater than T
(a) mglsin (b) mgl(1 sin )
(c)Equal to T (d) Infinite
(c) mgl(1 cos ) (d) mgl(1 cos ) (a)In this case time period of pendulum becomes
(d)Kinetic energy will be maximum at mean
l
position. T 2
qE
From law of conservation of energy maximum g
m
kinetic energy at mean position = Potential energy
at displaced position T T
K max mgh mgl(1 cos )

134 What is the velocity of the bob of a simple


pendulum at its mean position, if it is able to rise
to vertical height of 10cm (g = 9.8 m/s2)

132 The time period of a simple pendulum of (a) 2.2 m/s


length L as measured in an elevator descending (b) 1.8 m/s
g (c) 1.4 m/s A B

with acceleration 3 is (d) 0.6 m/s M

3L 3L
2 (c)According to the principle of conservation of energy,
g g
(a) (b) 1 2
mv mgh v 2gh 2 9.8 0.1 1.4 m/ s.
2 or
3L 2L
2 2 135 Two bodies M and N of equal masses are
(c) 2g (d) 3g
suspended from two separate massless springs of
(c)The effective acceleration in a lift descending force constants k1 and k2 respectively. If the two
g g 2g bodies oscillate vertically such that their maximum
gef g
with acceleration 3 is 3 3
SRI MARUTHI P.U COLLEGE, HOSKOTE I PUC PART-2 CET/NEET STUDY
MATERIAL
Waves/Sound, Oscillations/SHM, Mechanical Properties of Matter/Elasticity, Fluid Mechanics (Fluid
Statics, Fluid Dynamics, Viscosity, Surface Tension),
Thermal Properties Matter/Thermometry (Thermal Expansion, Calorimetry, Transmission of Heat),
Thermo Dynamics, Kinetic Theory of Gases (KTG)
velocities are equal, the ratio of the amplitude M figure. A mass m is suspended from them. The
to that of N is ratio of their frequencies of vertical oscillations will
be
k1 k1
k2 k2
(a) (b)

k2 k2
k1 (c) k1 B
k2 k1
(d)

k
a a
Maximum velocity m

A K1 K a1 K2
a1 a2 2
m a
m 2 K1
Given that (a) 2 : 1 (b) 1 : 1
136 A mass m is suspended by means of two (c) 1 : 2 (d) 4 : 1
coiled spring which have the same length in
m k
unstretched condition as in figure. Their force
n k n 2 1
constant are k1 and k2 respectively. When set into n
1 k S S s

2 n
m P kP np 2k 2
(c)
vertical vibrations, the period will be
S1 S2
139 In the figure, and are identical springs.

The oscillation frequency of the mass m is f . If one


B
A spring is removed, the frequency will become
m
m S1 S2 k
2 2 m 1
k1k2 k2
(a) (b)

m m
2
2

k1 k2 k1 k2
(c) (d)
(a) f (b) f 2
(d)Given spring system has parallel combination,
so (c) f 2 (d) f / 2
m
T 2 1 keq 1 2k
keq k1 k2 (k1 k2) f
and time period 2
m 2 m ..(i)
(d)For the given figure
137 In arrangement given in figure, if the block of If one spring is removed, then keq = k and
mass m is displaced, the
(A) B frequency is given by
A 1 k
f '
K m 2 m .(ii)
K1 K2

K f f
2 f
From equation (i) and (ii), f 2
140 A weightless spring which has a force
1 k1 k2 1 k1 k2 constant oscillates with frequency n when a mass
n n
2 m 2 m m is suspended from it. The spring is cut into two
m (a) (b)
equal halves and a mass 2m is suspended from it.
1 m The frequency of oscillation will now become
n
2 k k
1 2 K K
(c) (d) (a) n (b) 2n
1 m 1/ 2
n
2

k k
(B) (c) n / 2 (d) n(2)
1 2
(b)With respect to the block the springs are 1 k n k m k 2m
connected in parallel combination. n 1
(a) 2 m n m K m 2K n n
1 k1 k2 141 The vertical extension in a light spring by a
n
Combined stiffness k = k1+ k2 and 2 m
weight of 1 kg suspended
m from the wire is 9.8 cm.
138Two identical spring of constant K are The period of oscillation
connected in series and parallel as shown in
SRI MARUTHI P.U COLLEGE, HOSKOTE I PUC PART-2 CET/NEET STUDY
MATERIAL
Waves/Sound, Oscillations/SHM, Mechanical Properties of Matter/Elasticity, Fluid Mechanics (Fluid
Statics, Fluid Dynamics, Viscosity, Surface Tension),
Thermal Properties Matter/Thermometry (Thermal Expansion, Calorimetry, Transmission of Heat),
Thermo Dynamics, Kinetic Theory of Gases (KTG)
(a) 20 sec (b) 2 sec

(c) 2 / 10sec (d) 200 sec

m x m x
T 2 2
k g
(c) mg = kx k g
1 1
1: 2 : 2: 2 :
9.8 102 2 2 2
2 (a) (b)
9.8 10 sec
1 1
142 The length of a spring is l and its force : 2:1 2: :1
(c) 2 (d) 2
constant is k. When a weight W is suspended from
it, its length increases by x. If the spring is cut into 1 1 1 1 1
T T1 : T2 : T3 : : 1: 2 :
two equal parts and put in parallel and the same k k k/2 2k 2
(a)
weight W is suspended from them, then the
145 A mass m performs oscillations of period T
extension will be
when hanged by spring of force constant K. If
(a) 2x (b) x spring is cut in two parts and arranged in parallel
x x and same mass is oscillated by them, then the
(c) 2 (d) 4 new time period will beK
(d)Spring is cut into two equal halves so spring (a) 2T
constant of each part = 2k m (b) T m
K eq 2K 2K 4K
These parts are in parallel so T
Extension force (i.e. W) is same hence by using 2
(c)
x
x

F kx 4k x kx 4 . T

143 A mass m is vertically suspended from a (d) 2


spring of negligible mass; the system oscillates T2 k
1 k 1 T
with a frequency n. What will be the T 1 T2 1
k T1 k2 4k 2 2
(d)
frequency of the system if a mass 4 m is 146 What will be the force constant of the spring
suspended from the same spring system shown in the figure

K1 K1
n/ 4 K2
(a)KK (b) 4n
(a) 2
n/ 2 K1
(iii) (c)
m (d) 2n
1 1
1
K2

2K K2
n
1 k
n
1 n
1
m2 (b) 1
2 m m n m1
(c) 2
1 1

2K1 K 2

n

4m
n2
n (c) m
n2 m 2 1
2 1
144 Five identical springs are used in the following
K K
(ii) K
three configurations. The time periods of vertical (d) 1 1

oscillations in configurations (i), (ii) and (iii) are in


K
the ratio

(i)
m
m
k2
SRI MARUTHI P.U COLLEGE, HOSKOTE I PUC PART-2 CET/NEET STUDY
MATERIAL
Waves/Sound, Oscillations/SHM, Mechanical Properties of Matter/Elasticity, Fluid Mechanics (Fluid
Statics, Fluid Dynamics, Viscosity, Surface Tension),
Thermal Properties Matter/Thermometry (Thermal Expansion, Calorimetry, Transmission of Heat),
Thermo Dynamics, Kinetic Theory 2k1 of Gases (KTG)

k1

k1

2k1
k2

SRI MARUTHI P.U COLLEGE, HOSKOTE I PUC PART-2 CET/NEET STUDY


MATERIAL
Waves/Sound, Oscillations/SHM, Mechanical Properties ofK2Matter/Elasticity, Fluid Mechanics (Fluid
Statics, Fluid Dynamics, Viscosity, Surface Tension),
Thermal Properties Matter/Thermometry (Thermal Expansion,
K1 Calorimetry, Transmission of Heat),
Thermo Dynamics, Kinetic Theory of Gases (KTG)
(b) In series combination
1 K
1 1 1

kS 2k1 k2
(a)
2 m

1
1 (K1 K 2)m
1 m
1 2 K1K 2
kS (b)
2k
1 k2

K
2
(c) m

1 K1K 2
147 The effective spring constant of two spring 2 m(K1 K 2)
(d)
system as shown in
K2
figure will be
K1
1 keq 1 k1k2
n
2 m 2 (k1 k2)m
(d)
151 A mass m is suspended separately by two
different springs of spring constant K1 and K2 gives
t t
K1 K 2 K1K 2 / K1 K 2 the time-period 1 and 2 respectively. If same
(a) (b) mass m is connected by both springs as shown in
K1 K 2 K1K 2 / K1 K 2 figure then time-period t is given by the relation
(c) (d)
(a)When external force is applied, one spring gets extended t t1 t2
and another one gets contracted by the same distance hence (a)
force due to two springs act in same direction.
t1.t2
t
F F1 F2 kx k1x k2x k k1 k2 t1 t2
i.e. (b)
148 A mass m attached to a spring oscillates every
t2 t12 t22 m
2 sec. If the mass is increased by 2 kg, then time- (c)
period increases by 1 sec. The initial mass is t2 t12 t22
(d)
(a) 1.6 kg (b) 3.9 kg
(c) 9.6 kg (d) 12.6 kg m m
t1 2 t2 2
K1 K2
m T2 m2 3 m 2 9 m 2 (d) and
T 2
(a) k T1 m1
2 m 4 m Equivalent spring constant for shown combination is
m
8 t 2
m kg 1.6 kg K1 K 2
5 K1 + K2. So time period t is given by
m 2K
149.A mass M is suspended
K by two springs of force t2 t12 t22
K2 By solving these equations we get
constants K1 and K2 respectively as shown in the
diagram. The total elongation (stretch) of the two 152Two springs of force constants K and 2K are
K1
springs is connected to a mass as shown below. The
frequency of oscillation of the mass is
Mg
K1 K 2
(a)
Mg (K1 K 2)
m
K1K 2
(b)
Mg K1K 2
K1 K 2
(c)
K1 K 2
(1 / 2 ) (K /m) (1 / 2 ) (2K /m)
K1K 2Mg (a) (b)
(d)
(1 / 2 ) (3K /m) (1 / 2 ) (m/K )
k1k2 (c) (d)
keq
k1 k2
(b)For series combination 1 K efective 1 (K 2K ) 1 3K
n
F keqx mg
k1k2
x x mg(k1 k2) (c) 2 m 2 m =
2 m
k k k1k2
1 2
150 The frequency of oscillation of the springs
shown in the figure will be
K

SRI MARUTHI P.U COLLEGE, HOSKOTE I PUC PART-2 CET/NEET STUDY


MATERIAL
Waves/Sound, Oscillations/SHM, Mechanical Properties of Matter/Elasticity, Fluid Mechanics (Fluid
Statics, Fluid Dynamics, Viscosity, Surface Tension),
Thermal Properties Matter/Thermometry (Thermal Expansion, Calorimetry, Transmission of Heat),
Thermo Dynamics, Kinetic Theory of Gases (KTG)
k1 k2 rigidly fixed. A mass of 0.25 kg is attached at the
153 TwoA springs of constant and are joined in
middle of the spring and is slightly displaced along
series. The effective spring constant of the
the length. The time period of the oscillation of the
combination is given by
mass is
k1k2 (k1 k2) / 2
(a) (b)
s s
k k k1k2 /(k1 k2) (a) 20 (b) 10
(c) 1 2 (d)
(d)In series combination
K s s
1 1 1 k2 k1

kk
kS 1 2 (c) 5 (d) 200
kS k1 k2 k1k2 k1 k2
. (a)System is equivalent to parallel combination of
K eq K 1 K 2 400
154 The springs shown are identical. When A 4kg, springs and

the elongation of spring is 1 cm. If B 6kg, the T 2


m
2
0.25

elongation produced by it is K eq 400 20

K
157 When a mass m is attached to a spring, it
normally extends by 0.2 m. The mass m is given a
slight addition extension and released, then its
time period will be
1
(a) B7 sec (b) 1 sec
2 2
(c) 7 sec (d) sec
3
m x
Kx mg
(a)4 cm (b) 3 cm (c) K g
(c)2 cm (d) 1 cm
m x 0.2 2
T 2 2 2 sec
(b) F kx
mg kx m kx K g 9.8 7
So
m1 k1 x1

4

k

1 158 Infinite springs with force constant k, 2k, 4k
m2 k2 x2 6 k / 2 x2 and 8k.... respectively are connected in series. The
Hence
x2 3 cm.
effective force constant of the spring will be

155 When a body of mass 1.0 kg is suspended (a)2K (b) k


from a certain light spring hanging vertically, its
(c)k/2 (d) 2048
length increases by 5 cm. By suspending 2.0 kg
1 1 1 1 1
block to the spring and if the block is pulled ....
kef k 2 k 4 k 8 k
through 10 cm and released the maximum (c)
velocity in it in m/s is : (Acceleration due to gravity 1 1 1 1 1 1
1 ..... 2
k 2 4 8 k 1 1/ 2 k
10m/s2)
(By using sum of infinite geometrical progression
(a) 0.5 (b) 1 a a a
a 2 ...
(c) 2 (d) 4 r r sum (S) 1 r )
(b)Initially when 1 kg mass is suspended then by k
kef .
mg 1 10 N 2
k 2
200
using F kx mg kx x 5 10 m 159 Two masses m1 and m2 are suspended
Further, the angular frequency of together by a massless spring of constant K. When
oscillation of 2 kg mass is the masses are in equilibrium, m1 is removed
k 200 without disturbing the system. The amplitude of
10rad/ sec
M 2 oscillations is
m1
vmax a (10 102) 10 1m / s m1g
Hence, m2

156 A weightless spring of length 60 cm and force (a) K


constant 200 N/m is kept straight and unstretched m2 g
on a smooth horizontal table and its ends are (b) K
SRI MARUTHI P.U COLLEGE, HOSKOTE I PUC PART-2 CET/NEET STUDY
MATERIAL
Waves/Sound, Oscillations/SHM, Mechanical Properties of Matter/Elasticity, Fluid Mechanics (Fluid
Statics, Fluid Dynamics, Viscosity, Surface Tension),
Thermal Properties Matter/Thermometry (Thermal Expansion, Calorimetry, Transmission of Heat),
Thermo Dynamics, Kinetic Theory of Gases (KTG)
(m1 m2 )g FL 1
l l 2
(c) K (c) AY r (F, L and Y are constant)
(m1 m2 )g l2 r1
2

(2)2 4
(d) K l1 r2 l2 4l1 4cm

m2 2 1011 N / m2
(a)With mass alone, the extension of the spring l is given
164 If Young's modulus of iron is and
m2g kl the interatomic spacing between two molecules is
as ...(i)
3 1010 metre, the interatomic force constant is
(m1 m2) l is given by
With mass , the extension
(a) 60 N/m (b) 120 N/m
(m1 m2)g k(l l) (c) 30 N/m (d) 180 N/m
....(ii)

The increase in extension is l which is the (a)Interatomic force constant


K Y r0
amplitude of vibration. Subtracting (i) from (ii), we
get 2 1011 3 1010 60 N / m
m1g 165 The diameter of a brass rod is 4 mm and
m1g kl l
or k
9 1010 N / m2
160 A spring executes SHM with mass of 10kg Young's modulus of brass is . The force
attached to it. The force constant of spring is required to stretch by 0.1% of its length is
10N/m.If at any instant its velocity is 40cm/sec, 360N
the displacement will be (where amplitude is (a) (b) 36 N
0.5m) 3
(c) 144 10 N
5
(d) 36 10 N
(a) 0.09 m (b) 0.3 m
(c) 0.03 m (d) 0.9 m YAl 9 1010 4 106 0.1
F 360 N
(a) L 100
k 10 1

m 10 166 If x longitudinal strain is produced in a wire of
(b)Angular velocity
Young's modulus y, then energy stored in the
v2 (0.4)2 material of the wire per unit volume is
2 2 y2 a2 (0.5)2

v a y 2 12
Now 2 2yx2
2 y 0.3m (a) yx (b)
y 0.9 =
1 2 1 2
161 The S.H.M. of a particle is given by the yx yx
(c) 2 (d) 2
y 3 sin t 4 cos t
equation . The amplitude is 1
Stress Strain
(a) 7 (b) 1 (d)Energy stored per unit volume 2

(c) 5 (d) 12 1 2 1
Young's modulus (Strain) Y x2
2 2
32 42 5 167 A and B are two wires. The radius of A is twice
(c)Resultant amplitude
3. Mechanical Properties of Matter / that of B. They are stretched by the some load.
Elasticity Then the stress on B is
162 The ratio of the lengths of two wires A and B (a) Equal to that on A (b)Four times that on
A
of same material is 1 : 2 and the ratio of their
diameter is 2 : 1. They are stretched by the same (c) Two times that on A (d) Half that on A
force, then the ratio of increase in length will be force 1
Stress 2
Stress
(a) 2 : 1 (b) 1 : 4 (b) Area r

(c) 1 : 8 (d) 8 : 1 SB rA
2

(2)2 SB 4S A
l1 L1 d2
2
2 S A rB
FL L 1 1 1
l l
AY d l 2 L 2 d1 2 2 8
(c)
168 Two wires of equal lengths are made of the
163 The displacement of a particle varies according
same material. Wire A has a diameter that is twice
to the relation x = 4(cost + sint). The amplitude as that of wire B. If identical weights are
of the particle is suspended from the ends of these wires, the
(a) 8 (b) 4 increase in length is
(a) Four times for wire A as for wire B
(c) 4 (d) 4 2 (b) Twice for wire A as for wire B
SRI MARUTHI P.U COLLEGE, HOSKOTE I PUC PART-2 CET/NEET STUDY
MATERIAL
Waves/Sound, Oscillations/SHM, Mechanical Properties of Matter/Elasticity, Fluid Mechanics (Fluid
Statics, Fluid Dynamics, Viscosity, Surface Tension),
Thermal Properties Matter/Thermometry (Thermal Expansion, Calorimetry, Transmission of Heat),
Thermo Dynamics, Kinetic Theory of Gases (KTG)
(c) Half for wire A as for wire B FL FL FL
l 2 l 2
(d) One-fourth for wire A as for wire B (a) AY r Y r (Y = constant)
FL 1 2 2
l l 2 l2 F2 L2 r1 1
(d) AY r (F,L and Y are same) 2 2 1
l1 F1 L1 r2 2
2 2

l A rB r 1 lA
B l l
l B rA 2rB 4 l A 4lB lB
4 2 1 i.e. increment in its length will be l.
or
169 Steel and copper wires of same length are 173 A wire extends by 1 mm when a force is
applied. Double the force is applied to another
stretched by the same weight one after the other.
wire of same material and length but half the
Young's modulus of steel and copper are
radius of cross-section. The elongation of the wire
2 1011 N / m2 1.2 1011 N / m2 in mm will be
and . The ratio of increase
in length (a)8 (b) 4
2 3 (c)2 (d) 1
(a) 5 (b) 5
FL F
5 5 l 2
l 2
a) r r r (Y and L are constant)
(c) 4 (d) 2 2
FL l Y l2 F2 r1
2 (2)2 8
l
AY
S cu
lcu YS l1 F1 r2 l2 8l1 8 1 8mm
(b) (F,L and Y are constant)
ls 1.2 1011 3 3mm2
174 A wire of cross-sectional area is first
11
lcu 2 10 5 stretched between two fixed points at a
170 Increase in length of a wire is 1 mm when temperature of 20C. Determine the tension when
suspended by a weight. If the same weight is the temperature falls to 10C. Coefficient of linear
suspended on a wire of double its length and 5 1 Y 2 10 N / m 11 2
expansion 10 C and
double its radius, the increase in length will be
(a) 2 mm (b) 0.5 mm (a) 20 N (b) 30 N
(c) 4 mm (d) 0.25 mm (c) 60 N (d) 120 N
2 1011 3 106 105 (2010) 60N
l
FL L
l 2 (c) F YA t
(b) AY r (F and Y are same) 175 Two wires A and B are of same materials.
l2 L2 r1
2 2 Their lengths are in the ratio 1 : 2 and diameters
1 1 l1 l
2
l L1 r2 2 2 l2 0.5mm. FA
1 2 2 are in the ratio 2 : 1 when stretched by force
1kg / mm2 FB
171 Longitudinal stress of is applied on a and respectively they get equal increase in
wire. The percentage increase in length is F /F
their lengths. Then the ratio A B should be
11 2
(Y 10 N / m )
(a) 1 : 2 (b) 1 : 1
(a) 0.002 (b) 0.001 (c) 2 : 1 (d) 8 : 1
(c) 0.003 (d) 0.01
l r2
6 F Y A F
l stress 10 (d) L L (Y and l are constant)
11 10 5
(b)Longitudinal strain L Y 10 2 2
FA rA LB 2 2 8
5
Percentage increase in length 10 100 0.001% FB rB
LA 1 1 1
172 A wire of length L and radius r is rigidly fixed
10cm3
at one end. On stretching the other end of the wire 176 A wire of length 2 m is made from of
with a force F, the increase in its length is l. If copper. A force F is applied so that its length
another wire of same material but of length 2L and increases by 2 mm. Another wire of length 8 m is
radius 2r is stretched with a force of 2F, the made from the same volume of copper. If the force
increase in its length will be F is applied to it, its length will increase by
(a) l (b) 2l (a) 0.8 cm (b) 1.6 cm
l l (c) 2.4 cm (d) 3.2 cm
(c) 2 (d) 4 l
FL

FL2

FL2
(d) AY (AL)Y VY
SRI MARUTHI P.U COLLEGE, HOSKOTE I PUC PART-2 CET/NEET STUDY
MATERIAL
Waves/Sound, Oscillations/SHM, Mechanical Properties of Matter/Elasticity, Fluid Mechanics (Fluid
Statics, Fluid Dynamics, Viscosity, Surface Tension),
Thermal Properties Matter/Thermometry (Thermal Expansion, Calorimetry, Transmission of Heat),
Thermo Dynamics, Kinetic Theory of Gases (KTG)
2 (c) 2 : 1 (d) 1 : 1
l L If volume of the wire remains constant
1
l2 L 2
2
8
2 W Fl
16 (a) 2 Wl (F is constant)
l1 L1 2
W1 l1 l 1

l2 16 l1 16 2 32mm 3.2cm W2 l2 2l 2

5 182 A 5 metre long wire is fixed to the ceiling. A
177 The compressibility of water is 4 10 per unit
weight of 10 kg is hung at the lower end and is 1
atmospheric pressure. The decrease in volume of
metre above the floor. The wire was elongated by
100 cubic centimeter of water under a pressure of
1 mm. The energy stored in the wire due to
100 atmosphere will be
stretching is
4 105 cc (a) Zero (b) 0.05 joule
(a) 0.4 cc (b)
(c) 0.025 cc (d) 0.004 cc (c) 100 joule (d) 500 joule
1 V/V 1 1
C W F l mgl
(a) K P V C P V (b) 2 2

4 105 100 100 0.4cc 1


10 10 1 101 0.05 J
2
178 If a rubber ball is taken at the depth of 200 m
183 The ratio of Young's modulus of the material
in a pool, its volume decreases by 0.1%. If the
of two wires is 2 : 3. If the same stress is applied
1 103 kg / m3 g 10m / s2 on both, then the ratio of elastic energy per unit
density of the water is and ,
2 volume will be
then the volume elasticity in N / m will be (a) 3 : 2 (b) 2 : 3
(a) 10
8
(b) 2 10
8
(c) 3 : 4 (d) 4 : 3
2
9 9 (stress)
(c) 10 (d) 2 10
(a)Energy per unit volume = 2Y
P hg 200 103 10 E1 Y2 E1 3
K 2 109
d) V / V V / V 0.1/ 100 E2 Y1 E2 2
(Stress is constant)
179 A ball falling in a lake of depth 200 m shows
184 The length of a rod is 20 cm and area of
0.1% decrease in its volume at the bottom. What
is the bulk modulus of the material of the ball 2cm2
cross-section . The Young's modulus of the
8 2 10 2
19.6 10 N / m 19.6 10 N /m 1.4 1011 N / m2
(a) (b) material of wire is . If the rod is
19.6 1010 N / m2 19.6 108 N / m2 compressed by 5 kg-wt along its length, then
(c) (d)
increase in the energy of the rod in joules will be
6
(a) 8.57 10
4
(b) 22.5 10
p hg 200 103 9.8 5 5
B (c) 9.8 10 (d) 45.0 10
V/V 0.1/100 1/1000
(a) 1 1 FL 1 F 2L
8 2 Fl F
19.6 10 N/m
(a)Energy = 2 2 AY 2 AY
180 A cube of aluminium of sides 0.1 m is 1 (50)2 20 102
subjected to a shearing force of 100 N. The top 6
2 2 10 4 1.4 1011 8.57 10 J
face of the cube is displaced through 0.02 cm with 185 When a 4 kg mass is hung vertically on a light
respect to the bottom face. The shearing strain spring that obeys Hooke's law, the spring stretches
would be by 2 cms. The work required to be done by an
(a) 0.02 (b) 0.1 external agent in stretching this spring by 5 cms
(c) 0.005 (d) 0.002 (g 9.8 metres/ sexc2 )
will be
x 0.02cm
(a) 4.900 joule (b) 2.450 joule
(d)Shearing strain L 10cm 0.002
(c) 0.495 joule (d) 0.245 joule
181 Two wires of same diameter of the same
F 40
material having the length l and 2l. If the force F is K 0.2 N/m
(b) x 2 10 2
applied on each, the ratio of the work done in the
1 2 1
two wires will be Kx (0.2) (0.05)2 2.5 J
Work done 2 2
(a) 1 : 2 (b) 1 : 4
SRI MARUTHI P.U COLLEGE, HOSKOTE I PUC PART-2 CET/NEET STUDY
MATERIAL
Waves/Sound, Oscillations/SHM, Mechanical Properties of Matter/Elasticity, Fluid Mechanics (Fluid
Statics, Fluid Dynamics, Viscosity, Surface Tension),
Thermal Properties Matter/Thermometry (Thermal Expansion, Calorimetry, Transmission of Heat),
Thermo Dynamics, Kinetic Theory of Gases (KTG)
186 The work per unit volume to stretch the
(c) 2rTcos W (d) W 2rT cos
length by 1% of a wire with cross sectional area of
1mm2 [Y 9 1011 N / m2 ] (c) T T
will be.
9 1011 J 4.5 107 J
(a) (b)
7 4.5 1011 J
(c) 9 10 J (d)
2
1 1 1 Weight of metal disc = total upward force
U )2 9 1011
Y (Strain
2 2 100
(b) = upthrust force + force due to surface tension
7
4.5 10 J = weight of displaced water + T cos (2 r)
187 Two rods of different materials having = W + 2 rT cos
1, 2 190 A 10 cm long wire is placed horizontally on the
coefficients of linear expansion and Young's
surface of water and is gently pulled up with a
Y Y
moduli 1 and 2 respectively are fixed between force of 2 10-2 N to keep the wire in equilibrium.
two rigid massive walls. The rods are heated such The surface tension, in Nm-1, of water is
that they undergo the same increase in (a) 0.1 (b) 0.2
temperature. There is no bending of rods. If (c) 0.001 (d) 0.002
1 : 2 2 : 3 2 102
, the thermal stresses developed in the T
F
0.1 N / m
Y : Y2 (a) 2l 2 10 10 2
two rods are equally provided 1 is equal to
191 The force required to take away a flat circular
(a) 2 : 3 (b) 1 : 1 plate of radius 2 cm from the surface of water, will
(c) 3 : 2 (d) 4 : 9 be (the surface tension of water is 70 dyne/cm)
(c)Thermal stress = Y . (a)
280 dyne
(b)
250 dyne
If thermal stress and rise in temperature are equal
1 140 dyne 210 dyne
Y (c) (d)
then
(a)Force required, F 2rT 2 2 70 280 .Dyne
Y1 2 3
192 Energy needed in breaking a drop of radius R
Y2 1 2
into n drops of radii r is given by
188 The stress versus strain graphs for wires of
two materials A and B are as shown in the figure. If 4 3
2 2 (r n R 2)
YA
and
YB
are the Young s modulii of the (a) 4T(nr R ) (b) 3
materials, then 2 2 2 2
(c) 4T(R nr ) (d) 4T(nr R )
Y
YB 2Y A
(a) (a)Energy needed = Increment in surface energy
A
Y A YB = (surface energy of n small drops)
(b)
Stress

B (surface energy of
YB 3Y A 60o one big drop)
(c)
30o
Y A 3YB O Strain
X n4r 2T 4R2T 4T(nr2 R2)
(d)
193 A drop of liquid of diameter 2.8 mm breaks up
into 125 identical drops. The change in energy is
nearly (S.T. of liquid =75 dynes/cm)
Y A tan A tan60 3
3 Y A 3YB (a) Zero (b) 19 erg
YB tan B tan30 1/ 3
(d)
(c) 46 erg (d) 74 erg
4.Fluid Mechanics -Surface Tension 2 1/3
(d) E 4R T(n 1)
189 A thin metal disc of radius r floats on water
surface and bends the surface downwards along 1 2 1/ 3 74 erg
= 4 3.14 (1.4 10 ) 75(125 1)
the perimeter making an angle with vertical 194 Radius of a soap bubble is 'r', surface tension
edge of the disc. If the disc displaces a weight of of soap solution is T. Then without increasing the
water W and surface tension of water is T, then temperature, how much energy will be needed to
the weight of metal disc is double its radius
(a) 2rT W (b) 2rT cos W
SRI MARUTHI P.U COLLEGE, HOSKOTE I PUC PART-2 CET/NEET STUDY
MATERIAL
Waves/Sound, Oscillations/SHM, Mechanical Properties of Matter/Elasticity, Fluid Mechanics (Fluid
Statics, Fluid Dynamics, Viscosity, Surface Tension),
Thermal Properties Matter/Thermometry (Thermal Expansion, Calorimetry, Transmission of Heat),
Thermo Dynamics, Kinetic Theory of Gases (KTG)
2 2 (a) 1000 : 1 (b) 1 : 1000
(a) 4r T (b) 2r T
(c) 10 : 1 (d) 1 : 10
2 2
(c) 12r T (d) 24r T (d) As volume remain constant therefore
W 8T(R2 R2) 8T[(2r)2 (r)2] 24r 2T R n1/ 3r
(d) 2 1

195 Work done in splitting a drop of water of 1 mm energyof onebigdrop 4R2T


surface

radius into 106 droplets is (Surface tension of surface
energyof n drop n 4r 2T
3 2
water 72 10 J / m ) R2

n2 / 3r 2 1

1

1
1/ 3
nr 2
nr 2
= n (1000)1/ 3 10
9.58 105 J 8.95 105 J
(a) (b)
200 A big drop of radius R is formed by 1000 small
5.89 105 J 6 droplets of water, then the radius of small drop is
(c) (d) 5.98 10 J
(a) R/2 (b) R/5
(b)Work done in splitting a water drop of radius R
2 1/ 3
(c) R/6 (d) R/10
into n drops of equal size 4R T(n 1) 4 3 4
R 1000 r 3
3 2 3
4 (10 ) 72 10 (10 6/ 3
1) (d) 3 3 (As volume remains constant)
R
4 106 72 103 99 8.95 105 J 3 3
R 10r r
R 1000r 10
196 The surface tension of a liquid is 5 N/m. If a
thin film of the area 0.02 m2 is formed on a loop, 201 8000 identical water drops are combined to
then its surface energy will be form a big drop. Then the ratio of the final surface
energy to the initial surface energy of all the drops
5 102 J 2.5 102 J together is
(a) (b)
(a) 1 : 10 (b) 1 : 15
2 101 J 5 101 J
(c) (d) (c) 1 : 20 (d) 1 : 25
(c) W T A 5 2 (0.02) (Film has two free surfaces) (c) As volume remains constant R 3 8000r 3
2 101 J R 20r
197 What is ratio of surface energy of 1 small drop
Surfaceenergyof onebigdrop 4R2T
and 1 large drop, if 1000 small drops combined to
energyof 8000smalldrop 80004r 2T
Surface
form 1 large drop
(a) 100 : 1 (b) 1000 : 1

R2

20r 2
1
2 2
(c) 10 : 1 (d) 1 : 100 8000r 8000r 20
(d)Volume of liquid remain same i.e. volume of (Q202)8 mercury drops coalesce to form one
1000 small drops will be equal to volume of one mercury drop, the energy changes by a factor of
big drop
(a) 1 (b) 2
4 4 r 1
n r 3 R 3 (c) 4 (d) 6
1000r R R 10r
3 3
3 3 R 10
(c) As volume remains constant therefore
energyof onesmalldrop 4r 2T
surface 1 R n1/ 3r

energyof onebigdrop 4R2T 100
surface
Energyof bigdrop 4R 2T R 2
2
198 The amount of work done in forming a soap Energyof smalldrop 4r 2T
2/ 3
r (8) 4
10cm 10cm 203 A film of water is formed between two straight
film of size is (Surface tension parallel wires of length 10cm each separated by 0.5
T 3 102 N / m) cm. If their separation is increased by 1 mm while
still maintaining their parallelism, how much work
6 104 J 3 104 J will have to be done (Surface tension of water =
(a) (b)
7.2 102 N / m)
6 103 J 3 104 J
(c) (d)
7.22 106 Joule 1.44 105 Joule
2 4 4 (a) (b)
(a) E T A 3 10 2(100 10 ) 6 10 J
2.88 105 Joule 5.76 105 Joule
199 One thousand small water drops of equal radii (c) (d)
combine to form a big drop. The ratio of final A2 A1
surface energy to the total initial surface energy is (b)Increment in area of soap film =
2 [(10 0.6) (10 0.5)] 104 2 104 m2
SRI MARUTHI P.U COLLEGE, HOSKOTE I PUC PART-2 CET/NEET STUDY
MATERIAL
Waves/Sound, Oscillations/SHM, Mechanical Properties of Matter/Elasticity, Fluid Mechanics (Fluid
Statics, Fluid Dynamics, Viscosity, Surface Tension),
Thermal Properties Matter/Thermometry (Thermal Expansion, Calorimetry, Transmission of Heat),
Thermo Dynamics, Kinetic Theory of Gases (KTG)
Work done = T A 209 The pressure inside a small air bubble of
radius 0.1 mm situated just below the surface of
7.2 102 2 104 1.44 105 J water will be equal to
204 The surface tension of soap solution is 3 1
[Take surface tension of water 70 10 Nm and
25 103 Nm1
. The excess pressure inside a soap 5 2
atmospheric pressure = 1.013 10 Nm ]
bubble of diameter 1 cm is
3
(b) 1.027 10 Pa
3
(a) 10 Pa (b) 20 Pa (a) 2.054 10 Pa
(c) 5 Pa (d) None of the above 5
(c) 1.027 10 Pa
5
(d) 2.054 10 Pa
4T
P 2T
r
(b)Excess pressure r
(c)Excess pressure inside the air bubble
4 2 25 103
2T 2 70 103
1 10 2 20 N/m2 20 Pa asr d/2 Pin Pout 1400Pa
r 0.1 10 3
205 If the surface tension of a soap solution is
0.03 MKS units, then the excess of pressure inside P 1400 1.013 105
1.027 10 Pa 5
in
a soap bubble of diameter 6 mm over the
210 Two capillary tubes P and Q are dipped in
atmospheric pressure will be
water. The height of water level in capillary P is 2/3
(a) Less than 40 N/m2 (b)Greater than 40 to the height in Q capillary. The ratio of their
2
N/m diameters is
(c) Less than 20 N/m2 (d)Greater than 20 (a) 2 : 3(b) 3 : 2(c)3 : 4 (d)4 : 3
2
N/m
1 r hQ h 2
4T r P
P 40 N / m2 h rQ hP 2 3
r h
(b) (b) 3
206 The pressure of air in a soap bubble of 0.7cm
5. Fluid Mechanics pascals law,
diameter is 8 mm of water above the pressure
Archimedes principle, Bernoulli principle
outside. The surface tension of the soap solution is
&applications, Viscosity:-
(a) 100dyne/ cm (b) 211. A uniformly tapering vessel is filled with a
68.66dyne/ cm liquid of density 900 kg/m3. The force that acts on
the base of the vessel due Area to =the
103m2liquid is
(c) 137dyne/ cm (d)
(g 10ms2 )
150dyne/ cm
(a) 3.6 N 0.4 m
4T rhdg 0.35 0.8 1 103 (b) 7.2 N
P hdg T Area=2 103m2
b) r 4 4
(c) 9.0 N
70 dyne/cm 68.66 dyne/cm (d) 14.4 N

207 A spherical drop of water has radius 1 mm If


surface If the radius of a soap bubble is four times b)Force acting on the base
that of another, then the ratio of their pressures F P A hdgA 0.4 900 10 2 103 7.2N
will be 212. The height of a mercury barometer is 75 cm
(a) 1 : 4 (b) 4 : 1 at sea level and 50 cm at the top of a hill. Ratio of
(c) 16 : 1 (d) 1 : 16 density of mercury to that of air is 10 4. The height
of the hill is
1 P1 r2 r 1
P (a) 250 m (b) 2.5 km
r P2 r1 4r 4
(a)
(c) 1.25 km (d) 750 m
3
208 tension of water is 70 10 N/m difference of (b)Difference of pressure between sea level and the
pressures between inside and out side of the top of hill
spherical drop is (h h ) g (75 50) 102 Hg g
P 1 2 Hg (i)
35 N / m2 70 N / m2 140 N / m2
(a) (b) (c) (d)Zero and pressure difference due to h meter of air
2T 2 70 103 h air g
P 140N/m2 P = (ii)
(c) R 1 10 3
By equating (i) and (ii) we get
h air g (75 50) 102 Hg g
SRI MARUTHI P.U COLLEGE, HOSKOTE I PUC PART-2 CET/NEET STUDY
MATERIAL
Waves/Sound, Oscillations/SHM, Mechanical Properties of Matter/Elasticity, Fluid Mechanics (Fluid
Statics, Fluid Dynamics, Viscosity, Surface Tension),
Thermal Properties Matter/Thermometry (Thermal Expansion, Calorimetry, Transmission of Heat),
Thermo Dynamics, Kinetic Theory of Gases (KTG)
Hg h 76
h 25 10 2 l
2 cos60 1/2
air 25 10 10 2500m
4

Height of the hill = 2.5 km. l 152cm

213. Density of ice is

and that of water is . 217. The height to which a cylindrical vessel be
What will be the decrease in volume when a mass filled with a homogeneous liquid, to make the
M of ice melts average force with which the liquid presses the
M side of the vessel equal to the force exerted by the
liquid on the bottom of the vessel, is equal to
(a)
(b) M
(a) Half of the radius of the vessel
1 1 1 1 1
M (b) Radius of the vessel
M
(c) (d) (c)One-fourth of the radius of the vessel
(d)Three-fourth of the radius of the vessel
M M




(b) Pressure at the bottom = hg
(c)Volume of ice , volume of water .
1
M M 1 1 hg
M and pressure on the vertical surface = 2

Change in volume Now, according to problem
Force at the bottom = Force on the vertical surface
214. Equal masses of water and a liquid of density 1
2 are mixed together, then the mixture has a hg r 2 hg 2rh
2 h=r
density of
218. A vertical U-tube of uniform inner cross
(a) 2/3 (b) 4/3
section contains mercury in both sides of its arms.
(c) 3/2 (d) 3 A glycerin (density = 1.3 g/cm3) column of length
10 cm is introduced into one of its arms. Oil of
(b)If two liquid of equal masses and different density 0.8 gm/cm3 is poured into the other arm
densities are mixed together then density of until the upper surfaces of the oil and glycerin are
mixture in the same horizontal level. Find the length of the
21 2 2 1 2 4 oil column, Density of mercury = 13.6 g/cm3

1 2 1 2 3
Glycerine

Oil h
215. When a large bubble rises from the bottom of (a) 10.4 cm 10 cm
a lake to the surface. Its radius doubles. If
(b) 8.2 cm
atmospheric pressure is equal to that of column of
(c) 7.2 cm
water height H, then the depth of lake is
Mercury
(d) 9.6 cm
(a) H (b) 2H
(c) 7H (d) 8H
(d)
4 4
Glycerine

Oil
(P0 hg) r 3 P0 (2r)3 h
P1V1 P2V2 3 3 10 cm
(c) = 10h
Where, h = depth of lake A B
Hg
h 7 7H . Mercury
hg 7P0 g

216. A barometer tube reads 76 cm of mercury. If At the condition of equilibrium
the tube is gradually inclined at an angle of 60 o Pressure at point A = Pressure at point B
with vertical, keeping the open end immersed in PA PB
the mercury reservoir, the length of the mercury 10 1.3 g h 0.8 g (10 h) 13.6 g
column will be By solving we get h = 9.7 cm
(a) 152 cm (b) 76 cm d, 2d
219. Three liquids of densities and 3d are
(c) 38 cm (d) 38 3cm mixed in equal volumes. Then the density of the
l mixture is
60 (a) d (b) 2d
h
cos60 h h (c) 3d (d) 5d
(a) l
m1 m2 m3 V(d 2d 3d)

Water mix 3V 3V
(b) = = 2d.
SRI MARUTHI P.U COLLEGE, HOSKOTE I PUC PART-2 CET/NEET STUDY
MATERIAL
Waves/Sound, Oscillations/SHM, Mechanical Properties of Matter/Elasticity, Fluid Mechanics (Fluid
Statics, Fluid Dynamics, Viscosity, Surface Tension),
Thermal Properties Matter/Thermometry (Thermal Expansion, Calorimetry, Transmission of Heat),
Thermo Dynamics, Kinetic Theory of Gases (KTG)
d, 2d of relative density 0.8. The relative density of
220. Three liquids of densities and 3d are
silver is 10.5. The tension in the string in kg-wt is
mixed in equal proportions of weights. The relative
(a) 1.6 (b) 1.94
density of the mixture is
(c) 3.1 (d) 5.25
11d 18d
M
(a) 7 (b) 11 V( )g ( )g

(b)Apparent weight
13d 23d
(c) 9 (d) 18
M 1 g 2.1 1
0.8
g 1.94g
10.5
3m 3m N

V1 V2 V3 m m m 3 6 18 = 1.94 Kg-wt
d d
mix d 2d 3d 11 11
(b) = =
224. The fraction of a floating object of volume
V0
221. An ice berg of density 900 Kg/m3 is floating in
water of density 1000 Kg/m3. The percentage of d
and density 0 above the surface of a liquid of
volume of ice-cube outside the water is
density d will be
(a) 20% (b) 35%
d0 dd0
(c) 10% (d) 25% d d0
(c) Let the total volume of ice-berg is V and its (a) d (b)
density is . If this ice-berg floats in water with d d0 dd0
d d0
Ving Vg (c) d (d)
volume Vin inside it then
d0
Vin V0
Vin V V0d0 g Vin d g d
(c)For the floatation
[ density of water]
d0 V d d0
Vout V0 Vin V0 V0 0
Vout V Vin V d d
or
Vout d d0
Vout 1000 900 1 .
V0 d

V 1000 10
225. A block of steel of size 5 cm 5 cm 5 cm is
Vout 10% weighed in water. If the relative density of steel is
of V
7, its apparent weight is
222. In making an alloy, a substance of specific
(a) 6 5 5 5 gf (b) 4 4 4 7 gf
s1 m1
gravity and mass is mixed with another (c) 5 5 5 7 gf (d) 4 4 4 6 gf
s m
substance of specific gravity 2 and mass 2 ; then (a) Apparent weight = V( )g
the specific gravity of the alloy is 6 5 5 5 gf
= 5 5 5(7 1)g =
m1 m2 s1s2
226. A ball whose density is 0.4 103 kg/m3 falls
s s m m
(a) 1 2 (b) 1 2 into water from a height of 9 cm . To what depth
does the ball sink
m1 m2 m1 m2
(a) 9 cm (b) 6 cm
m1 m2 s1 s2
(c) 4.5 cm (d) 2.25 cm
s1 s2 m1 m2
(c) (d) (b) The velocity of ball before entering the water
Densityof alloy surface

(c)Specific gravity of alloy Densityof water
v 2gh 2g 9
Massof alloy When ball enters into water, due to upthrust of water the

Volumeof alloy density
of water velocity of ball decreases (or retarded)
m1 m2 apparent
weight
m1 m2 m m2
1


m1 m2
w m 1 m 2 m 1 m The retardation, a = massof ball
2
1 2 1 / w 2 / w s1 s2
V( )g
g 0.4 1 g 3 g

density
of substance V 0.4 2
As specific
gravityof substance
density
of water If h be the depth upto which ball sink, then,
223. A silver ingot weighing 2.1 kg is held by a 3
0 v2 2 g h
string so as to be completely immersed in a liquid 2 2g 9 3gh h = 6 cm.
SRI MARUTHI P.U COLLEGE, HOSKOTE I PUC PART-2 CET/NEET STUDY
MATERIAL
Waves/Sound, Oscillations/SHM, Mechanical Properties of Matter/Elasticity, Fluid Mechanics (Fluid
Statics, Fluid Dynamics, Viscosity, Surface Tension),
Thermal Properties Matter/Thermometry (Thermal Expansion, Calorimetry, Transmission of Heat),
Thermo Dynamics, Kinetic Theory of Gases (KTG)
227.An ice block contains a glass ball when the ice 231. Air is streaming past a horizontal air plane
melts within the water containing vessel, the level wing such that its speed in 120 m/s over the upper
of water surface and 90 m/s at the lower surface. If the
density of air is 1.3 kg per metre3 and the wing is
(a) Rises (b) Falls
10 m
(c) Unchanged (d) First rises and then
long and has an average width of 2 m, then the
falls difference of the pressure on the two sides of the
Ans(b) wing of
228. Two water pipes of diameters 2 cm and 4 cm (a) 4095.0 Pascal (b) 409.50 Pascal
are connected with the main supply line. The (c) 40.950 Pascal (d) 4.0950 Pascal
velocity of flow of water in the pipe of 2 cm (a)From the Bernoulli's theorem
diameter is 1 1
P1 P2 (v22 v12) 1.3 [(120)2 (90)2]
(a) 4 times that in the other pipe 2 2

1 4095N / m2
or Pascal
(b) 4 times that in the other pipe 232. There is a hole of area A at the bottom of
(c) 2 times that in the other pipe cylindrical vessel. Water is filled up to a height h
and water flows out in t second. If water is filled to
1 a height 4h, it will flow out in time equal to
(d) 2 times that in the other pipe
(a) t (b) 4t
d 2cm d 4 cm r 1 cm r 2 cm
(a) A and B A and B (c) 2 t (d) t/4
From equation of continuity, av = constant
A 2H
vA aB (rB )2 2
2 t
vA 4vB A0 g
2 (c)Time required to emptied the tank
vB aA (rA) 1
t2 H2 4h
229. An incompressible liquid flows through a 2
t1 H1 h t2 2t
horizontal tube as shown in the following fig. Then
the velocity v of the fluid is 233. A square plate of 0.1 m side moves parallel to
1.5 A
v A a second plate with a velocity ofv2 0.1
= 1.5 m/s,
m/s both
plates being immersed in water. If the viscous
force is 0.002 N and the coefficient of viscosity is
v1 = 3 m/s A
0.01 poise, distance between the plates in m is
(a) 0.1 (b) 0.05
(c) 0.005 (d) 0.0005
A (0.1)2 0.01m2 , 0.01 Poise 0.001 decapoise
(a) 3.0 m/s (b) 1.5 m/s (d)
(c) 1.0 m/s (d) 2.25 m/s (M.K.S. unit),
(c)If the liquid is incompressible then mass of liquid entering dv = 0.1 m/s and F = 0.002 N
through left end, should be equal to mass of liquid coming out dv
from the right end. F A
dx
M m1 m2 Av1 Av2 1.5A . v
Adv 0.001 0.01 0.1
dx 0.0005m
A 3 A 1.5 1.5A . v v 1 m/ s F 0.002 .

234. Two drops of the same radius are falling
230. Water is moving with a speed of 5.18 ms1
through air with a steady velocity of 5 cm per sec.
through a pipe with a cross-sectional area of 4.20
If the two drops coalesce, the terminal velocity
cm2. The water gradually descends 9.66 m as the
would be
pipe increase in area to 7.60 cm2. The speed of
flow at the lower level is (a)10 cm per sec (b)2.5 cm per sec
(a) 3.0 ms1 (b) 5.7 ms1 1/ 3
(c) 5 (4) cm per sec (d) 5 2 cm per
(c) 3.82 ms1 (d) 2.86 ms1 sec
a1v1 a2v2 (c)If two drops of same radius r coalesce then
(d)
radius of new drop is given by R
4.20 5.18 7.60 v2 v2 2.86 m / s
4 3 4 3 4 3
R r r 3 3 1/ 3
3 3 3 R 2r R 2 r
SRI MARUTHI P.U COLLEGE, HOSKOTE I PUC PART-2 CET/NEET STUDY
MATERIAL
Waves/Sound, Oscillations/SHM, Mechanical Properties of Matter/Elasticity, Fluid Mechanics (Fluid
Statics, Fluid Dynamics, Viscosity, Surface Tension),
Thermal Properties Matter/Thermometry (Thermal Expansion, Calorimetry, Transmission of Heat),
Thermo Dynamics, Kinetic Theory of Gases (KTG)
If drop of radius r is falling in viscous medium then 237. We have two (narrow) capillary tubes T1 and
2 T2. Their lengths are l1 and l2 and radii of cross-
it acquire a critical velocity v and v r
section are r1 and r2 respectively. The rate of flow
2 2
v2 R 21 / 3r of water under a pressure difference P through


v1 r r tube T1 is 8cm3/sec. If l1 = 2l2 and r1 =r2, what will
be the rate of flow when the two tubes are
v2 22 / 3 v1 22 / 3 (5) 5 (4)1 / 3m / s
connected in series and pressure difference across
235. A ball of radius r and density falls freely the combination is same as before (= P)
under gravity through a distance h before entering (a) 4 cm3/sec (b) (16/3) cm3/sec
water. Velocity of ball does not change even on (c) (8/17) cm3/sec (d) None of these
entering water. If viscosity of water is , the value Pr4 8cm3
V
of h is given by 8l sec
(b)
2 2 1 For composite tube
r g
9 h
(a) P r 4 2 Pr 4
V1
l 3 8l 2 16 cm3
2 2 1 8 l 8
r g 2
81 3 3 sec
(b)
l
2 4 1
2
l1 l 2l2 or l2 2
r g
81
(c) 238. Water is flowing through a tube of non-
uniform cross-section ratio of the radius at entry
2 and exit end of the pipe is 3 : 2. Then the ratio of
2 4 1
r g velocities at entry and exit of liquid is
9
(d) (a) 4 : 9 (b) 9 : 4
(c)Velocity of ball when it strikes the water surface (c) 8 : 27 (d) 1 : 1
v 2gh (a) If velocities of water at entry and exit points
(i)
are v1 and v2, then according to equation of
Terminal velocity of ball inside the water
continuity,
2 2 1
v r g v1 A2 r2
2
2
2
4
9
(ii) A1v1 A2v2 v A1 r1 3 9
2
2 r 2g
2gh ( 1) 239. At what speed the velocity head of a stream
9
Equating (i) and (ii) we get
of water be equal to 40 cm of Hg
2
2 4 1 (a) 282.8 cm/sec (b) 432.6 cm/sec
h r g
81
(c) 632.6 cm/sec (d) 832.6 cm/sec
236. A liquid is flowing in a horizontal uniform
capillary tube under a constant pressure difference
P. The value of pressure for which the rate of flow v2
h
of the liquid is doubled when the radius and length v 2gh
(a) 2g
both are doubled is
3P 2 103 40 = 2 2 102 = 282.8 cm/s
(a) P (b) 4 240. A sniper fires a rifle bullet into a gasoline tank
P P making a hole 53.0 m below the surface of
(c) 2 (d) 4 gasoline. The tank was sealed at 3.10 atm. The
stored gasoline has a density of 660 kgm3. The
Pr 4 V8l
V P velocity with which gasoline begins to shoot out of
8l r 4
(d)From the hole is
4 4
P2 V2 l2 r1 1 1 27.8 ms1 41.0 ms1
2 2 (a) (b)
P1 V1 l1 r2 2 4

9.6ms1 19.7ms1
P P (c) (d)
P2 1
4 4.
(b) According to Bernoulli's theorem,
SRI MARUTHI P.U COLLEGE, HOSKOTE I PUC PART-2 CET/NEET STUDY
MATERIAL
Waves/Sound, Oscillations/SHM, Mechanical Properties of Matter/Elasticity, Fluid Mechanics (Fluid
Statics, Fluid Dynamics, Viscosity, Surface Tension),
Thermal Properties Matter/Thermometry (Thermal Expansion, Calorimetry, Transmission of Heat),
Thermo Dynamics, Kinetic Theory of Gases (KTG)
1 pr4
PB hg PA v2A V
2 (As vA vB ) V P r4 andl
8l
(d) ( are constants)
1 2
3.10P 53 660 10 P 660vA 4
2 V2 P2 r2 1
4

2 1
V2
Q
1 V1 P1 r1 2 8 8
5 5
2
660 vA = =
2.1 1.01 10 3.498 10 2
1
5.619 105 660 v2A
2

2 5.619 105 (244) A liquid flows in a tube from left to right as


vA
660 shown
= 41 m/s
241. An L-shaped tube with a small orifice is held A1 A2
and
are the cross-sections of the
in a water stream as shown in fig. The upper end in figure.
of the tube is 10.6 cm above the surface of water. portions of the tube as shown. Then the ratio of
What will be the height of the jet of water coming v1 /v2
speeds will be
from the orifice? Velocity of water stream is 2.45
m/s A1 / A2
(a) A1
A2
(a) Zero v1 v2
2.45 m/s A2 / A1
(b) 20.0 cm (b)
(c) 10.6 cm
A2 / A1
(d) 40.0 cm (c)

A1 / A2
(d)
v2
h
2g Ans(b)
(b)According to Bernoulli's theorem,
245 A liquid flows through a horizontal tube. The
(2.45)2
h 0.314 31.4 cm velocities of the liquid in the two sections, which
2 10
Height of jet coming from orifice A1 A2 v1
have areas of cross-section and , are and
31.4 10.6 20.8 cm
= v2
respectively. The difference in the levels of the
242. Two capillary tubes of the same length but liquid in the two vertical tubes is h
different radii r1 and r2 are fitted in parallel to the
(a) The volume of the liquid flowing through
bottom of a vessel. The pressure head is P. What
should be the radius of a single tube that can the tube in unit time is
A1v1
h
replace the two tubes so that the rate of flow is
same as before v2 v1 2gh
(b) A1 v1 A2

r1 r2 r12 r22 v2
(a) (b) v22 v12 2gh
(c)
r14 r24
(c) (d) None of these
V V1 V2 (d) The energy per unit mass of the liquid is
(d)
the same in both sections of the tube
Pr 4 Pr14 Pr24
246. A manometer connected to a closed tap
8l 8l 8l r 4 r14 r24
5
reads 4.5 10 pascal. When the tap is opened the
r (r14 r24 )1 / 4
5
reading of the manometer falls to 4 10 pascal.
243. Water flows in a streamlined manner through Then the velocity of flow of water is
a capillary tube of radius a, the pressure difference 1 1
being P and the rate of flow Q. If the radius is (a) 7 ms (b) 8 ms
reduced to a/2 and the pressure increased to 2P, 1 1
(c) 9 ms (d) 10 ms
the rate of flow becomes
P1 P2 v2 4.5 105 4 105 v2

(a) 4Q (b) Q 103 g 2g
(d) g 2g v=10m/s
Q Q 247. A vessel contains oil (density = 0.8 gm/cm3)
(c) 4 (d) 8 over mercury (density = 13.6 gm/cm3). A
homogeneous sphere floats with half of its volume
SRI MARUTHI P.U COLLEGE, HOSKOTE I PUC PART-2 CET/NEET STUDY
MATERIAL
Waves/Sound, Oscillations/SHM, Mechanical Properties of Matter/Elasticity, Fluid Mechanics (Fluid
Statics, Fluid Dynamics, Viscosity, Surface Tension),
Thermal Properties Matter/Thermometry (Thermal Expansion, Calorimetry, Transmission of Heat),
Thermo Dynamics, Kinetic Theory of Gases (KTG)
immersed in mercury and the other half in oil. The 249. Water is filled in a cylindrical container to a
density of the material of the sphere in gm/cm3 is height of 3m. The ratio of the cross-sectional area
(a) 3.3 (b) 6.4 of the orifice and the beaker is 0.1. The square of
(c) 7.2 (d) 12.8 the speed of the liquid coming out from the orifice
is (g = 10 m/s2)
(c) (a) 50 m2/s2

Oil
(b) 50.5 m2/s23m
(c) 51 m2/s2 52.5 cm
Mercury 2 2
(d) 52 m /s

As the sphere floats in the liquid. Therefore its (a)Let A = cross-section of tank
weight will be equal a = cross-section hole
to the upthrust force on it V = velocity with which level decreases
Weight of sphere v = velocity of efflux
4 3 v A
R g
3 (i)
3m
...... (i) B
52.5 cm
Upthrust due to oil and mercury
2 3 2
R oil g R3 Hg g
3 3 (ii) av
av AV V
Equating (i) and (ii) From equation of continuity A
4 3 2 2 By using Bernoulli's theorem for energy per unit
R g R 3 0.8g R 3 13.6g
3 3 3 volume
2 0.8 13.6 14.4 7.2 Energy per unit volume at point A
= Energy per unit volume at
248 A liquid is kept in a cylindrical vessel which is
point B
being rotated about a vertical axis through the
centre of the circular base. If the radius of the 1 2 1
P gh V P 0 v2
2 2
vessel is r and angular velocity of rotation is ,
2gh 2 10 (3 0.525)
then v2 2
2
2
50(m/sec)
a 1 (0.1)
1
A
the difference in the heights of the liquid at the
centre of the vessel and the edge is 250(Q-i)Why a small iron needle sinks in water
while a large iron ship floats
r r 2 2 Ans.For floatation, the density of body must be
lesser or equal to
(a) 2g (b) 2g that of liquid. In case of iron needle, the density of
2 needle, i.e., iron
2gr 2 is more than that of water, so it will sink. However,
(c) h (d) 2gr the density of a
(b)From Bernoulli's
B theorem, ship due to its large volume is lesser than that of
A r
1 1 water, so it will
PA dv2A dghA PB dvB2 dghB float.
2 2
hA hB (Q-ii)A man is sitting in a boat which is floating in
Here,
a pond. If the
1 2 1
PA dvA PB dvB2 man drinks some water from the pond, what will
2 2
happen to the level
1 2 2 of water in the pond
PA PB d[vB vA ]
2 Ans:-If the man drinks m g of water from the
pond, the weight of
vA 0, vB r PA PB hdg
Now, and (boat + man) system will increase by mg and so
the system will
1 2 2 r 2 2
hdg dr h displace mg more water for floating. So due to
2 or 2g
removal of water
SRI MARUTHI P.U COLLEGE, HOSKOTE I PUC PART-2 CET/NEET STUDY
MATERIAL
Waves/Sound, Oscillations/SHM, Mechanical Properties of Matter/Elasticity, Fluid Mechanics (Fluid
Statics, Fluid Dynamics, Viscosity, Surface Tension),
Thermal Properties Matter/Thermometry (Thermal Expansion, Calorimetry, Transmission of Heat),
Thermo Dynamics, Kinetic Theory of Gases (KTG)
from pond, the water level in pond will fall but due V1 (M / W)
to water i.e,
displaced by the floating system the water level in (a) Now if the unloaded pieces floats in the pond,
the pond will rise m V2 W
the water displaced by them ,
and so the water removed from the pond is equal
to the water V2 (m / W )
i.e,
displaced by the system; the level of water in So the total water displaced by the boat and the
the pond will remain floating pieces
unchanged. M m
V1 V2
W W .....(ii)
(Q-iii) A boy is carrying a fish in one hand and a
bucket full of Which is same as the water displaced by the
water in the other hand. He then places the fish in floating system initially (eqn. 1); so the level of
the bucket water in the pond will remain unchanged.
thinking that in accordance with Archimedes' (b) Now if the unloaded pieces sink the water
principle he is now displaced by them will be equal to their own
carrying less weight as the weight of the fish will volume, i.e,
reduce due to m m
upthrust. Is he right V2 as V

Ans. No, when he places the fish in water in the
bucket, no doubt the and so in this situation the total volume of water
weight of fish is reduced due to upthrust, but the displaced by boat and sinking pieces will be
weight of (water + M m
V1 V2
bucket) system is increased by the same amount,
W .....(iii)
so that the total
weight carried by him remains unchanged. Now as the pieces are sinking
W , so this
volume will be lesser than initial water displaced by
(Q-iv) A bucket of water is suspended from a the floating system (eq. 1); so the level of water
spring balance. Does in the pond will go down (or fall)
the reading of balance change (a) when a piece of In this problem if the pieces (either sinking
stone suspended or floating) are
from a string is immersed in the water without unloaded on the ground, the water displaced after
touching the bucket?
(b) when a piece of iron or cork is put in the water unloading, 2
V M / W , will be lesser than before
in the bucket? V (M m) / W ; so the level of water in
Ans.(a) Yes, the reading of the balance will unloading.
increase but the increase the pond will fall.
in weight will be equal to the loss in weight of the
(251) In which one of the following cases will the
stone (Vg) and not the weight of stone liquid flow in a pipe be most streamlined
(Vg)[ Vg as ]
.
(a)Liquid of high viscosity and high density flowing
(b)Yes, the reading of the balance will increase but
through a pipe of small radius
the increase in
(b)Liquid of high viscosity and low density flowing
weight will be equal to the weight of iron or cork
through a pipe of small radius
piece.
(c)Liquid of low viscosity and low density flowing
through a pipe of large radius
(Q-v) A boat containing some pieces of material is
(d)Liquid of low viscosity and high density flowing
floating in a pond.
through a pipe of large radius
What will happen to the level of water in the pond
Solution : (b)For streamline flow Reynold's
if on unloading
the pieces in the pond, the piece (a) floats (b) r
NR
sinks?
number

Ans.If M is the mass of boat and m of pieces in it,
should be less.
M m VD W
then initially as the system is floating N
For less value of R , radius and density should be
M m small and
VD
.ie the system displaces water W W ...... viscosity should be high.
(i)
When the pieces are dropped in the pond, the (252)(Q-i)Two different liquids are flowing in two
tubes of equal
M V1 W, radius. The ratio of coefficients of viscosity of
boat will still float, so it displaces water
liquids is 52:49 and
SRI MARUTHI P.U COLLEGE, HOSKOTE I PUC PART-2 CET/NEET STUDY
MATERIAL
Waves/Sound, Oscillations/SHM, Mechanical Properties of Matter/Elasticity, Fluid Mechanics (Fluid
Statics, Fluid Dynamics, Viscosity, Surface Tension),
Thermal Properties Matter/Thermometry (Thermal Expansion, Calorimetry, Transmission of Heat),
Thermo Dynamics, Kinetic Theory of Gases (KTG)
the ratio of their densities is 13:1, then the ratio of poise, distance between the plates in m is
their critical [EAMCET (Med.) 2003]
velocities will be (a) 0.1 (b) 0.05 (c) 0.005 (d) 0.0005
Solution : (d)
(a)4 : 49(b)49 : 4(c) 2 : 7(d)7 : 2 A (0.1)2 0.01m2 , 0.01 Poise 0.001 decapoise
Solution : (a)
(M.K.S. unit), dv = 0.1 m/s and F = 0.002 N

v NR dv Adv 0.001 0.01 0.1
r F A dx 0.0005m
Critical velocity dx F 0.002 .
v1 1 2 52 1 4
(Q-v) Spherical balls of radius 'r' are falling in a
v2 2 1 49 13 49
. viscous fluid of
viscosity '' with a velocity 'v'. The retarding
(Q-ii)Air is streaming past a horizontal air plane viscous force acting on
wing such that its the spherical ball is
speed in 120 m/s over the upper surface and 90 (a)Inversely proportional to 'r' but directly
m/s at the lower proportional to velocity 'v'
surface. If the density of air is 1.3 kg per metre3 (b)Directly proportional to both radius 'r' and
and the wing is 10 m velocity 'v'
long and has an average width of 2 m, then the (c)Inversely proportional to both radius 'r' and
difference of the velocity 'v'
pressure on the two sides of the wing of (d)Directly proportional to 'r' but inversely
(a)4095.0 Pascal(b) 409.50 Pascal proportional to 'v'
(c)40.950 Pascal(d) 4.0950 Pascal F 6 rv
Solution : (b)
Solution:(a)From the Bernoulli's theorem
(Q-VI)Two drops of the same radius are falling
1 1
P1 P2 (v22 v12 ) 1.3 [(120)2 (90)2 ] 4095N / m2 through air with a
2 2 steady velocity of 5 cm per sec. If the two drops
or coalesce, the
Pascal terminal velocity would be
(a)10 cm per sec (b) 2.5 cm per
(Q-iii) A cylindrical tank has a hole of 1 cm2 in its sec
bottom. If the
1/ 3
water is allowed to flow into the tank from a tube (c) 5 (4) cm per sec(d) 5 2 cm per sec
above it at the Solution : (c)If two drops of same radius r
rate of 70 cm3/sec. then the maximum height up coalesce then radius of
to which water can new drop is given by R
rise in the tank is
(a)2.5 cm(b)5 cm(c)10 cm(d)0.25 cm 4 3 4 3 4 3
Solution : (a)The height of water in the tank R r r
3 3 3
becomes maximum
3 3 1/ 3
when the volume of water flowing into the tank R 2r R 2 r
per second becomes If drop of radius r is falling in viscous medium then
equal to the volume flowing out per 2
second.Volume of water flowing it acquire a critical velocity v and v r
2 2
A 2gh v2 R 21/ 3 r
out per second = Av
v1 r
and volume of water flowing in per second r
70cm3 / sec. v2 22 / 3 v1 22 / 3 (5) 5 (4)1 / 3 m/ s

A 2gh 70 1 2gh 70 1 2 980 h 70
4900 (Q-VII)A ball of radius r and density falls freely
h 2.5 cm. under gravity through a distance h before entering
1960
water. Velocity of ball does not change even on
entering water. If viscosity of water is , the value
(Q-iv) A square plate of 0.1 m side moves parallel of h is given by
to a second plate
with a velocity of 0.1 m/s, both plates being 2 2 1 2 2 1
r g r g
immersed in water. If 9 81 h
(a) (b)
the viscous force is 0.002 N and the coefficient of
viscosity is 0.01
SRI MARUTHI P.U COLLEGE, HOSKOTE I PUC PART-2 CET/NEET STUDY
MATERIAL
Waves/Sound, Oscillations/SHM, Mechanical Properties of Matter/Elasticity, Fluid Mechanics (Fluid
Statics, Fluid Dynamics, Viscosity, Surface Tension),
Thermal Properties Matter/Thermometry (Thermal Expansion, Calorimetry, Transmission of Heat),
Thermo Dynamics, Kinetic Theory of Gases (KTG)
2 2 254. Oxygen boils at 183C. This temperature is
2 4 1 2 4 1
r g r g approximately
81 9 (a) 215F (b) 297F
(c) (d)
Solution : (c) (c) 329F (d) 361F
Velocity of ball when it strikes the water surface C F 32 183 F 32

v 2gh ..........(i) (b) 5 9 5 9 F 297F
Terminal velocity of ball inside the water 255. Recently, the phenomenon of
2
v r 2g
1 superconductivity has been observed at 95 K. This
temperature is nearly equal to
9
..........(ii) (a) 288F (b) 146F
Equating (i) and (ii) we get (c) 368F (d) +178F
2
2 r 2g 2 4 1
2gh ( 1) h r g
9 81 F 32 K 273

F 32 95 273


(a) 9 5 9 5 F 288F

(252) (Q-i)A liquid is flowing in a horizontal 256. Thermoelectric thermometer is based on


uniform capillary tube (a) Photoelectric effect (b)Seeback effect
under a constant pressure difference P. The value (c) Compton effect (d) Joule effect
of pressure for (b)Thermoelectric thermometer is based on
which the rate of flow of the liquid is doubled when Seeback Effect.
the radius and
257. A constant volume gas thermometer shows
length both are doubled is
pressure reading of 50cm and 90cm of mercury at
3P P P
0C and 100C respectively. When the pressure
(a)P (b) 4 (c) 2 (d) 4 reading is 60 cm of mercury, the temperature is
Pr 4 (a) 25C (b) 40C
V
8l
Solution : (d)From (c) 15C (d) 12.5C
4
P2 V2 l 2 r1 (Pt P0) (60 50)
V 8l t 100C 100 25C
P
r 4 P1 V1 l1 r2 (a) (P100 P0) (90 50)

258. A vertical column 50 cm long at 50C
4
1 P1 P 1 balances another column of same liquid 60 cm
2 2 P2
2 4 4. 4 long at 100C. The coefficient of absolute
(Q-ii)A capillary tube is attached horizontally to a expansion of the liquid is
constant head (a) 0.005/C (b) 0.0005/C
arrangement. If the radius of the capillary tube is
(c) 0.002/C (d) 0.0002/C
increased by 10%
then the rate of flow of liquid will change nearly by h1 1 (1 1) 0
(a)+ 10%(b)+ 46%(c) 10% (d) 40%
h2 2 (1 2) (1 )
4
(a)
Pr 4 V2 r2 50 1 50
V
8l V1 r1 60 1 100
Solution : (b) 0.005/ C
110
4 259. A uniform metal rod is used as a bar
V2 V1 pendulum. If the room temperature rises by 10C,
100 V1(1.1)4 1.4641V
and the coefficient of linear expansion of the metal
V V2 V1 1.4641V V of the rod is 2 106 per C, the period of the
0.46 or 46%
V V V . pendulum will have percentage increase of
6.Thermal Properties of (a) 2 103 (b) 1 103
Matter/Thermometry (Thermal
(c) 2 103 (d) 1 103
Expansion, Calorimetry, Transmission of
Heat) (d)Fractional change in period
253. On the Celsius scale the absolute zero of T 1 1
2 10 6 10 10 5
temperature is at T 2 2
(a) 0C (b) 32C T
100 10 5 100 10 3 %
(c) 100C (d) 273.15C % change T
260. A bar of iron is 10 cm at 20C. At 19C it will
(d) T 273.15 tC 0 273.15 tC t 273.15C
be ( of iron = 11 106/C)
SRI MARUTHI P.U COLLEGE, HOSKOTE I PUC PART-2 CET/NEET STUDY
MATERIAL
Waves/Sound, Oscillations/SHM, Mechanical Properties of Matter/Elasticity, Fluid Mechanics (Fluid
Statics, Fluid Dynamics, Viscosity, Surface Tension),
Thermal Properties Matter/Thermometry (Thermal Expansion, Calorimetry, Transmission of Heat),
Thermo Dynamics, Kinetic Theory of Gases (KTG)
(a) 11 106 cm longer (b) 11 106 cm 2 105
10 5 / C
shorter (d) 2 2
(c) 11 105 cm shorter (d)11 105 cm longer 265. Density of substance at 0C is 10 gm/cc and
L1 1 ( )1 at 100C, its density is 9.7 gm/cc. The coefficient

L L0(1 ) L2 1 ( )2 of linear expansion of the substance will be
(c)
(a) 102 (b) 102
6
10 1 11 10 20 (c) 103 (d) 104

L2 1 11 10 6 19 L2 9.99989
(d)Coefficient of volume expansion
Length is shorten by ( ) (10 9.7)
1 2 3 10 4
10 9.99989 0.00011 11 10 cm 5 .T .( ) 10 (100 0)
Hence, coefficient of linear expansion
261. A pendulum clock keeps correct time at 0C.

Its mean coefficient of linear expansions is / C ,
3
10 4 / C
then the loss in seconds per day by the clock if the
266. Coefficient of real expansion of mercury is
temperature rises by tC is
0.18 103/C. If the density of mercury at 0C is
13.6 gm/cc. its density at 473K is
1
t 864000 (a) 13.11 gm/cc (b) 26.22 gm/cc
2
t 1 (c) 52.11 gm/cc (d) None of these
1 t 86400
(a) 2 (b) 2
0(1 . ) 13.6[1 0.18 103(473 273)]
1 (a)
t 86400 1
2 t 86400 13.6[1 0.036] 13.11gm/ cc
2 2 .
t t
1 1 267. The real coefficient of volume expansion of
(c) 2 (d) 2
glycerine is 0.000597 perC and linear coefficient
T 1 1 of expansion of glass is 0.000009 perC. Then the
(t 0)
(b)Loss in time per second T 2 2 apparent volume coefficient of expansion of
loss in time per day glycerine is
1 1 1 (a) 0.000558 perC (b) 0.00057 perC
t t t t (24 60 60) t 86400
2 2 2 (c) 0.00027 perC (d) 0.00066 perC
262. If the length of a cylinder on heating real app. vessel
(b)As we know
increases by 2%, the area of its base will increase
app. glycerine glass
by
(a) 0.5% (b) 2% 0.000597 0.000027 0.00057/ C
(c) 1% (d) 4% 268. The volume of a metal sphere increases by
A L A 0.24% when its temperature is raised by 40C. The
2
2 2 2 4% coefficient of linear expansion of the metal
(d) A L A L A .
is .......... C
263. The volume of a gas at 20C is 100 cm3 at (a) 2 105 (b) 6 105
normal pressure. If it is heated to 100C, its
(c) 2.1 105 (d) 1.2 105
volume becomes 125 cm3 at the same pressure,
then volume coefficient of the gas at normal
pressure is V 0.24
6 105 / C
a) V.T 100 40
(a) 0.0015/C (b) 0.0045/C

(c) 0.0025/C (d) 0.0033/C 2 10 5 / C
3
V1 1 t1 100 1 20
269. The length of a metallic rod is 5m at 0C and
V2 1 t2 125 1 100
(d) = 0.0033/C becomes 5.01 m, on heating upto 100C. The
264. The coefficient of superficial expansion of a linear expansion of the metal will be
solid is 2 105 /C. It's coefficient of linear (a) 2.33 105 /C (b) 6.0 105 /C
expansion is (c) 4.0 105 /C (d) 2.0 105 /C
(a) 4 105/C (b) 3 105/C
(c) 2 105/C (d) 1 105/C L 0.01
2 10 5 / C
L0 5 100
(d)
(Q3 = mcW) (Q2 = mcW)
ice
0C

(Q3 = mLV)
SRI
Steam at 100C MARUTHI
ice P.U COLLEGE, HOSKOTE
Water at100CI PUC PART-2 CET/NEET STUDY
0C
MATERIAL
Waves/Sound, Oscillations/SHM, Mechanical Properties of Matter/Elasticity, Fluid Mechanics (Fluid
Statics, Fluid Dynamics, Viscosity, Surface Tension),
Thermal Properties Matter/Thermometry (Thermal Expansion, Calorimetry, Transmission of Heat),
Thermo Dynamics, Kinetic Theory of Gases (KTG)
270. Two rods, one of aluminum and the other Heat lost = Heat gain
l l 80 1 (30 0) m 80 m 30gm
made of steel, having initial length 1 and 2 are
(Q2 = mLf)
connected together to form a single rod of length
l1 l2 274. Two spheres made of same substance have
. The coefficients of linear expansion for

diameters in the ratio 1 : 2. Their thermal
aluminum and steel are a and s respectively. If capacities
Steam at 100Care in the ratio of
the length of each rod increases by the same (a) 1 : 2 (b) 1 : (Q4
8 = mLV)
o
amount when their temperature are raised by t C , (c) 1 : 4 (d) 2 : 1
l1 (b)Thermal capacity = Mass Specific heat
(l1 l2) Due to same material both spheres will have same
then find the ratio
specific heat. Also mass =Volume (V)
s a
Density ()
a s
(a) (b) Ratio of thermal capacity
s a 4 3
( a s) ( a s) r1 3 3
m1 V1 r 1
(c) (d) 3 1 1: 8
m2 V2 4
r23 r2 2
3
l1 l2 l1 at l2 st
(c)Given or 275. Work done in converting one gram of ice at
l1 s l1 s 10C into steam at 100C is

l2 a or l1 l2 a s
.
(a) 3045 J (b) 6056 J
271. How much heat energy is gained when 5 kg
of water at 20C is brought to its boiling point (c) 721 J (d) 616 J
(Specific heat of water = 4.2 kJ kg1c1) (a)Ice (10C) converts into steam as follows
(a) 1680 kJ (b) 1700 kJ (ci = Specific heat of ice, cW = Specific heat of
(c) 1720 kJ (d) 1740 kJ water)

(a) Q m.c. 5 (1000 4.2) (100 20)


1680 103 J 1680kJ
272. Heat required to convert one gram of ice at
0C into steam at 100C is (given Lsteam = 536
cal/gm)
(a) 100 calorie (b) 0.01 kilocalorie
(c) 716 calorie (d) 1 kilocalorie
Water at 0C

(c)Conversion of ice (0C) into steam (100C) is as


follows Q Q1 Q2 Q3 Q4
Total heat required
Q 1 0.5(10) 1 80 1 1 (100 0) 1 540
725cal
(Q1 = mLi)
W JQ 4.2 725 3045J
Hence work done
276. 50 gm of copper is heated to increase its
temperature by 10C. If the same quantity of heat
is given to 10 gm of water, the rise in its
temperature is (Specific heat of copper = 420
Joule-kg1 C1)
Q1 Q2 Q3 (a) 5C (b) 6C
Heat required in the given process
(c) 7C (d) 8C
1 80 1 1 (100 0) 1 536 716cal (a)Same amount of heat is supplied to copper and
273. 80 gm of water at 30C are poured on a large mccc c mWcW W
block of ice at 0C. The mass of ice that melts is water so
mccc( )c 50 103 420 10
(a) 30 gm (b) 80 gm W 5C
mWcW 10 10 3 4200
(c) 1600 gm (d) 150 gm
277. Two liquids A and B are at 32C and 24C.
(a)If m gm ice melts then When mixed in equal masses the temperature of
SRI MARUTHI P.U COLLEGE, HOSKOTE I PUC PART-2 CET/NEET STUDY
MATERIAL
Waves/Sound, Oscillations/SHM, Mechanical Properties of Matter/Elasticity, Fluid Mechanics (Fluid
Statics, Fluid Dynamics, Viscosity, Surface Tension),
Thermal Properties Matter/Thermometry (Thermal Expansion, Calorimetry, Transmission of Heat),
Thermo Dynamics, Kinetic Theory of Gases (KTG)
the mixture is found to be 28C. Their specific 281. A body of mass 5 kg falls from a height of 30
heats are in the ratio of metre. If its all mechanical energy is changed into
(a) 3 : 2 (b) 2 : 3 heat, then heat produced will be
(c) 1 : 1 (d) 4 : 3 (a) 350 cal (b) 150 cal
(c) 60 cal (d) 6 cal

mix
AcA BcB (a) W JQ mgh J Q
cA cB
(c)Temperature of mixture mgh 5 9.8 30
Q 350cal
32 cA 24 cB J 4.2
28
cA cB
282. In supplying 400 calories of heat to a system,
cA 1 the work done will be

28c 28c 32c 24c cB 1 (a) 400 joules (b) 1672 joules
A B A B

278. A beaker contains 200 gm of water. The heat (c) 1672 watts (d) 1672 ergs
capacity of the beaker is equal to that of 20 gm of W JQ 4.18 400 1672joule
(b)
water. The initial temperature of water in the
beaker is 20C. If 440 gm of hot water at 92C is 283. The height of a waterfall is 84 metre.
poured in it, the final temperature (neglecting Assuming that the entire kinetic energy of falling
radiation loss) will be nearest to water is converted into heat, the rise in
(a) 58C (b) 68C temperature of the water will be
(c) 73C (d) 78C (g 9.8 m / s2, J 4.2 joule/ cal)
(b)Heat lost by hot water = Heat gained by cold
water in beaker + Heat absorbed by beaker (a) 0.196C (b) 1.960C
(c) 0.96C (d) 0.0196C
440(92 ) 200 ( 20) 20 ( 20)
mgh J (m.c. )
(a) W JQ
68C
279. A liquid of mass m and specific heat c is gh
0.0023h 0.0023 84 0.196C
heated to a temperature 2T. Another liquid of Jc
mass m/2 and specific heat 2c is heated to a 284.Of two masses of 5 kg each falling from height
temperature T. If these two liquids are mixed, the of 10 m, by which 2kg water is stirred. The rise in
resulting temperature of the mixture is temperature of water will be
(a) (2/3)T (b) (8/5)T
(a) 2.6C (b) 1.2C
(c) (3/5)T (d) (3/2)T
(c) 0.32C (d) 0.12C
(d)Temperature of mixture
m (d) W JQ (2m)gh J mc
m c 2T (2c)T
m1c11 m2c2 2 2 3
mix
m1c1 m2c2

m
T
2 2 5 10 10 4.2(2 1000 )
m.c (2c)
2
0.1190C 0.12C
280.A water fall is 84 metres high. If half of the
285. A lead ball moving with a velocity V strikes a
potential energy of the falling water gets
wall and stops. If 50% of its energy is converted
converted to heat, the rise in temperature of water
into heat, then what will be the increase in
will be
temperature (Specific heat of lead is S)
(a) 0.098C (b) 0.98C
(c) 9.8C (d) 0.0098C 2V 2 V2 V2 V 2S

1 gh (a) JS (b) 4 JS (c) J (d) 2J


(mgh) J mc
(a)As W JQ 2 2Jc V2
1 1 2
mV J mS

9.8 84
0.098C (b) W JQ 2 2 4 JS
2 4.2 1000
286. A lead bullet of 10 g travelling at 300 m/s
cal strikes against a block of wood and comes to rest.
(cwater 1000 )
kg C Assuming 50% of heat is absorbed by the bullet,
g the increase in its temperature is
0.0023
JcW (Specific heat of lead = 150J/kg, K)
Short trick : Remember the value of ,
1 1 (a) 100C(b)125C(c)150C(d)200C
(0.0023)h 0.0023 84 0.098C
here 2 2
SRI MARUTHI P.U COLLEGE, HOSKOTE I PUC PART-2 CET/NEET STUDY
MATERIAL
Waves/Sound, Oscillations/SHM, Mechanical Properties of Matter/Elasticity, Fluid Mechanics (Fluid
Statics, Fluid Dynamics, Viscosity, Surface Tension),
Thermal Properties Matter/Thermometry (Thermal Expansion, Calorimetry, Transmission of Heat),
Thermo Dynamics, Kinetic Theory of Gases (KTG)
(c)Since specific heat of lead is given in Joules, t 2
t
hence use W Q instead of W JQ . (c) 2 (d) 3

1 1 2 v2 (300)2 t
mv m.c. 150C m1c11 m2c2 2 m s(2t) 1.5 (m s) 3
2 2 4c 4 150 . mix
m1c1 m2c2

m s 1.5 (m s)
t
(b)
287. 1 g of a steam at 100C melt how much ice
290. The heat is flowing through two cylindrical
at 0C? (Latent heat of ice = 80 cal/gm and latent
rods of same material. The diameters of the rods
heat of steam = 540 cal/gm)
are in the ratio 1 : 2 and their lengths are in the
(a) 1 gm(b)2 gm (c)4 gm(d)8 gm ratio 2 : 1. If the temperature difference between
(d)Suppose m gm ice melted, then heat required their ends is the same, the ratio of rate of flow of
mL m 80cal heat through them will be
for its melting
(a) 1 : 1 (b) 2 : 1
Heat available with steam for being condensed
and then brought to 0C (c) 1 : 4 (d) 1 : 8
1 540 1 1 (100 0) 640cal Q KA Q A d2

Heat lost = Heat taken (d) t l t l l (d = Diameter of rod)
m 8 gm 2
640 m 80
2
(Q / t)1 d1 l 1 1 1
2
Short trick: You can remember that amount of (Q / t)2 d2 l1 2 2 8

steam (m') at 100C required to melt m gm ice at 291. Two identical square rods of metal are welded
m end to end as shown in figure (i), 20 calories of
m'
0C is 8 . heat flows through it in 4 minutes. If the rods are
m 8 m' 8 1 8 gm welded as shown in figure (ii), the same amount of
Here, heat will flow through the rods in
288. Calculate the amount of heat (in calories)
required to convert 5 gm of 0oC
ice at 0C to100oC
steam at
100C 0oC100oC
(ii)
(a) 3100 (b) 3200 (i)

(c) 3600 (d) 4200 (a) 1 minute (b) 2 minutes


(c)Ice (0C) converts into steam (100C) in (c) 4 minutes (d) 16 minutes
following three steps.
Q KA

ice (a) t l (l / KA) R (R = Thermal resistance)
0C (Q1 = mLi)

Water at 0C t R ( Q and are same)


(Q2 = tP RP R / 2 1
tS 4
mcW) tP 1 min.
tS RS 2R 4 4 4

(Q3 = mLV) RS R1 R2
(Series resistance and parallel resistance
Steam at
100C
R1R2
RP
Water at R1 R2
100C
292. The ratio of thermal conductivity of two rods
of different material is 5 : 4. The two rods of same
area of cross-section and same thermal resistance
Q Q1 Q2 Q3
Total heat required will have the lengths in the ratio
5 80 5 1 (100 0) 5 540 3600cal (a) 4 : 5 (b) 9 : 1
289. A liquid of mass M and specific heat S is at a (c) 1 : 9 (d) 5 : 4
temperature 2t. If another liquid of thermal K1 5

t A1 A2 K2 4
(d)Given and
capacity 1.5 times, at a temperature of 3 is added
to it, the resultant temperature will be l1 l l1 K1 5
2 .
R1 R2 K1A K 2 A l2 K 2 4
4
t
(a) 3 (b) t 293. Two rectangular blocks A and B of different
metals have same length and same area of cross-
T2

SRI MARUTHI P.U COLLEGE, HOSKOTE I PUC PART-2 CET/NEET STUDY


MATERIAL K2
Waves/Sound, Oscillations/SHM, Mechanical Properties of Matter/Elasticity, Fluid Mechanics (Fluid
Statics, Fluid Dynamics, Viscosity, Surface Tension),
Thermal Properties Matter/Thermometry (Thermal Expansion, Calorimetry, Transmission of Heat),
Thermo Dynamics, Kinetic Theory of Gases (KTG)
section. They are kept in such a way that their k1T1d2 k2T2d1 k1T1 k2d2
cross-sectional area touch each other. The k1d2 k2d1 d1 d2
temperature at one end of A is 100C and that of (a) (b)
B at the other end is 0C. If the ratio of their k1d1 k2d2 k1d1T1 k2d2T2
thermal conductivity is 1 : 3, then under steady T1T2
T T k1d1 k2d2
(c) 1 2 (d)
state,
the temperature of the junction in contact will be (a)In series both walls have same rate of heat flow.
(a)25C (b) Therefore
50C T1
dQ K1A(T1 ) K 2 A( T2)
(c)75C (d) 100C
dt d1 d2
K1 1
K1d2(T1 ) K 2d1( T2)
K2 3 K1 K K 2 3K
(a)It is given that then
K1d2T1 K 2d1T2

the temperature of the junction in contact K1 K1d2 K 2d1
K 1 1 K 2 2 1 100 3 0 100
d2
K1 K 2 1 3 4 d1 297. A slab consists of two parallel layers of
= 25C
copper and brass of the same thickness and
Q Junction temperature
having thermal conductivities in the ratio 1 : 4. If
Q
o
the free face of brass is at 100 C and that of
100C 0C o
A B copper at 0 C , the temperature of interface is
(a) 80C (b) 20C
(c) 60C (d) 40C
K 1 1 K 2 2
294. Two vessels of different materials are similar K1 K 2
(a)Temperature of interface =
in size in every respect. The same quantity of ice
K1 1
filled in them gets melted in 20 minutes and 30 (
K2 4
minutes. The ratio of their thermal conductivities If K1 = K then K2 = 4K)
will be K 0 4K 100

5K
(a) 1.5 (b) 1 = 80C
(c) 2/3 (d) 4 298. The temperature gradient in a rod of 0.5 m
o
long is 80 C / m. If the temperature of hotter end of
KA( 1 2 )t
Q ; o
the rod is 30 C , then the temperature of the cooler
l ( )
(a) in both the cases , A, l and 1 2
end is
K1 t1 30 3
o o
are same so Kt = constant K 2 t2 20 2
= 1.5. (a) 40 C (b) 10 C
295. The thickness of a metallic plate is 0.4 cm. o
(c) 10 C
o
(d) 0 C
o
The temperature between its two surfaces is 20 C . 1 2 30 2
80 80 2 10o C
The quantity of heat flowing per second is 50 (b) l 0.5
2
calories from 5cm area. In CGS system, the 299. Two rods of same length and material transfer
a given amount of heat in 12 seconds, when they
coefficient of thermal conductivity will be
are joined end to end. But when they are joined
(a) 0.4 (b) 0.6 lengthwise, then they will transfer same heat in
(c) 0.2 (d) 0.5 same conditions in
Q KA( ) 5 20 K 1 (a) 24 s (b) 3 s
50 K 0.2
(c) t l 0.4 5 (c) 1.5 s (d) 48 s
(d)Let the heat transferred be Q.
296. Two walls of thicknesses d1 and d2 and
thermal conductivities k1 and k2 are in contact. In
the steady state, if the temperatures at the outer
ll l
T T2
surfaces are 1 and , the temperature at the When rods are joined end to end. Heat transferred
common wall is KA
Q 12
by each rod l ..(i)
SRI MARUTHI P.U COLLEGE, HOSKOTE I PUC PART-2 CET/NEET STUDY
MATERIAL
Waves/Sound, Oscillations/SHM, Mechanical Properties of Matter/Elasticity, Fluid Mechanics (Fluid
Statics, Fluid Dynamics, Viscosity, Surface Tension),
Thermal Properties Matter/Thermometry (Thermal Expansion, Calorimetry, Transmission of Heat),
Thermo Dynamics, Kinetic Theory of Gases (KTG)
KA 2 1 1 2K1K 2
Q t K eq
When rods are joined lengthwise, 2l ..(ii) K K K K1 K 2
eq 1 2

t 48 s 304. The ends of two rods of different materials
From equation (i) and (ii) we get
300. Wires A and B have identical lengths and with their thermal conductivities, radii of cross-
sections and lengths all are in the ratio 1 : 2 are
have circular cross-sections. The radius of A is
maintained at the same temperature difference. If
r 2r the rate of flow of heat in the larger rod is 4
twice the radius of B i.e. A B . For a given

4 cal/ sec
temperature difference between the two ends, , that in the shorter rod in cal/ sec will be
both wires conduct heat at the same rate. The (a) 1 (b) 2
relation between the thermal conductivities is
(c) 8 (d) 16
given by
dQ
K A 4K B K A 2K B
(a) (b) dt K s rs2 ll
s

KA KB /2 KA KB /4 dQ K (r )d 2 dQ K l rl2 ls 1 1 2
(c) (d)
(a) dt dl dt l = 2 4 1
2
Q KA K A r 1 K
A B B KA B dQ
t l K B AA rB 4 4
(d) dQ dt 4
l
1
301. Two identical plates of different metals are s dt 4 4
joined to form a single plate whose thickness is
305. Two spheres of different materials one with
double the thickness of each plate. If the
double the radius and one-fourth wall thickness of
coefficients of conductivity of each plate are 2
the other, are filled with ice. If the time taken for
and 3 respectively, then the conductivity of
complete melting ice in the large radius one is 25
composite plate will be
minutes and that for smaller one is 16 minutes,
(a) 5 (b) 2.4 the ratio of thermal conductivities of the materials
(c) 1.5 (d) 1.2 of larger sphere to the smaller sphere is
(b)Thermal conductivity of composite plate (a) 4 : 5 (b) 5 : 4
2K 1K 2 2 2 3 12
K eq (c) 25 : 1 (d) 1 : 25
K1 K 2 2 3 5 =2.4
KA ( )t
302. If the radius and length of a copper rod are Q
(d) l
both doubled, the rate of flow of heat along the
rod increases Q and are same for both spheres hence
(a) 4 times (b) 2 times 2
l l l r
K larger t
(c) 8 times (d) 16 times K l s s
At r 2t K smaller ls rl tl
. It is given that
A r2 Q2 r22 l1
Q 1
Q1 r12 l2
(b) l l rl 2rs, ll 4 ls t 25 t 16
and 1 min, s min.
Q2 4 1 2
K larger 1 1 16 1
Q1 1 2 Q2 2Q1

K smaller 4 2 25 25
303. If two metallic plates of equal thicknesses
306. The ratio of the diameters of two metallic rods
K1 K2
and thermal conductivities and are put of the same material is 2 : 1 and their lengths are in
together face to face and a common plate is the ratio 1 : 4. If the temperature difference
constructed, then the equivalent thermal between their ends are equal, the rate of flow of
conductivity of this plate will be heat in them will be in the ratio
(a) 2 : 1 (b) 4 : 1
K1
K2
(c) 8 : 1 (d) 16 : 1
K 1K 2 2K 1K 2
Q KA( ) Q A r2
K1 K 2 K1 K 2
(a) (b)
(d) t l t l l
(K 12 K 22 )3 / 2 (K 12 K 22 )3 / 2
Q / t 1
2
r1 l2 2 2 4 16
K 1K 2 2K 1K 2
(c) (d) (Q / t) r
2 l1 1 1 1
2
2l l l
307. Two cylinders P and Q have the same length
Req R1 R2 K eqA K1A K 2 A
(b)In series and diameter and are made of different materials
having thermal conductivities in the ratio 2 : 3.
SRI MARUTHI P.U COLLEGE, HOSKOTE I PUC PART-2 CET/NEET STUDY
MATERIAL
Waves/Sound, Oscillations/SHM, Mechanical Properties of Matter/Elasticity, Fluid Mechanics (Fluid
Statics, Fluid Dynamics, Viscosity, Surface Tension),
Thermal Properties Matter/Thermometry (Thermal Expansion, Calorimetry, Transmission of Heat),
Thermo Dynamics, Kinetic Theory of Gases (KTG)
These two cylinders are combined to make a cylindrical bar shown in the figure. What will be
cylinder. the temperature at the junction of copper and
steel
One end of P is kept at 100C and another
100oC end of 0oC
o
Q at 0C. The temperature at the interface of PSteel
Copper (a) 75 C
and Q is o
o
18 cm 6 cm
o
(b) 67 C
(a) 30 C (b) 40 C o
(c) 33 C
o o
(c) 50 C (d) 60 C o
(d) 25 C
K 1 1 K 2 2

K1 K 2
(b)Temperature of interface K11l2 K 2 2l1

K1 2 K1l2 K 2l1
(a)Temperature of interface

K 2 3 K Cu 9K S . K S K1 K K Cu K 2 9K
where K1 = 2K and K2 = 3K It is given that So if then
2K 100 3K 0 200K
40C 9K 100 6 K 0 18 5400K
75C
2K 3K 5K
9K 6 K 18 = 72K
308. Two bars of thermal conductivities K and 3K 311 The lengths and radii of two rods made of
and lengths 1cm and 2cm respectively have equal same material are in the ratios 1 : 2 and 2 : 3
cross-sectional area, they are joined lengths wise respectively. If the temperature difference
as shown in the figure. If the temperature at the between the ends for the two rods be the same,
o o then in the steady
ends of this composite bar is 0 C and 100 C
state, the amount of heat flowing per second
respectively (see figure), then the temperature through them will be in the ratio
of the interface is (a) 1: 3 (b) 4 : 3
(c) 8 : 9 (d) 3 : 2
K
3K 0oC 100oC

Q KA(1 2) Q A r2
2 1cm
cm
(c) t l t l l
100 o
C ( 1 2 )
o
(a) 50 C (b) 3 [As and K are constants]
Q
200 o
o
C t r12 l2 4 2 8
(c) 60 C (d) 3 1

Q r22 l1 9 1 9

(c)Temperature of interface t 2
K l K 2 2l1 K 0 2 3K 100 1 312A slab consists of two parallel layers of two
1 12
K 1l 2 K 2l1 K 2 3K 1 different materials of same thickness having
300K thermal conductivities K1 and K2. The equivalent

5K = 60C conductivity of the combination is
309. One end of a metal rod of length 1.0 m and K1 K 2
K1 K 2 2
2 o (a) (b)
area of cross section 100cm is maintained at 100 C.
o
2K1K 2 K1 K 2
If the other end of the rod is maintained at 0 C , K1 K 2 2K1K 2
(c) (d)
the quantity of heat transmitted through the rod
(b)In parallel combination equivalent conductivity
per minute is (Coefficient of thermal conductivity
of material of rod =100W/m-K) K 1 A1 K 2 A2 K 1 K 2
K
A1 A2 2 A1 A2
3
(a) 3 10 J
3
(b) 6 10 J (As )
3 3 313 The heat is flowing through a rod of length 50
(c) 9 10 J (d) 12 10 J 2
cm and area of cross-section 5cm . Its ends are
4
Q KA(1 2) 100 100 10 (100 0) o o
respectively at 25 C and 125 C . The coefficient of
(b) t l 1
thermal conductivity of the material of the rod is
Q 0.092 kcal/msoC. The temperature gradient in
100Joule/ sec 6 103 Joule/ min
t the rod is
310.The coefficient of thermal conductivity of o o
copper is nine times that of steel. In the composite (a) 2 C / cm (b) 2 C / m
SRI MARUTHI P.U COLLEGE, HOSKOTE I PUC PART-2 CET/NEET STUDY
MATERIAL
Waves/Sound, Oscillations/SHM, Mechanical Properties of Matter/Elasticity, Fluid Mechanics (Fluid
Statics, Fluid Dynamics, Viscosity, Surface Tension),
Thermal Properties Matter/Thermometry (Thermal Expansion, Calorimetry, Transmission of Heat),
Thermo Dynamics, Kinetic Theory of Gases (KTG)
20o C / cm 20o C / m 3( 1 ) ( 2 )
(c) (d)
d (125 25)o C 3 1 3 2 4 1 4 1 2
2C / cm
dx 50 cm
(a)Temperature gradient 4( 1 ) ( 1 2 )

314 Two rods of same length and cross section are
joined along the length. Thermal conductivities of 4( 1 ) 20 ( 1 ) 5C

K andK 317 A metal rod of length 2m has cross sectional
first and second rod are 1 2 . The temperature

of the free ends of the first and second rods are areas 2A and A as shown in figure. The ends are
maintained at temperatures 100C and 70C. The
1 and 2
maintained at respectively. The temperature at middle point C is
temperature of the common junction is
1 2 K 2K 2 100C (a) 80C
( 1 2 ) C
70C
K1 K 2
(a) 2 (b) (b) 85C
2A A
K 1 1 K 2 2 K 2 1 K 1 2 (c) 90C
1m (c) K1 K 2 K1 K 2 (d) 95C
1m (d)
(c)At steady state, rate of heat flow for both blocks
K 1 A( 1 ) K 2 A( 2 ) (c)Let be temperature middle point C and in series rate of

l1 l2 l1 l 2 heat flow is same
K (2A)(100 ) KA( 70)
will be same i.e., (given )

K 1 1 K 2 2 200 2 70 3 270 90C
K 1 A( 1 ) K 2 A( 2 ) K1 K 2
1 2 318.Two metal cubes A and B of same size are
arranged as shown in the figure. The extreme ends
of the combination are maintained at the indicated
K1 K2 temperatures. The arrangement is thermally
insulated. The coefficients of thermal conductivity
l1 l2 300W / m o C 200W / m o C
of A and B are and ,
respectively. After steady state is reached, the
315 Consider a compound slab consisting of two temperature of the interface will be
different materials having equal thickness and o
(a) 45 C
thermal conductivities K and 2K respectively. The
equivalent thermal conductivity of the slab is 100oC A oC
90 B 0oC
(b)
(a) 2K
o
(b) 3K (c) 30 C
4 2 o
K K (d) 60 C
(c) 3 (d) 3
2K 1 K 2 2.K .2K 4 K11 K 2 2
K K T
K 1 K 2 K 2K 3 K1 K 2
(c) (d)Temperature of interface
316 A wall has two layers A and B made of 300 100 200 0
60C
different materials. The thickness of both the 300 200
layers is the same. The thermal conductivity of A 319 The ratio of the coefficient of thermal
and B are KA and KB such that KA = 3KB. The conductivity of two different materials is 5 : 3. If
temperature across the wall is 20C. In thermal
the thermal resistance of the rod of same
equilibrium
thickness resistance of the rods of same thickness
(a) The temperature difference across A 15C of these materials is same, then the ratio of the
(b) The temperature difference across A 5C length of these rods will be
(c) The temperature difference across A is (a) 3 : 5 (b) 5 : 3
10C (c) 3 : 4 (d) 3 : 2
(d) The rate of transfer of heat through A is (b)Thermal resistances are same
more than that through B. l1 l l1 l
(b)In series rate of flow of heat is same
1 2 2 2
K 1 A1 K 2 A2 K 1 K 2 ( A1 A2 )

K A A( 1 ) K B A( 2 )
A B l1 K 1 5
l l
l2 K 2 3

3K B ( 1 ) K B ( 2) KA KB
320 If wavelengths of maximum intensity of
radiations emitted by the sun and the moon are
SRI MARUTHI P.U COLLEGE, HOSKOTE I PUC PART-2 CET/NEET STUDY
MATERIAL
Waves/Sound, Oscillations/SHM, Mechanical Properties of Matter/Elasticity, Fluid Mechanics (Fluid
Statics, Fluid Dynamics, Viscosity, Surface Tension),
Thermal Properties Matter/Thermometry (Thermal Expansion, Calorimetry, Transmission of Heat),
Thermo Dynamics, Kinetic Theory of Gases (KTG)
0.5 106 m and 104 m respectively, the ratio of their 325The rate of radiation of a black body at 0C is
EJ/sec. The rate of radiation of this black body at
temperatures is
273o C will be
(a) 1/100 (b) 1/200 16 E 8E
(c) 100 (d) 200 (a) (b)
4E
T1 2
104 (c) (d) E
mT T2 1 5
(d) =constant 0.5 10 =200. E1 T1
4
E 273 0
4

321 The wavelength of maximum emitted energy E2 T2 E2 273 273 E 2 16 E
(a) .
of a body at 700 K is 4.08 m . If the temperature o
of the body is raised to 1400 K, the wavelength of 326 An object is at a temperature of 400 C . At
maximum emitted energy will be what temperature would it radiate energy twice as
fast? The temperature of the surroundings may be
1.02 m
(a) (b) 16.32 m assumed to be negligible
(c) 8.16 m (d) 2.04 m o
(a) 200 C (b)
200K
m T1 o
m1 T1 m2 T2 m2 1
4.08
700
2.04m (c) 800 C (d) 800 K
T2 1400
(d) 4 4 4
E2 T2 2 420 273 673
322 Four pieces of iron heated in a furnace to
E1 T1 1 T T
different temperatures show different colours (d)
listed below. Which one has the highest 1/ 4
T 2 673 = 800K.
temperature
327 If temperature of a black body increases from
(a) White (b) Yellow
(c) Orange (d) Red 7o C to 287o C , then the rate of energy radiation
increases by
4
287
(a) At low temperature short wavelength radiation
(a) 7 (b) 16
is emitted. As the temperature rise colour of
emitted radiation are in the following order (c) 4 (d) 2
RedYellowBlueWhite (at highest temperature Q
P AT 4
323 The wavelength of maximum energy released (b)For a block body rate of energy t
10
during an atomic explosion was 2.93 10 m . Given
4 4
P1 T1 (273 7) 1

3
that Wein's constant is 2.93 10 m K , the maximum 4 P2 T2 (273 287) 16
PT
temperature attained must be of the order of 328 The temperature at which a black body of unit
7
(a) 10 K
7
(b) 10 K area loses its energy at the rate of 1 joule/second
is
13 7
(c) 10 K (d) 5.86 10 K o o
(a) 65 C (b) 65 C
b 2.93 103
T 107 K (c) 65 K (d) None of these
m 2.93 1010
(b) 4 8
(c) E T 5.6 10 T 1
4

200cm2
324 A metal ball of surface area and 1
1/ 4
T 8
o
5.6 10
temperature 527 C is surrounded by a vessel at =65 K
o 4 2
27 C . If the emissivity of the metal is 0.4, then the 329 The area of a hole of heat furnace is 10 m . It
rate of loss of heat from the ball is 5
8 2 4
radiates 1.58 10 calories of heat per hour. If the
( 5.67 10 J / m s k )
emissivity of the furnace is 0.80, then its
(a) 108 joules approx. (b)168 joules approx. temperature is
(c) 182 joules approx. (d)192 joules approx. (a) 1500 K (b) 2000 K
4
E eA(T T04 ) (c) 2500 K (d) 3000 K
(c)Rate of heat loss
4 4
5.67 108 0.4 200 104 [(273 527) (273 27) ] E AT 4
(c)According to Stefens law
4 4 1.58 105 4.2
5.67 108 0.4 200 104 (800) (300) =182J/sec 5.6 108 10 4 0.8 T 4
60 60
T 2500K
SRI MARUTHI P.U COLLEGE, HOSKOTE I PUC PART-2 CET/NEET STUDY
MATERIAL
Waves/Sound, Oscillations/SHM, Mechanical Properties of Matter/Elasticity, Fluid Mechanics (Fluid
Statics, Fluid Dynamics, Viscosity, Surface Tension),
Thermal Properties Matter/Thermometry (Thermal Expansion, Calorimetry, Transmission of Heat),
Thermo Dynamics, Kinetic Theory of Gases (KTG)
330 Two spheres P and Q , of same colour having
2 4
440 12 500

P 1760W 1800W
8 cm 2 cm P2 6 1000 2
radii and are maintained at temperatures
r1 r2
o o
127 C and 527 C respectively. The ratio of energy 334 Two spherical black bodies of radii and
T T
radiated by P and Q is and with surface temperature 1 and 2

(a) 0.054 (b) 0.0034 respectively radiate the same power. Then the
(c) 1 (d) 2 r1 r2
ratio of and will be
4
(c)Total energy radiated from a body Q AT t
2 4
T2 T2

T T
4 2 4 2 1 1
Q AT r T ( A 4r ) (a) (b)
2 4 2 4 2 4
QP rP TP
8 (273 127) T1

T1
1
QQ rQ T T T
Q 2 (273 527) (c) 2 (d) 2

331 A thin square steel plate with each side equal 1


4 A
to 10 cm is heated by a blacksmith. The rate of (a)For black body, P AT . For same power T4
radiated energy by the heated plate is 1134 W. 2 4 2
The temperature of the hot steel plate is (Stefan's r1 T2 r1 T2

r r2 T1
2 T1
5.67 108 wattm2 K 4
constant , emissivity of the
335 Two black metallic spheres of radius 4m, at
plate = 1)
2000 K and 1m at 4000 K will have ratio of
1000K 1189K energy radiation as
(a) (b)
2000K 2378K (a) 1 : 1 (b) 4 : 1
(c) (d)
(c) 1 : 4 (d) 2 : 1
Q
P AT 4 Q1 r12T14 4 2 2000
4

(b)The rate of radiated energy t 1


Q2 r22T24 12 4000
(a)
1134 5.67 108 (0.1)2 T 4 T 1189K
336 The energy spectrum of a black body exhibits
332 The energy emitted per second by a black o .
a maximum around a wavelength The
o 10 J
body at 27 C is . If the temperature of the temperature of the black body is now changed
327o C , the energy such that the energy is maximum around a
black body is increased to
emitted per second will be 3 o

20 J 40 J wavelength 4 .The power radiated by the black


(a) (b) body will now increase by a factor of
80 J 160J (a) 256/81 (b) 64/27
(c) (d)
(c) 16/9 (d) 4/3
(Q)Blackbody AT 4t 4
(d) QT (a)According to Wein's law mT = constant
4 4 4
T2 273 327 600 m1 0 4
Q2 Q1
10 10 160J m1 T1 m2 T2 T2 T1 T1 T1
T1 273 27 300 m2 30 / 4 3

12cm
333 A spherical black body with a radius of P2 T2
4
P2 4 / 3 T1
4
256
P T4

440W 500K P1 T1 P1 T1 81
radiates power at . If the radius were Now
halved and the temperature doubled, the power o
337 Two identical metal balls at temperature 200 C
radiated in watt would be
o o
(a) 225 (b) 450 and 400 C kept in air at 27 C . The ratio of net heat
(c) 900 (d) 180 loss by these bodies is
Q (a) 1/4 (b) 1/2
P AT 4
(d)Radiated power by blackbody t 4734 3004
4 4
P1 r1
2
T1
4
(c) 1/16 (d) 673 300

4 2 4 P r T (d)If temperature of surrounding is considered
P AT r T 2 2 2
then
SRI MARUTHI P.U COLLEGE, HOSKOTE I PUC PART-2 CET/NEET STUDY
MATERIAL
Waves/Sound, Oscillations/SHM, Mechanical Properties of Matter/Elasticity, Fluid Mechanics (Fluid
Statics, Fluid Dynamics, Viscosity, Surface Tension),
Thermal Properties Matter/Thermometry (Thermal Expansion, Calorimetry, Transmission of Heat),
Thermo Dynamics, Kinetic Theory of Gases (KTG)
net loss of energy of a body by radiation 342 Rate of cooling at 600K, if surrounding
Q1 T14 T04 temperature is 300K is R. The rate of cooling at

Q A (T 4
T04 )t Q (T 4
T04 ) Q2 T24 T04 900K is

16
(273 200)4 (273 27)4 (473)4 (300)4 R
(a) 3 (b) 2R
(273 400)4 (273 27)4 (673)4 (300)4
2
338 Two spheres of same material have radius 1m R
and 4 m and temperature 4000K and 2000K (c) 3R (d) 3
respectively. The energy radiated per second by R1 (T14 T04 )
the first sphere is 4
(T T04 ) R2 (T24 T04 )
(a)Rate of cooling
(a) Greater than that by the second
4 4
(b) Less than that by the second R (600) (300) 16
R2 R
(c) Equal in both cases R2 (900)4 (300)4 3
or
(d) The information is incomplete 343 A black body of surface area 10cm2 is heated
Q to 127C and is suspended in a room at
P AT 4
(c)Energy radiated per sec t temperature 27C. The initial rate of loss of heat
4 from the body at the room temperature will be
P2 r22 T24 42 2000
P r 2T 4 . 1 (a) 2.99 W (b) 1.89 W
P1 r12 T14 12 4000
(c) 1.18 W (d) 0.99 W
339 The radiation emitted by a star A is 10,000
times that of the sun. If the surface temperatures of Q A (T 4 T04 )t
d)Loss of heat
the sun and the star A are 6000 K and 2000 K
respectively, the ratio of the radii of the star A and Q
A (T 4 T04 )
the sun is Rate of loss of heat t
(a) 300 : 1 (b) 600 : 1 10 104 1 5.67 108{273 127)4 (273 27)4}
(c) 900 : 1 (d) 1200 : 1 = 0.99 W.
Qstar r 2star.T 4star o o
4 2 4
344 Hot water cools from 60 C to 50 C in the first
Qsun r 2sun Tsun4
(c) Q AT r T o
4
10 minutes and to 42 C in the next 10 minutes.
2
10000 rstar 6000 rstar 100 9 900 The temperature of the surrounding is
2

1 rsun 2000 rsun 1 1 o o
(a) 5 C (b) 10 C
340 If the suns surface radiates heat at
o o
6.3 107Wm2 . Calculate the temperature of the sun (c) 15 C (d) 20 C
8 2 4 (b)According to Newton's law of cooling
assuming it to be a black body ( 5.7 10 Wm K ) 1 2
3 3
K 1 2 0
(a) 5.8 10 K (b) 8.5 10 K t 2
8 8 (60 50) 60 50
(c) 3.5 10 K (d) 5.3 10 K K 0
10 2
4 In the first case,
(a)From Stefans law E T
1 K (55 )
.(i)
E 6.3 107
T4 1.105 1015 0.1105 1016 (50 42) 50 42
5.7 108 K 0
10 2
T 0.58 104 K 5.8 103 K In the second case,
341 A sphere at temperature 600K is placed in an 0.8 K (46 0)
.(ii)
environment of temperature is 200K. Its cooling
1 55 0
rate is H. If its temperature reduced to 400K then
0.8 46 0
cooling rate in same environment will become Dividing (i) by (ii), we get
(a) (3/16)H (b) (16/3)H 46 0 44 0.8 0 0 10o C
or
(c) (9/27)H (d) (1/16)H o
4 345 A bucket full of hot water cools from 75 C to
(T T04 )
(a)Rate of cooling 700 C in time T1 , from 70o C to 65o C in time T2 and
H (T14 T04 ) 4004 2004
o o T
H ' (T24 T04 ) 6004 2004 from 65 C to 60 C in time 3 , then

T1 T2 T3 T1 T2 T3
(16 4)(16 4)H 3
(a) (b)
H' H
or (36 4)(36 4) 16 T1 T2 T3 T1 T2 T3
(c) (d)
SRI MARUTHI P.U COLLEGE, HOSKOTE I PUC PART-2 CET/NEET STUDY
MATERIAL
Waves/Sound, Oscillations/SHM, Mechanical Properties of Matter/Elasticity, Fluid Mechanics (Fluid
Statics, Fluid Dynamics, Viscosity, Surface Tension),
Thermal Properties Matter/Thermometry (Thermal Expansion, Calorimetry, Transmission of Heat),
Thermo Dynamics, Kinetic Theory of Gases (KTG)
(c)According to Newton's law of cooling 348 A body takes 4 minutes to cool from to
o
Rate of cooling Mean temperature difference 100 C
. To cool from to it will take (room
e 1 2
Fall in temperatur
0 70o C 70o C 40o C
Time 2
temperature is )
1 2 1 2 1 2 15o C

2 1 2 2 2 3 (a) 7 minutes (b) 6 minutes
(c) 5 minutes (d) 4 minutes
T1 T2 T3
(b)
B1 B 1 2
346 Consider two hot bodies and 2 which have K 1 2 0
t 2
temperatures and respectively at .
t0 = 60K K =
100o C 80o C 100 70 100 70 1
The temperature of the surroundings is . The K 15
4 2 8
40o C
ratio of the respective rates of cooling and Again = 5 t = 6 min.
R1 R2 70 40 1 70 40
15
t 8 2
of these two bodies at will be
t0 349 A cup of tea cools from to in one
800 C 60o C
(a) (b)
R1 : R2 3 : 2 R1 : R2 5 : 4 minute. The ambient temperature is . In
30o C
(c) (d) cooling from to it will take
R1 : R2 2 : 3 R1 : R2 4 : 5
60o C 50o C
(a)Initially at t = 0 (a) (b)
Rate of cooling (R) Fall in temperature of body ( 30 seconds 60 seconds
0) (c) (d)
90 seconds 50 seconds
R1 1 0 100 40 3

R2 2 0 80 40 2
(d)
347 Liquid is filled in a vessel which is kept in a 80 60 80 60 1
K 30 K
room with temperature . When the 1 2 2
20o C
Again sec.
temperature of the liquid is , then it loses 60 50 1 60 50 t 0.8 60 48
30
80o C t 2 2
heat at the rate of . What will be the rate 350 If a metallic sphere gets cooled from to
60cal/ sec
62o C
of loss of heat when the temperature of the liquid in and in the next gets
is 50o C 10 minutes 10 minutes
40o C cooled to , then the temperature of the
(a) (b) 42o C
180cal/ sec 40cal/ sec surroundings is
(c) (d) (a) (b)
30cal/ sec 20cal/ sec 30o C 36o C

(d)Rate of loss of heat temperature (c) (d)


Q 26o C 20o C

t (c)
difference 1 2
K 1 2 0
t 2

Q 60

80 60 Q 20cal In the first 10 minute

t 2 Q 40 20 t sec .... (i)
1
2
Q 1 t 2 62 50 62 50 1.2 K [56 0]
K 0
t 2 10 2
.... (ii)
50 42 50 42 0.8 K [46 0]
K 0
10 2
SRI MARUTHI P.U COLLEGE, HOSKOTE I PUC PART-2 CET/NEET STUDY
MATERIAL
Waves/Sound, Oscillations/SHM, Mechanical Properties of Matter/Elasticity, Fluid Mechanics (Fluid
Statics, Fluid Dynamics, Viscosity, Surface Tension),
Thermal Properties Matter/Thermometry (Thermal Expansion, Calorimetry, Transmission of Heat),
Thermo Dynamics, Kinetic Theory of Gases (KTG)
from equations (i) and (ii) In second case .(ii)
1.2 (56 0) 0 26C 40 2 40 2
K 20
0.8 (46 0) 10 2
351 A body cools from to in 10 minutes. By solving .
60o C 50o C 2 33.3o C
If the room temperature is and assuming 353.Two rods (one semi-circular and other straight)
25o C of same material and of same cross-sectional area
Newton's law of cooling to hold good, the are joined as shown in the figure. The points A and
temperature of the body at the end of the next 10 B are maintained at different temperature. The
minutes will be ratio of the heat transferred through a cross-
(a) (b) section of a semi-circular rod to the heat
38.5o C 40o C transferred through a cross section of the straight
(c) (d) rod in a given time is
42.85o C 45o C (a) 2 :
(c) ..(i) (b) 1 : 2
60 50 60 50
K 25 (c) : 2
10 2
(d) 3 : 2 A Straight rod B
..(ii)
50 50
K 25
10 2
On dividing, we get (a) , For both rods K, A and are same
10 60 dQ KA
42.85o C
50 dt l
352 In a room where the temperature is , a So .
30o C dQ 1 (dQ / dt)semicircular lstraight 2r 2

body cools from to in 4 minutes. The dt l (dQ / dt)straight lsemicircular r
61o C 59o C
time (in min.) taken by the body to cool from 7.Thermo Dynamics
0
51 C 354. Heat given to a system is 35 joules and work
to will be done by the system is 15 joules. The change in the
490 C internal energy of the system will be
(a) 4 min (b) 6 min (a) 50 J (b) 20 J
(c) 5 min (d) 8 min (c) 30 J (d) 50 J
(b)First case, ..(i) (b)
61 59 61 59 U Q W 35 15 20 J
K 30
4 2
355. If the amount of heat given to a system be 35
Second case, ..(ii) joules and the amount of work done by the
51 49 51 49
K 30
t 2 system be joules, then the change in the
15
By solving equation (i) and (ii) we get t = 6 min.
internal energy of the system is
400 The temperature of a body falls from to
50o C (a) joules (b) 20 joules
50
in 10 minutes. If the temperature of the
(c) 30 joules (d) 50 joules
40o C
(d)
surroundings is Then temperature of the body Q W U 35 15 U U 50J
20o C
356. A system is given 300 calories of heat and it
after another 10 minutes will be does 600 joules of work. How much does the
(a) (b) internal energy of the system change in this
36.6o C 33.3o C process
(c) (d) (J = 4.18 joules/cal)
35o C 30o C (a) 654 Joule (b) 156.5 Joule
(b)In first case .(i) (c) 300 Joule (d) 528.2 Joule
50 40 50 40 (a)
K 20
10 2 J Q U W, U J Q W
SRI MARUTHI P.U COLLEGE, HOSKOTE I PUC PART-2 CET/NEET STUDY
MATERIAL
Waves/Sound, Oscillations/SHM, Mechanical Properties of Matter/Elasticity, Fluid Mechanics (Fluid
Statics, Fluid Dynamics, Viscosity, Surface Tension),
Thermal Properties Matter/Thermometry (Thermal Expansion, Calorimetry, Transmission of Heat),
Thermo Dynamics, Kinetic Theory of Gases (KTG)
U 4.18 300 600 654 Joule 362. In a thermodynamic process, pressure of a
fixed mass of a gas is changed in such a manner
357. If R = universal gas constant, the amount of
that the gas molecules gives out 20 J of heat and
heat needed to raise the temperature of 2 mole of
10 J of work is done on the gas. If the initial
an ideal monoatomic gas from 273K to 373K when internal energy of the gas was 40 J, then the final
no work is done internal energy will be
(a) 100 R (b) 150 R (a) 30 J (b) 20 J
(c) 300 R (d) 500 R (c) 60 J (d) 40 J
(c) (c)
Q U W W 0 Q U f Q U W (U f Ui ) W
RT
2

= 300R. 30 (U f 40) 10 U f 60J
3
2R(373 273)
2 363. One mole of an ideal monoatomic gas is
358. Find the change in internal energy of the heated at a constant pressure of one atmosphere
system when a system absorbs 2 kilocalorie of from to . Then the change in the internal
0o C 100o C
heat and at the same time does 500 joule of work
energy is
(a) 7900 J (b) 8200 J (a) 6.56 joules (b) joules
8.32 102
(c) 5600 J (d) 6400 J
(a) and (c) joules (d) 20.80 joules
2
12.48 10
Q 2k cal 2 103 4.2J 8400J W 500J .
(c)Change in internal energy is always equal to the
Hence from = 8400 500
Q U W, W Q U
heat supplied at constant volume.
i.e.
= 7900 J U (Q)V CV T.
359. When the amount of work done is 333 cal and
change in internal energy is 167 cal, then the heat For monoatomic gas
3
supplied is CV R
2
(a) 166 cal (b) 333 cal
(c) 500 cal (d) 400 cal U
3 3
R T 1 8.31 (100 0)
2 2
(c)
Q U W 167 333 500cal
12.48 102 J
360. In thermodynamic process, 200 Joules of heat 364. If the ratio of specific heat of a gas at
is given to a gas and 100 Joules of work is also constant pressure to that at constant volume is ,

done on it. The change in internal energy of the
gas is the change in internal energy of a mass of gas,
when the volume changes from V to 2V constant
(a) 100 J (b) 300 J
pressure p, is
(c) 419 J (d) 24 J
(a) (b)
(b) ; and R /( 1) pV
Q U W Q 200J W 100J
(c) (d)
pV /( 1) pV /( 1)
U Q W 200 (100) 300J
(c)
361. A system is provided with 200 cal of heat and R
the work done by the system on the surrounding is U CV T n T
1
40 J. Then its internal energy
PV P (2V V) PV
(a) Increases by 600 J (b)Decreases by 800 J U
( 1) 1 ( 1)
(c) Increases by 800 J (d)Decreases by 50 J
365. In a thermodynamics process, pressure of a
(c)
Q U W fixed mass of a gas is changed in such a manner
that the gas releases 20 J of heat and 8J of work is
and
Q 200cal 200 4.2 840J W 40J done on the gas. If the initial internal energy of the
gas was 30J. The final internal energy will be

U Q W 840 40 800J (a) 18J (b) 9J


SRI MARUTHI P.U COLLEGE, HOSKOTE I PUC PART-2 CET/NEET STUDY
MATERIAL
Waves/Sound, Oscillations/SHM, Mechanical Properties of Matter/Elasticity, Fluid Mechanics (Fluid
Statics, Fluid Dynamics, Viscosity, Surface Tension),
Thermal Properties Matter/Thermometry (Thermal Expansion, Calorimetry, Transmission of Heat),
Thermo Dynamics, Kinetic Theory of Gases (KTG)
(c) 4.5J (d) 36J
(a)Given and 1.5 104
Q 1.5 104 J cal 3.6 103 cal
Q 20J , W 8J U i 30J 4.18

(Q369)540 calories of heat convert 1 cubic


Q U W U (Q W) centimeter of water at into 1671 cubic
= 100o C
(U f Ui ) (U f 30) 20 (8) U f 18J centimeter of steam at at a pressure of one
366.In an isothermal reversible expansion, if the 100o C
volume of 96 gm of oxygen at 27C is increased atmosphere. Then the work done against the
from 70 litres to 140 litres, then the work done by atmospheric pressure is nearly
the gas will be (a) 540 cal (b) 40 cal
(a) (b) (c) Zero cal (d) 500 cal
300R log10 2 81R loge 2
(b)Amount of heat given
(c) (d) 540calories
900R log10 2 2.3 900R log10 2
Change in volume
(d) V 1670c.c
V
W RT loge 2 Atmospheric pressure
V1
P 1.01 106 dyne/ cm2

m V
RT loge 2 2.3
m V
RT log10 2
Work done against atmospheric pressure
M V1 M V1 W PV 1.01 106 1670
40 cal
96 140 4.2 107
2.3 R (273 27) log10 2.3 900R log10 2
32 70 370. One mole of an ideal gas expands at a
367. One mole of gas having a volume equal to constant temperature of 300 K from an initial
O2 volume of 10 litres to a final volume of 20 litres.
22.4 litres at and 1 atmospheric pressure in The work done in expanding the gas is
0o C (R = 8.31 J/mole-K)
compressed isothermally so that its volume (a) 750 joules (b) 1728 joules
reduces to 11.2 litres. The work done in this
(c) 1500 joules (d) 3456 joules
process is
(a) (b) 1728 J (b)
V2 20
1672.5 J Wiso RT loge 1 8.31 300loge 1728J
V1 10
(c) (d)
1728J 1572.5 J 371. The volume of an ideal gas is 1 litre and its
pressure is equal to 72cm of mercury column. The
(d) volume of gas is made 900 cm3 by compressing it
V2 22.4 isothermally. The stress of the gas will be
W RT loge 1 8.31 (273 0) loge
V1 11.2 (a) 8 cm (mercury) (b) 7 cm (mercury)
[ ] (c) 6 cm (mercury) (d) 4 cm (mercury)
8.31 273 loge 2 1572.5J loge 2 0.693
(a)For isothermal process
368. When an ideal gas in a cylinder was P1V1 P2V2
compressed isothermally by a piston, the work =80 cm
done on the gas was found to be . PV 72 1000
1.5 104 joules P2 1 1
V2 900
During this process about Stress
(a) cal of heat flowed out from the gas P P2 P1 80 72 8cm
3.6 103
372. The pressure in the tyre of a car is four times
(b) cal of heat flowed into the gas the atmospheric pressure at 300 K. If this tyre
3.6 103 suddenly bursts, its new temperature will be
(c) cal of heat flowed into the gas ( 1.4)
1.5 104
(a) (b)
(d) cal of heat flowed out from the gas 300(4)1.4 / 0.4 1
0.4 / 1.4

1.5 104 300


4

(a)In isothermal compression, there is always an (c) (d)


300(2)0.4 / 1.4 300(4)0.4 / 1.4
increase of heat. which must flow out the gas.
Q U W Q W (U 0)
SRI MARUTHI P.U COLLEGE, HOSKOTE I PUC PART-2 CET/NEET STUDY
MATERIAL
Waves/Sound, Oscillations/SHM, Mechanical Properties of Matter/Elasticity, Fluid Mechanics (Fluid
Statics, Fluid Dynamics, Viscosity, Surface Tension),
Thermal Properties Matter/Thermometry (Thermal Expansion, Calorimetry, Transmission of Heat),
Thermo Dynamics, Kinetic Theory of Gases (KTG)
(d)For adiabatic process constant 376. One gm mol of a diatomic gas is
T ( 1.4)

P 1 compressed adiabatically so that its temperature
rises from to . The work done will be
1 (1 1.4)

0.4 27o C 127o C
T2 P1 T2 4 1.4 T2 300(4) 1.4

T1 P2 300 1 (a) 2077.5 joules (b) 207.5 joules
373. A gas at NTP is suddenly compressed to one- (c) 207.5 ergs (d) None of the above
fourth of its original volume. If is supposed to (a)
R
W (T1 T2)
be , then the final pressure is 1
3
2 8.31 {(273 27) (273 127)}
2077.5 joules
1.4 1
(a) 4 atmosphere (b) atmosphere
3
2 (b)
(c) 8 atmosphere (d) atmosphere P2 V1

V

1 P2 P1 1 P0 (8)4 / 3 16P0 .
P1 V2 V2
4
(c) 377. An ideal gas is expanded adiabatically at an
3/ 2
P2 V1 P2 V1 initial temperature of 300 K so that its volume is
PV constant
8
P1 V2 1 V1 / 4 doubled. The final temperature of the hydrogen
. gas is
P2 8 atm ( 1.40)

374. The pressure and density of a diatomic gas (a) 227.36 K (b) 500.30 K
change adiabatically from (P, d) to (P', d'). (c) 454.76 K (d)
( 7 / 5)
47o C
If , then should be
d' P' (a) constant
32
d P TV 1 T2 V1 V1
T2 T1
(a) 1/128 (b) 32 T1 V2 V2

(c) 128 (d) None of the above
0.4
(c)Volume of the gas and using = 1
T2 300 227.36 K
m PV 2
V
d
constant 378. At N.T.P. one mole of diatomic gas is
We get compressed adiabatically to half of its volume
P' V

d'
. The work done on gas will be
(32)7 / 5 128 1.41
P V' d
375. An ideal gas at is compressed (a) 1280 J (b) 1610 J
o
27 C (c) 1815 J (d) 2025 J
adiabatically to of its original volume. If , (c)
8 5 1 273 1.328 363K
V1
27 3 T2 T1
273(2)0.41
V2
then the rise in temperature is
(a) 450 K (b) 375 K W
R(T1 T2) 8.31(273 363) 1824

1 1.41 1
(c) 225 K (d) 405 K
(b) |W| 1815 J
5 2
T2 V1
1
27 3
1
27 3 379. Two moles of an ideal monoatomic gas at
T2 300 300
T1 V2 8 8 occupies a volume of V. If the gas is
2
27o C
27 1/ 3
3
2
300 800 675K expanded adiabatically to the volume then the
8 2 2V,

T 675 300 375K work done by the gas will be


[ 5 / 3, R 8.31J / molK ]

(a) (b)
2767.23J 2767.23J
(c) (d)
2500J 2500J
SRI MARUTHI P.U COLLEGE, HOSKOTE I PUC PART-2 CET/NEET STUDY
MATERIAL
Waves/Sound, Oscillations/SHM, Mechanical Properties of Matter/Elasticity, Fluid Mechanics (Fluid
Statics, Fluid Dynamics, Viscosity, Surface Tension),
Thermal Properties Matter/Thermometry (Thermal Expansion, Calorimetry, Transmission of Heat),
Thermo Dynamics, Kinetic Theory of Gases (KTG)
(b) (c)

R(T1 T2) RT1 T2 P2 V1 P'
W 1 (8)5 / 2 P ' P (2)15/ 2
( 1) ( 1) T1 P1 V2 P

RT1 V
1 383. A gas is suddenly compressed to 1/4 th of its
1 1
( 1) original volume at normal temperature. The
V2
increase in its temperature is
5
1 2767.23 J ( 1.5)
2 8.31 300 1 3
1 (a) 273 K (b) 573 K
5 2
1
3 (c) 373 K (d) 473 K
380. At a gas is suddenly compressed such (a) constant
27o C TV 1 T1V1 1 T2V2 1

that its pressure becomes of original pressure.


1
1 V
th T2 T1 1 T1(4)1.51 2T1
8
V2
Temperature of the gas will be change in temperature
( 5 / 3)
T2 T1 2T1 T1 T1 273K
(a) 420K (b)
327o C
384. A gas for which is suddenly compressed
1.5
(c) (d) to th of the initial volume. Then the ratio of the
300K 142o C 1
(d) constant 4
T P 1 1
final to the initial pressure is

TP
(a) 1 : 16 (b) 1 : 8

1 5 / 31
(c) 1 : 4 (d) 8 : 1
T2 P2 1 5/ 3
(d)

T1 P1 8 P1V1 P2V2 3/ 2
P2 V1 4 8

1
0.4 P1 V2 1 1
T2 300 131K 142C
8 385. One mole of an ideal gas with , is
1.4
381. An ideal gas at a pressures of 1 atmosphere
and temperature of is compressed adiabatically compressed so that its temperature
27 C o rises from 27C to 35C. The change in the
internal energy of the gas is
adiabatically until its pressure becomes 8 times (R 8.3 J /mol.K )
the initial pressure, then the final temperature is (
(a) 166 J (b) 166 J
= 3/2)
(c) 168 J (d) 168 J
(a) (b)
(b)Change in internal energy of the gas
627o C 527o C
R
(c) (d) U W T2 T1 8.3 [308 300] 166J
1 (1.4 1)
427o C 327o C
386. Work done by air when it expands from 50
(d)Using relation . litres to 150 litres at a constant pressure of 2
1
3 / 21
T2 P2 atmosphere is
(8) 3/ 2
2
T1 P1 (a) joules (b) joules
2 104 2 100

T2 2T1 T2 2(273 27) 600K 327C (c) joules (d) joules
382. If and volume is equal to times to 2 105 100 2 105 100
2.5 1 (a)
8
W P V 2 105 (150 50) 103 2 104 J
the initial volume then pressure P' is equal to
387. Work done by 0.1 mole of a gas at to
(Initial pressure = P) o
27 C
(a) (b)
P' P P ' 2P
double its volume at constant pressure is (R = 2
cal mol1 oC1)
(c) (d)
P ' P (2)15/ 2 P ' 7P (a) 54 cal (b) 600 cal
(c) 60 cal (d) 546 cal
SRI MARUTHI P.U COLLEGE, HOSKOTE I PUC PART-2 CET/NEET STUDY
MATERIAL
Waves/Sound, Oscillations/SHM, Mechanical Properties of Matter/Elasticity, Fluid Mechanics (Fluid
Statics, Fluid Dynamics, Viscosity, Surface Tension),
Thermal Properties Matter/Thermometry (Thermal Expansion, Calorimetry, Transmission of Heat),
Thermo Dynamics, Kinetic Theory of Gases (KTG)
(c) T2 273 273
W PV nRT 0.1 2 300 60 cal K 9
T1 T2 303 273 30
388. Two kg of water is converted into steam by 393. An ideal gas heat engine operates in a
boiling at atmospheric pressure. The volume
Carnot's cycle between and . It absorbs
changes from to The work done by
227o C 127o C
2 103 m3 3.34m3 .
6 104 J at high temperature. The amount of heat
the system is about
converted into work is ....
(a) 340 kJ (b) 170 kJ
(a) (b)
(c) 170 kJ (d) 340 kJ
4.8 104 J 3.5 104 J
(d)
W PV 1.01 105 (3.34 2 103 ) (c) (d)
1.6 104 J 1.2 104 J
337 103 J 340KJ
(d)
389. An ideal gas has volume at It is T 400 1 W 1 W
V0 1 2 1
27o C. T1 500 5 Q 5 Q
heated at constant pressure so that its volume

becomes The final temperature is Q 6
2V0 . W 104 1.2 104 J
5 5
(a) (b) 394. A Carnot's engine used first an ideal
54o C 32.6o C
monoatomic gas then an ideal diatomic gas. If the
(c) (d) 150 K
327C source and sink temperature are and
o
411 C 69o C
(c)
T2 V2 respectively and the engine extracts 1000 J of heat
2 T2 2 T1 2 300 600K 327o C
T1 V1 in each cycle, then area enclosed by the PV
diagram is
390.In an isochoric process if and
T1 27o C (a) 100 J (b) 300 J
then will be equal to (c) 500 J (d) 700 J
o
T2 127 C, P1 / P2

(a) 9 / 59 (b) 2 / 3 (c)


T2 (273 69)
(c) 3 / 4 (d) None of these 1 1 0.5
T1 (273 411)
(d)At constant volume
PT P1 T1 P1 300 3 Work done
Q 0.5 1000 500J
P2 T2 P2 400 4

391. A Carnot engine working between and 395. In a Carnot engine, when and
300K T2 0o C
its efficiency is and when and
600K has work output of 800 J per cycle. What is 1
T1 200o C, T1 0 o C
amount of heat energy supplied to the engine
, Its efficiency is , then what is
from source per cycle T2 200 C o 2 1 / 2
(a) 1800 J/cycle (b) 1000 J/cycle (a) 0.577 (b) 0.733
(c) 2000 J/cycle (d) 1600 J/cycle (c) 0.638 (d) Can not be calculated
(d) (a)
T T2 W T1 T2 T1 T2 (473 273) 200
1 Q W 1 1
T1 Q
T1 T2 T1 T1 473 473

=1600 J and
600 273 73 200
800 2
(600 300) 273 273

392. The coefficient of performance of a Carnot So required ratio


1 273
refrigerator working between and is 0.577
2 473
30o C 0o C
396. An ideal refrigerator has a freezer at a
(a) 10 (b) 1
temperature of The coefficient of
(c) 9 (d) 0 13C.
(c)Coefficient of performance performance of the engine is 5. The temperature
of the air (to which heat is rejected) will be
SRI MARUTHI P.U COLLEGE, HOSKOTE I PUC PART-2 CET/NEET STUDY
MATERIAL
Waves/Sound, Oscillations/SHM, Mechanical Properties of Matter/Elasticity, Fluid Mechanics (Fluid
Statics, Fluid Dynamics, Viscosity, Surface Tension),
Thermal Properties Matter/Thermometry (Thermal Expansion, Calorimetry, Transmission of Heat),
Thermo Dynamics, Kinetic Theory of Gases (KTG)
(a) 325C (b) 325K pressure on the hydrogen gas is (Density of
(c) 39C (d) 320C hydrogen gas is
(c)Coefficient of performance , 1 atmosphere )
8.99 102 kg / m3 1.01 105 N / m2
T2 (273 13) 260
K 5 (a)0.1 atm(b)1.5 atm(c)2.0 atm(d)3.0 atm
T1 T2 T1 (273 13) T1 260
Solution : (d)

5T1 1300 260 5T1 1560 1 2 1
P vrms (8.99 10 2 ) (3180)2 3.03 105 N/m2 3.0 atm
3 3

T1 312K 39C 401. The temperature of a gas is raised while its


397. An ideal gas heat engine operates in Carnot volume remains
cycle between 227C and 127C. It absorbs constant, the pressure exerted by a gas on the
6 104 walls of the container
cals of heat at higher temperature. Amount of heat increases because its molecules
converted to work is (a)Lose more kinetic energy to the wall
(a) cal (b) cal
(b)Are in contact with the wall for a shorter time
2.4 104 6 104
(c) cal (d) cal (c)Strike the wall more often with higher velocities
1.2 104 4.8 104 (d)Collide with each other less frequency
Solution : (c)Due to increase in temperature root
(c) mean square
T T2 W Q(T1 T2)
1 W velocity of gas molecules increases. So they strike
T1 Q T1
the wall more
6 104 (227 273) (273 127) often with higher velocity. Hence the pressure

(227 273)
exerted by a gas on
4
6 104 100 1.2 10 cal the walls of the container increases.

500
402. A cylinder of capacity 20 litres is filled with
8.Kinetic Theory of Gases gas. The total
398. The temperature of a gas at pressure P and H2
volume V is 27C. Keeping its volume constant if
its temperature is raised to 927C, then its average kinetic energy of translatory motion of its
pressure will be molecules is
(a) 2 P (b) 3 P . The pressure of hydrogen in the cylinder
(c) 4 P (d) 6P 1.5 105 J

(c)Using Charles law is


P1 T1 (a)

P2 T2
2 106 N / m2
or = 4P. (b)
PT P(273 927) 3 106 N / m2
P2 1 2
T1 (273 27)
(c)
399. moles of an ideal gas is at 0C. At constant 4 106 N / m2
pressure it is heated to double its volume, then its
final temperature will be (d)
(a) 0C (b) 273C 5 106 N / m2
(c) 546C (d) 136.5C Solution : (d)Kinetic energy E = ,
(b) 1.5 105 J
V1 T1 2 (273 0) 546K
T2 2 T1 volume V = 20 litre =
V2 T2
20 103 m3
273C Pressure .
T2 273 2 546K 273C 2E
2 1.5 105
5 106 N/m2
400. The root mean square speed of hydrogen
3V 3 20 10 3
molecules of an ideal
403. A flask contains gas. At a
hydrogen gas kept in a gas chamber at 0C is 103 m3
3180 m/s. The
temperature, the number of
SRI MARUTHI P.U COLLEGE, HOSKOTE I PUC PART-2 CET/NEET STUDY
MATERIAL
Waves/Sound, Oscillations/SHM, Mechanical Properties of Matter/Elasticity, Fluid Mechanics (Fluid
Statics, Fluid Dynamics, Viscosity, Surface Tension),
Thermal Properties Matter/Thermometry (Thermal Expansion, Calorimetry, Transmission of Heat),
Thermo Dynamics, Kinetic Theory of Gases (KTG)
molecules of oxygen are . The mass of an released to maintain the temperature of the gas in
3.0 10 22 the flask at 52C
oxygen and the pressure remaining the same is
molecule is kg and at that temperature [EAMCET (Engg.) 2000]
5.3 10 26 (a)2.5 g(b)2.0 g(c)1.5 g(d)1.0g
the rms velocity of molecules is 400 m/s. The Solution : (d)PV Mass of gas Temperature
pressure in of the gas in the flask is In this problem pressure and volume remains
N / m2 constant so
(a) (b) = M2T2 = constant
8.48 10 4
2.87 104 M 1T1

(c) (d)
M2 T (27 273) 300 12
25.44 104 12.72 104 1
M 1 T2 (52 273) 325 13
Solution : (a) , ,
3 3 22 12 12
V 10 m N 3.0 10 M 2 M1 13 gm 12gm
, 13 13
m 5.3 1026kg vrms 400m/s 407. If the intermolecular forces vanish away, the
volume occupied
.
by the molecules contained in 4.5 kg water at
1 mN 2 1 5.3 1026 3.0 1022
P vrms (400)2 8.48 104 N/m2 standard temperature
3 V 3 10 3
and pressure will be given by
404. A gas at 27C has a volume V and pressure P. (a) (b) (c)11.2 litre(d)
On heating its 5.6 m3 4.5 m3 11.2 m3
pressure is doubled and volume becomes three
Solution(a)
times. The resulting
,
temperature of the gas will be Massof water 4.5 kg
(a)1800C(b)162C(c)1527C(d)600C 250
Molecularwt. of water 18 10 3 kg
Solution:(c)From ideal gas equation
PV RT T = 273 K and (STP)
5 2
P 10 N/m
we get
T2 P2 V2 2P1 3V1 From
6 PV RT
T1 P1 V1 P1 V1
.
RT 250 8.3 273
T2 6T1 6 300 1800K 1527C. V 5
5.66m3
P 10
405. A balloon contains of helium at 27C 408. If the value of molar gas constant is 8.3
500m3
J/mole-K, the n specific
and 1 atmosphere gas constant for hydrogen in J/mole-K will be
pressure. The volume of the helium at 3C (a)4.15(b)8.3(c)16.6(d)None of these
temperature and 0.5 Solution : (a)
atmosphere pressure will be Specific gas constant
(a) (b) (c) (d) Joule/mole-K.
500m3 700m3 900m3 1000m3 Universalgasconstant(R) 8.3
r 4.15
Solution:(c)From Molecularweightof gas(M ) 2
PV RT 409. Two identical glass bulbs are interconnected
we get by a thin glass
V2 T2 P1 270 1 9 tube. A gas is filled in these bulbs at N.T.P. If one

V1 T1 P2 300 0.5 5 bulb is placed in
ice and another bulb is placed in hot bath, then
the pressure of the
9
V2 500 900m3 gas becomes 1.5 times. The temperature of hot
5 bath will be
406. A flask is filled with 13 gm of an ideal gas at
27C and its (a)100C
temperature is raised to 52C. The mass of the gas (b)182C
that has to be
Ice (c)256C
Hot bath
SRI MARUTHI P.U COLLEGE, HOSKOTE I PUC PART-2 CET/NEET STUDY
MATERIAL
Waves/Sound, Oscillations/SHM, Mechanical Properties of Matter/Elasticity, Fluid Mechanics (Fluid
Statics, Fluid Dynamics, Viscosity, Surface Tension),
Thermal Properties Matter/Thermometry (Thermal Expansion, Calorimetry, Transmission of Heat),
Thermo Dynamics, Kinetic Theory of Gases (KTG)
(d)546C P(2V) P1V P2V

RT RT1 RT2

P P P
1 2
T 2T1 2T2
Solution : (d)Quantity of gas in these bulbs is
constant i.e. Initial No. of 411. At the top of a mountain a thermometer
moles in both bulb = final number of moles reads 7C and a
barometer reads 70 cm of Hg. At the bottom of the
1 2 1' 2'
mountain these
read 27C and 76 cm of Hg respectively.
PV PV 1.5 PV 1.5PV Comparison of density of

R(273) R(273) R(273) R(T) air at the top with that of bottom is

2 1.5 1.5 7oC, 70 cm of Hg

273 273 T (a)75/76
. (b)70/76
T 819K 546C (c)76/75
410. Two containers of equal volume contain the (d)76/70 27oC, 76 cm of Hg
same gas at
pressures and and absolute temperatures
P1 P2 T1 Solution : (a) Ideal gas equation, in terms of
and density constant
T2 P1 P2 1 P T
1 2
respectively. On joining the vessels, the gas 1T1 2T2 2 P2 T1
reaches a common
pressure P and common temperature T. The ratio Top PTop TBottom 70 300 75
P/T is equal to
Bottom PBottom TTop 76 280 76
412.At room temperature, the rms speed of the
Initially molecules of certain
P1 T1 P2 T2 diatomic gas is found to be 1930 m/s. The gas is
V V
[IIT-JEE 1984; MP PET 2000;
BCECE 2003]
Finally
P T P T (a) (b) (c) (d)
V V H2 F2 O2 Cl 2

Solution :(a)Root means square velocity


(given)
3RT
(a) (b) vrms 1930m/s
M
P1 P2 P1T1 P2T2

T1 T2 (T1 T2)2 i.e. the
3RT 3 8.31 300
(c) (d) M 2 10 3 kg 2 gm
(1930) 2 1930 1930
P1T2 P2T1 P1 P
2
(T1 T2)2 2T1 2T2 gas is hydrogen.
413. Let A and B the two gases and given :
Solution : (d)Number of moles in first vessel
; where T
P1V TA T
1 4. B
RT1 MA MB
and number of moles in second vessel is the temperature and M is the molecular mass. If
P V and are
2 2
RT2 CA CB

If both vessels are joined together then quantity of the rms speed, then the ratio will be equal to
gas CA
remains same i.e CB
1 2 (a) 2 (b) 4 (c) 1 (d) 0.5
SRI MARUTHI P.U COLLEGE, HOSKOTE I PUC PART-2 CET/NEET STUDY
MATERIAL
Waves/Sound, Oscillations/SHM, Mechanical Properties of Matter/Elasticity, Fluid Mechanics (Fluid
Statics, Fluid Dynamics, Viscosity, Surface Tension),
Thermal Properties Matter/Thermometry (Thermal Expansion, Calorimetry, Transmission of Heat),
Thermo Dynamics, Kinetic Theory of Gases (KTG)
Solution : (a)As 416. The speeds of 5 molecules of a gas (in
3RT arbitrary units) are as
vrms
M follows : 2, 3, 4, 5, 6. The root mean square speed
for these

molecules is
CA TA / TB
42 (a)2.91 (b) 3.52 (c) 4.00 (d) 4.24
CB MA / MB
Solution:(d)
TA M v12 v22 v32 v42 v52 22 3 2 4 2 5 2 6 2
As 4 A given vrms
TB MB 5 5

414. The root mean square speed of hydrogen


molecules at 300 K is 100
20 4.24
1930 m/s. Then the root mean square speed of 5
oxygen molecules at 417. Gas at a pressure in contained as a vessel.
900 K will be P0
(a) (b)836 m/s(c)643 m/s(d) If the masses of
1930 3 m/ s 1930 all the molecules are halved and their speeds are
m/ s
3 doubled, the
Solution : (b) resulting pressure P will be equal to
3RT (a) (b) (c) (d)
vrms 4P0 2P0 P0 P0
M
2
Solution : (b)
vH 2 TH 2 M O2 1 mN 2
P vrms
vO2 M H2 TO2 3 V

2
P mvrms
1930 300 32

vO2 2 900 so
2 2
P2 m2 v2 m1 / 2 2v1
. 2
P1 m1 v1 m1 v1
1930 3
vO2 836m/s
4
P2 2P1 2P0
415. At what temperature is the root mean square
velocity of 418. Let and respectively denote the
gaseous hydrogen molecules is equal to that of v, vrms vmp
oxygen molecules at mean speed, root
47C mean square speed and most probable speed of
(a)20 K(b)80 K(c) 73 K(d)3 K the molecules in an
Solution : (a)For oxygen and ideal monoatomic gas at absolute temperature T.
3RTO2 The mass of a
vO2 molecule is m. Then
M O2
(a)No molecule can have speed greater than
For hydrogen 2 vrms
TH 2
vH 2 3R (b)No molecule can have speed less than
M H2
vmp/ 2
According to problem (c)
3RTO2 TH 2 vmp v vrms
3R
M O2 M H2 (d)The average kinetic energy of a molecule is
3 2
mvmp
4
TO2 TH 2 47 273 TH 2
Solution : (c, d)
M O2 M H2 32 2 We know that ,
. 3RT 8 RT
vrms vav
320 M M
TH 2 2 20 K
32
SRI MARUTHI P.U COLLEGE, HOSKOTE I PUC PART-2 CET/NEET STUDY
MATERIAL
Waves/Sound, Oscillations/SHM, Mechanical Properties of Matter/Elasticity, Fluid Mechanics (Fluid
Statics, Fluid Dynamics, Viscosity, Surface Tension),
Thermal Properties Matter/Thermometry (Thermal Expansion, Calorimetry, Transmission of Heat),
Thermo Dynamics, Kinetic Theory of Gases (KTG)
and possess potential energy so we can say that
RT absolute zero degree temperature is not zero
vmp 2 energy temperature
M
421.The ratio of rms speeds of the gases in the
mixture of nitrogen
vrms : vav : vmp 3 : 2.5 : 2
oxygen will be
so
vmp vav vrms (a)1 : 1(b) (c) (d)
3 :1 8: 7 6: 7
and or
vrms 3 2
vrms
3 2
vmp Solution : (c)

vmp 2 2 3RT vN2 M O2 32 8
vrms
M vO2 M N2 28 7
Average kinetic energy
1 2 1 3 2
mvrms m vmp 422. A vessel is partitioned in two equal halves by
2 2 2
a fixed diathermic
.
3 2 separator. Two different ideal gases are filled in left
mvmp
4 (L) and right (R)
halves. The rms speed of the molecules in L part is
419.The molecules of a given mass of a gas have equal to the
a rms velocity of
mean speed of molecules in the R part. Then the
200 m/sec at 27C and pressure.
1.0 105 N / m2 ratio of the mass of

When the a molecule in L part to that of a molecule in R part


temperature is 127C and pressure is is
0.5 105 N / m2
, the rms (a)
velocity in m/sec will be 3
(a) (b) (c) (d)None of these 2 L R

100 2 100 2 400 (b)


3 3 /4
Solution : (c)Change in pressure will not affect the (c)
rms velocity of molecules. 2/ 3
So we will calculate only the effect of temperature.
As (d)
3 / 8
vrms T

v300o 300 3
Solution :(d)Root means square velocity of
v400o 400 4 molecule in
. left part
200 3 200 2 400 3KT
v400 m/s vrms
v400 4 3 3 mL
420.Which of the following statement is true Mean or average speed of molecule in
(a)Absolute zero degree temperature is not zero right part
energy temperature 8 KT
vav
(b)Two different gases at the same temperature mR
pressure have equal root mean square velocities
According to problem
(c)The rms speed of the molecules of different
3KT 8 KT
ideal gases, maintained at the same temperature
mL mR
are the same
(d)Given sample of 1cc of hydrogen and 1cc of .
oxygen both at N.T.P.; oxygen sample has a large 3 8 mL 3

number of molecules mL mR mR 8
Solution : (a)At absolute temperature kinetic
energy of gas molecules becomes zero but they
SRI MARUTHI P.U COLLEGE, HOSKOTE I PUC PART-2 CET/NEET STUDY
MATERIAL
Waves/Sound, Oscillations/SHM, Mechanical Properties of Matter/Elasticity, Fluid Mechanics (Fluid
Statics, Fluid Dynamics, Viscosity, Surface Tension),
Thermal Properties Matter/Thermometry (Thermal Expansion, Calorimetry, Transmission of Heat),
Thermo Dynamics, Kinetic Theory of Gases (KTG)
423. An ideal gas ( = 1.5) is expanded (a)0.0015(b)0.003(c)0.048(d)0.768
adiabatically. How many Solution : (c)Average translational kinetic energy
does not depends
times has the gas to be expanded to reduce the
upon the molar mass of the gas. Different gases
root mean square
will possess same
velocity of molecules 2 times average translational kinetic energy at same
(a)4 times(b)16 times(c)8 times(d)2 times temperature.
Solution : (b)To reduce the rms velocity two times, 426. At constant temperature on increasing the
pressure of a gas by
temperature should be reduced by four times (As
5% will decrease its volume by
)
vrms T (a)5%(b)5.26%(c)4.26%(d)4.76%
Solution : (d)If then 5% of P = 1.05 P
, P1 P P2 P
T1 T T V1 V
T2 From Boyles law PV = constant
4
From adiabatic law constant
1 V2 P P 100
TV 1
V1 P2 1.05 P 105
we get
V2
1
T1
Fractional change in volume
4 V V2 V1
V1 T2
V V1
[ = 3/2 given]
1 100 105 5
V2
1 105 105
(4)
V1
Percentage change
in volume
1
V1(4)2 16V1 V 5
V2 V1(4) 21
3 / 100% 100% 4.76%
V 105
i.e. volume decrease by 4.76%.
V2
16 427. A cylinder contained 10 kg of gas at pressure
V1
. The
424 At which of the following temperature would 7
10 N / m 2
the molecules of a
gas have twice the average kinetic energy they quantity of gas taken out of cylinder if final
have at 20C pressure is

[MP PET 1992; is (assume the temperature of gas is


6
BVP 2003] 2.5 10 N / m
(a)40C(b)80C(c)313C(d)586C constant)
Solution : (c)
ET (a)Zero(b)7.5 kg (c) 2.5 kg(d)5 kg
Solution : (b)At constant temperature for the given
E 2 T2 2E1 T2
volume of gas
E1 T1 E1 (20 273)
P1 m
1
. P2 m2
T2 293 2 586K 313C

425. The average translational kinetic energy of
10 7
10 2.5 106 10
O2 m2 2.5 kg
2.5 106 m2 107
(molar mass 32)
molecules at a particular temperature is 0.048 eV. The quantity of gas taken out of the cylinder =
The translational 10 2.5 = 7.5 kg.
kinetic energy of (molar mass 28) molecules in 428. If a given mass of gas occupies a volume of
N2
10 cc at 1
eV at the same
atmospheric pressure and temperature of 100C
temperature is (373.15 K). What
SRI MARUTHI P.U COLLEGE, HOSKOTE I PUC PART-2 CET/NEET STUDY
MATERIAL
Waves/Sound, Oscillations/SHM, Mechanical Properties of Matter/Elasticity, Fluid Mechanics (Fluid
Statics, Fluid Dynamics, Viscosity, Surface Tension),
Thermal Properties Matter/Thermometry (Thermal Expansion, Calorimetry, Transmission of Heat),
Thermo Dynamics, Kinetic Theory of Gases (KTG)
will be its volume at 4 atmospheric pressure; the Solution : (a)As
temperature being VT V2 T2 313
V2 V1
the same V1 T1 293
(a)100 cc(b)400 cc(c)2.5 cc(d)104 cc Fraction of gas comes out
Solution : (c) .
1 V2 P 1
P 1 V2 10 2.5 cc 313
V V1 P2 4 V1 V1
V2 V1 293 20
0.07
429. An air bubble of volume is released by a V1 V1 293
V0
431. If pressure of a gas contained in a closed
fish at a depth h vessel is increased by
in a lake. The bubble rises to the surface. Assume 0.4% when heated by 1C, the initial temperature
constant must be
temperature and standard atmospheric pressure P 1982; EAMCET (Engg.) 1995;
RPMT 1996; MP PET 1999]
above the lake.
(a)250 K(b)250C(c)2500 K(d)25C
The volume of the bubble just before touching the
Solution : (a) , T1 = T ,
surface will be
P1 P
(density of water is )
(0.4% of P)
P2V2 P2 P 0.4 P T2 T 1
P PP
100 250
h From Gay Lussac's law
P1 T P T
1
(P1 V1) P2 T2 P T 1
P
(a) (b) 250
V0 [As V = constant for closed vessel]
(c) (d) By solving we get T = 250 K.
V0 ( gh/ P ) V0 gh 432. Temperature of an ideal gas is T K and
V0 1
gh P average kinetic energy is
1
P T Joule/molecule. Number of
23
Solution : (d)According to Boyles law E 2.07 10
multiplication of pressure and molecules in 1 litre
gas at S.T.P. will be
volume will remains constant at the bottom and (a) (b) (c) (d)
top. 2.68 1022 2.68 1025 2.68 1028 1.68 1022
If P is the atmospheric pressure at the top of the Solution : (a) As we know that at S.T.P. 22.4 litre of
lake and the volume of bubble is V then from
gas contains molecules
P1V1 P2V2 6.023 10 23

1 litre of gas contain


(P h g)V0 PV P h g
V V0 6.023 1023
P 2.68 1022
22.4
molecules.
gh
V V0 1 433.The average kinetic energy per molecule of
P
helium gas at
430. Hydrogen gas is filled in a balloon at 20C. If temperature T is E and the molar gas constant is
temperature is R, then Avogadros
made 40C, pressure remaining same, what
number is
fraction of hydrogen will
come out (a) (b) (c) (d)
RT 3RT E 3RT
(a)0.07(b)0.25(c)0.5(d)0.75
2E E 2RT 2E
SRI MARUTHI P.U COLLEGE, HOSKOTE I PUC PART-2 CET/NEET STUDY
MATERIAL
Waves/Sound, Oscillations/SHM, Mechanical Properties of Matter/Elasticity, Fluid Mechanics (Fluid
Statics, Fluid Dynamics, Viscosity, Surface Tension),
Thermal Properties Matter/Thermometry (Thermal Expansion, Calorimetry, Transmission of Heat),
Thermo Dynamics, Kinetic Theory of Gases (KTG)
Solution : (d)Average kinetic energy per unit volume , exerts a pressure of 1.40 MPa and
3
molecule 0.10 m
that in B of
3 2E volume exerts a pressure 0.7 MPa. The two
E kT k
2 3T 0.15 m3
But Avagadro number containers are
R R united by a tube of negligible volume and the
NA
k (2E / 3T) gases are allowed to
intermingle. Then it the temperature remains
. constant, the final
3RT pressure in the container will be (in MPa)
NA
2E (a)0.70(b)0.98(c)1.40(d)2.10
434. One mole of a gas filled in a container at Solution : (b)As the quantity of gas remains
N.T.P., the number of constant
A B
molecules in 1 cm3 of volume will be
(a) (b) PA VA PB VB P(VA VB )
23 23
6.02 10 / 22400 6.02 10 RT RT RT
(c) 1/22400 (d)
6.02 1023 / 76 PA VA PB VB 1.4 0.1 0.7 0.15
P
VA VB 0.1 0.15
Solution : (a) Number of molecule in 22.4 litre gas
at N.T.P. .
23 P 0.98 MPa
6.023 10
Or number of molecule in 437. A closed vessel contains 8g of oxygen and 7g
of nitrogen. The
22.4 103 cm3 6.023 1023
total pressure is 10 atm at a given temperature. If
[As 22.4 litre ] now oxygen is
22.4 103 cm3 absorbed by introducing a suitable absorbent the
pressure of the
Number of molecules in . remaining gas in atm will be
23
6.023 10 (a)2(b)10(c)4(d)5
1cm3
22400 Solution : (d)From Daltons law final pressure of
435. The capacity of a vessel is 3 litres. It contains the mixture of nitrogen and oxygen
6 gm oxygen, 8
gm nitrogen and 5 gm mixture at 27C. If R = P P1 P2 1 RT 2 RT m RT m2 RT
1
CO2 V V M1 V M2 V
8.31 J/mole ..(i)
kelvin, then the pressure in the vessel in will 8 RT 7 RT RT RT
10
N / m2 32 V 28 V 2V 2V
be (approx.) When oxygen is absorbed then for nitrogen let
(a) (b) (c) (d)
pressure is ..(ii)
5 105 5 104 106 105
7 RT RT
Solution : (a) Daltonslaw P P
28 V 4V
1RT 2 RT 3 RT RT RT m m m From equation (i) and (ii) we get pressure of the
P P1 P2 P3 [1 2 3 ] 1 2 3
V V V V V M1 M 2 M 3 nitrogen atm.
P5
438. Energy of all molecules of a monoatomic gas
. having a volume V
8.31 300 6 8 5

3 32 28 44
498 103 ~
500 103 ~
5 105 N/m2 and pressure P is . The total translational
3 10
3
PV
436. Two gases occupy two containers A and B the 2
gas in A, of kinetic energy of all
molecules of a diatomic gas as the same volume
and pressure is
SRI MARUTHI P.U COLLEGE, HOSKOTE I PUC PART-2 CET/NEET STUDY
MATERIAL
Waves/Sound, Oscillations/SHM, Mechanical Properties of Matter/Elasticity, Fluid Mechanics (Fluid
Statics, Fluid Dynamics, Viscosity, Surface Tension),
Thermal Properties Matter/Thermometry (Thermal Expansion, Calorimetry, Transmission of Heat),
Thermo Dynamics, Kinetic Theory of Gases (KTG)
(a) (b) (c) (d)3 PV Solution : (c)Given velocity of sound ,
1 3 5 m
PV PV PV vs 330
2 2 2 sec
Solution : (b)Energy of 1 mole of gas Density of gas , Atomic pressure
f f kg
RT PV 1.3
2 2 m3
where f = Degree of freedom
N
P 1.01 105
m2
Monoatomic or diatomic both gases posses equal
Substituting these value in we
degree of freedom for translational motion and
P
that is equal to 3 i.e. f = 3 vsound


3 get
E PV
2 1.41

Although total energy will be different, Now from we get


For monoatomic gas [As f = 3] 2 2 2
1 f 5.
3 f 1 1.4 1
E total PV
2
441. If the mean free path of atoms is doubled
For diatomic gas [As f = 5] then the pressure of
5
E total PV gas will become
2
(a) (b) (c) (d)P
(Q439) is a triatomic gas. Mean P /4 P /2 P /8
CO2(O C O)
Solution : (b)As
kinetic energy 1 kT

of one gram gas will be (If N-Avogadro's number, 2 d 2 P
k-Boltzmann's i.e. by increasing two times pressure
constant and molecular weight of ) 1
P
CO 2 44

(a) (b) will become half.


3 / 88NkT 5 / 88NkT 442. The mean free path of nitrogen molecules at
a pressure of 1.0
(c) (d)
6 / 88NkT 7 / 88NkT atm and temperature 0C is . If the
7
0.8 10 m
Solution : (d)Mean kinetic energy for mole gas
number of density
f of molecules is
. RT
2
, then the molecular diameter is
2.7 1025 perm3
7 7
RT
m 7 1 7
E NkT NkT NkT (a) (b) (c) (d)
2 M 2 44 2
88 3.2nm 3.2m 2.3mm
3.2
[As f = 7 and M = 44 for ] Solution : (b)Mean free path 0.8 107 m
CO 2

440. At standard temperature and pressure the number of


density of a gas is molecules per unit volume per m3
n 2.7 1025
1.3 gm/ m3 and the speed of the sound in gas is
330 m/sec. Then the Substituting these value in
1
degree of freedom of the gas will be
2nd2
(a) 3 (b) 4 (c) 5 (d) 6
we get
d 1.04 1019 3.2 1010 m 3.2
SRI MARUTHI P.U COLLEGE, HOSKOTE I PUC PART-2 CET/NEET STUDY
MATERIAL
Waves/Sound, Oscillations/SHM, Mechanical Properties of Matter/Elasticity, Fluid Mechanics (Fluid
Statics, Fluid Dynamics, Viscosity, Surface Tension),
Thermal Properties Matter/Thermometry (Thermal Expansion, Calorimetry, Transmission of Heat),
Thermo Dynamics, Kinetic Theory of Gases (KTG)
443. For a gas . This gas is made up of (a)8475 J/kg - K (b) 5186 J/kg - K
R (c)1660 J/kg - K (d) 10375 J/kg - K
0.67
Cv Solution : (d)Given ..(i)
molecules which c p cv 4150
are and ..(ii)
(a)Diatomic(b)Mixture of diatomic and polyatomic cp cp 1.4cv
molecules 1.4
cv
(c)Monoatomic (d)Polyatomic
Solution :(c)By comparing with relation By substituting the value of in equation (i)
R cp
Cv
1 we get
we get or = 1.67 i.e. the gas is 1.4cv cv 4150 0.4cv 4150
1 0.67
.
monoatomic. 4150
cv 10375J /kg- K
(Q444)40 calories of heat is needed to raise the 0.4
temperature of 1 447 Two cylinders A and B fitted with pistons
mole of an ideal monoatomic gas from 20C to contain equal
30C at a constant
amounts of an ideal diatomic gas at 300K. The
pressure. The amount of heat required to raise its piston of A is free to
temperature over
move while that of B is held fixed. The same
the same interval at
amount of heat is given
a constant volume is
to the gas in each cylinder. If the rise in
(R 2 caloriemole1 K 1 )
temperature of the gas in A
(a)20 calorie(b)40 calorie(c)60 calorie(d)80 calorie is 30 K, then the rise in temperature of the gas in
Solution : (a)At constant pressure B is
(Q) p C p T
(a)30 K (b) 18 K (c) 50 K (d) 42 K
Solution : (d)In both cylinders A and B the gases
1 C p (30 20) 40 calorie
Cp 4 are diatomic ( =
molekelvin
1.4).
Piston A is free to move i.e. it is isobaric process.
Cv C p R calorie
4 2 2 Piston B is fixed
mole kelvin
i.e. it is isochoric process. If same amount of heat
Now
(Q)v C v T 1 2 (30 20) 20calorie Q is given to both then
(Q)isobaric (Q)isochoric
445. At constant volume the specific heat of a gas
is then the
3R
, C p (T) A C v (T)B
2
value of will be
Cp
(T )B (T ) A (T ) A 1.4 30 42 K .
(a) (b) (c) (d)None of the above Cv
3 5 5
448 The specific heat at constant volume for the
2 2 3
monoatomic
Solution :(c)Specific heat at constant volume argon is 0.075 kcal/kg-K whereas its gram
(given) molecular specific heat
R 3R cal/mole/K. The mass of the argon atom
Cv
1 2 C v 2.98

. is (Avogadros
2 5 number molecules/mole)
1
3 3 6.02 1023
446 For a gas the difference between the two [MP PET 1993]
specific heats is 4150 (a) (b)
J/kg K. What is the specific heats at constant 6.60 1023 gm 3.30 1023 gm
volume of gas if the (c) (d)
ratio of specific heat is 1.4 2.20 1023 gm 13.20 1023 gm
SRI MARUTHI P.U COLLEGE, HOSKOTE I PUC PART-2 CET/NEET STUDY
MATERIAL
Waves/Sound, Oscillations/SHM, Mechanical Properties of Matter/Elasticity, Fluid Mechanics (Fluid
Statics, Fluid Dynamics, Viscosity, Surface Tension),
Thermal Properties Matter/Thermometry (Thermal Expansion, Calorimetry, Transmission of Heat),
Thermo Dynamics, Kinetic Theory of Gases (KTG)
Solution : (a)Molar specific heat = Molecular Joule/kelvin.
weight Gram specific heat Cv
80
4
20
C v M cv
451 A gas, is heated at constant pressure. The
fraction of heat
calorie kcal
2.98 M 0.075 supplied used for external work is
mole kelvin kg - kelvin
(a) (b) (c) (d)
0.075 103 calorie 1 1 1 1
M 1 1
103 gm kelvin

2
molecular weight of argon Solution : (b)We know fraction of given energy that
2.98 goes to increase
M 39.7 gm
0.075 the internal energy
i.e. mass of atom = 39.7 gm mass of 1

6.023 1023

single atom . So we can say the fraction of given energy that


39.7 supplied for external
23
6.60 10 23 gm
6.023 10 work .
1
449 When an ideal diatomic gas is heated at 1
constant pressure, the

fraction of the heat energy supplied which


increases the internal 452 A monoatomic gas expands at constant
energy of the gas is pressure on heating.
(a)2/5(b)3/5(c)3/7(d)5/7 The percentage of heat supplied that increases the
Solution : (d)When a gas is heated at constant internal energy of
pressure then its one the gas and that is involved in the expansion is
part goes to increase the internal energy and (a)75%, 25%(b)25%, 75%(c)60%, 40%(d)40%,
another part for work 60%
done against external pressure i.e. Solution : (c)Fraction of energy supplied for
(Q)p U W increment in internal energy
1 3

C p T C v T PV 5

So fraction of energy that goes to increase the 5


internal energy As 3 for monoatomicgas

[As for diatomic gas]
Percentage energy
U C 1 5 7
v 30
(Q)p Cp 7 5 60%
5
450 The temperature of 5 mole of a gas which Fraction of energy supplied for external work done
was held at constant
5
volume was changed from 100oC to 120oC. The 1
1 1 3 2
change in internal 1
5 5
energy was found to be 80 J. The total heat 3
capacity of the gas at
Percentage energy .
constant volume will be equal to 2
[CPMT 1988] 100% 40%
5
(a) (b) (c) (d)
453 The average degrees of freedom per molecule
8 J K 1 0.8 J K 1 4 J K 1 0.4 J K 1 for a gas is 6. The
Solution : (c)At constant volume total energy will gas performs 25 J of work when it expands at
be utilised in increasing the temperature of gas constant pressure. The
i.e. heat absorbed by gas is
(Q)v C vT C v (120 100) 80 (a)75 J(b)100 J(c) 150 J(d)125 J
SRI MARUTHI P.U COLLEGE, HOSKOTE I PUC PART-2 CET/NEET STUDY
MATERIAL
Waves/Sound, Oscillations/SHM, Mechanical Properties of Matter/Elasticity, Fluid Mechanics (Fluid
Statics, Fluid Dynamics, Viscosity, Surface Tension),
Thermal Properties Matter/Thermometry (Thermal Expansion, Calorimetry, Transmission of Heat),
Thermo Dynamics, Kinetic Theory of Gases (KTG)
Solution : (b)As f = 6 (given) Solution : (b)Ideal gas equation for m gram gas
2 2 4 [where r = Specific gas constant]
1 1
f 6 3 PV mrT

Fraction of energy given for external work or


m rT
W 1 P rT
1 V
Q
P 1.013 105
r 466.7
T 0.795 273
25 1 3 1
1 1
Q 4 / 3 4 4 Specific heat at constant volume

Q 25 4 100 Joule r 466.7 J


cv 1400
1 4 kg.kelvin
1
454 Certain amount of an ideal gas are contained 3
in a closed vessel.
The vessel is moving with a constant velocity v. 4
3 for polyatomicgas
The molecular mass
of gas is M. The rise in temperature of the gas (Q456)22 gm of at 27C is mixed with 16 gm
when the vessel is CO2
suddenly stopped is
of at 37C.
( C P / CV )
O2
(a) (b)
2 The temperature of the mixture is
Mv Mv2( 1)
2R( 1) 2R (a)32C(b)27C(c)37C(d)30.5C

(c) (d) Solution : (a)Let t is the temperature of mixture


Mv 2 Mv 2 Heat gained by = Heat lost by
2R( 1) 2R( 1) CO 2 O2

Solution : (b)If m is the total mass of the gas then


1C v1 T1 2C v2 T2
its kinetic energy
1
mv2
2
22 16 5
(3R)(t 27) R (37 t)
When the vessel is suddenly stopped then total 44 32 2
kinetic energy will
increase the temperature of the gas (because
5
process will be 3(t 27) (37 t)
2
adiabatic) i.e.
[As ] By solving we get .
t 32C
1 m R
mv2 C v T C v T Cv
2 M 1 457. A gas mixture consists of 2 mole of oxygen
and 4 mole of
.
m R 1 Mv 2 ( 1) argon at temperature T. Neglecting all vibrational
T mv2 T
M 1 2 2R modes, the total

455 The density of a polyatomic gas is standard internal energy of the system is
conditions is 0.795 (a)4 RT(b)15 RT (c) 9 RT(d) 11 RT
. The specific heat of the gas at constant Solution : (d)Total internal energy of system
3
kgm
f1 f
volume is U oxygen U argon 1 RT 2 2 RT
2 2
(a) (b)
5 3
930 J -kg1 K 1 1400 J - kg1 K 1 2 RT 4 RT 5 RT 6 RT 11RT
2 2
(c) (d)
[As f1 = 5 (for oxygen) and f2 = 3 (for
1120 J - kg1 K 1 925 J - kg1 K 1
argon)]
SRI MARUTHI P.U COLLEGE, HOSKOTE I PUC PART-2 CET/NEET STUDY
MATERIAL
Waves/Sound, Oscillations/SHM, Mechanical Properties of Matter/Elasticity, Fluid Mechanics (Fluid
Statics, Fluid Dynamics, Viscosity, Surface Tension),
Thermal Properties Matter/Thermometry (Thermal Expansion, Calorimetry, Transmission of Heat),
Thermo Dynamics, Kinetic Theory of Gases (KTG)
xxxxx The end xxxxx

You might also like